Download as pdf or txt
Download as pdf or txt
You are on page 1of 205

1 Subject-Verb Agreement

Then three massive turtles come out of the sea. You Because major Japanese corporations 3 recruits new
watch as 1 each slowly dig a hole and fill it, one egg at employees by arrangement from particular universities
a time, until there are over a hundred eggs. year after year, getting into the right university is

1 important for students.

A) NO CHANGE 3
B) they each slowly dig a hole and fill A) NO CHANGE
C) they each slowly digs a hole and fills B) recruit
D) they slowly dig a hole and fills C) do recruits
D) recruiting

Acceptance into the best Japanese high 2 schools


however, are highly competitive.

A) NO CHANGE
B) schools, however, is
C) schools, however, are
D) schools however, is

1
2 Subject-Verb Agreement

A recent trend among college students 4 involve their As a result, the competition for roles in commercials 7
own organizing of public readings of works ranging from have been fierce.
John Milton’s seventeenth-century epic poem Paradise 7
Lost to Toni Morrison’s recent prize-winning novel
A) NO CHANGE
Beloved. B) were
4 C) are
D) is
A) NO CHANGE
B) involves their
C) involve they’re
D) involves they’re Performers in Hollywood and New York compare
commercial auditions to a lottery. The average
performer 8 is casting in only a tiny proportion of the
Nearly every public ritual in our lives 5 include commercials auditioned for.
someone reading aloud from a shared text. 8
5 A) NO CHANGE
A) NO CHANGE B) is cast
B) include somebody C) cast
C) includes someone D) cast off
D) includes them

Both the extremes of darkness and light and the


Each species 6 being a different size, shape, color, and constantly shifting day length 9 effects people in
temperature. subtle and curious ways.

6 9

A) NO CHANGE A) NO CHANGE
B) having B) affects
C) has C) effect
D) DELETE the underlined portion. D) affect

CONTINUE
Subject-Verb Agreement 3

The narrow cobbled streets and wide paved avenues of The Navajo language is complex, with a structure and
Katmandu, Nepal, 10 buzzes with the interplay of old sounds that 13 makes them unintelligible to anyone
and new ways. without extensive exposure to it.

10 13

A) NO CHANGE A) NO CHANGE
B) buzz B) makes it
C) is buzzing C) make it
D) was buzzing D) make them

Leon says that one of the most challenging and One of the great stories of the Civil Rights Movement 14
rewarding accomplishments of her life 11 have been to have been that this musically gifted young woman
compose an opera and then conduct it herself. developed into a leader of national stature.

11 14

A) NO CHANGE A) NO CHANGE
B) have been composing B) were
C) has been to compose C) are
D) has been composing D) is

Tradition has it that the first fireworks were created in Those constructed according to Grant’s design fit so
the eleventh century when Chinese soldiers blasted well and so comfortably that the general health of his
their gunpowder into the sky to celebrate a victory in patients 15 have been measurably improved.
battle. The science of pyrotechnics 12 have changed 15
considerably since then, and today audiences can enjoy
A) NO CHANGE
special effects that were undreamed of even a century ago. B) was
12 C) are
D) were
A) NO CHANGE
B) has changed
C) had changed
D) change

CONTINUE
4 Subject-Verb Agreement

Garbage cans must now be placed on the front sidewalk She could now hear the thrum and whir of the bee
and collected from the main street. What was once colony 19 sitting in her living room.
hidden away behind the houses 16 are now out in the 19
open for all to see.
A) NO CHANGE
16 B) as she sat
C) seated
A) NO CHANGE
D) being seated
B) is
C) were
D) have been
Although pharmaceutical companies 20 that begun
to produce the drug determined that taxol could be
As the potter removes one brick at a time to create an extracted from the bark of all species of yew, their
opening into the oven, an expanding view of gleaming attention soon focused on the Pacific yew.
shapes 17 rewards the artist for months of hard work. 20
17 A) NO CHANGE
A) NO CHANGE B) who’d began producing
B) rewarding C) who begun producing
C) reward D) that began to produce
D) as a reward for

Perhaps this is because 21 being a writer and I never


Rosie must 18 have heard us talking, because she began really feel comfortable without a pen in my possession.
to amble over to us. 21
18 A) NO CHANGE
A) NO CHANGE B) being that I’m
B) have heard of C) my being
C) of heard about D) I’m
D) of heard

CONTINUE
Subject-Verb Agreement 5

Equipment for both types of kayaks 22 are similar, and The works in Moua’s anthology 25 records the
fairly simple. challenges of adjusting to a new homeland while

22 preserving the writers’ ethnic traditions.

A) NO CHANGE 25
B) is A) NO CHANGE
C) were B) has recorded
D) was C) recording
D) record

I had known I would be teaching English to


teenagers 23 whom were studying to become The students also experiment with form and function
everything from shipbuilders to chemical engineers. by building their own shelters. One student 26 who

23 creates a rectangular structure from donated glass, wood,


and steel to span a dry riverbed.
A) NO CHANGE
B) whom are 26
C) who were
A) NO CHANGE
D) who have
B) who created
C) creating
D) created
Furthermore, highway maintenance costs and 24
pollution decreases in areas planted with native flowers
and grasses, which require little, if any, mowing or
chemical treatment.

24

A) NO CHANGE
B) pollution decrease
C) pollution has decreased
D) also decreasing pollution

CONTINUE
6 Subject-Verb Agreement

I was immediately overwhelmed with images of my Almost every diving center in the Florida Keys 30 have
great-grandfather. I remembered the way he 27 is used boats equipped to take divers to the coral reef that lies
to read to me when I was very young. close offshore along Florida’s Atlantic coast.

27 30

A) NO CHANGE A) NO CHANGE
B) used to B) have boats and equipped
C) use to C) has boats equipped
D) used a D) has boats that equip

Their sheer number—thousands upon thousands of Her efforts to deter crime 31 have been praised by
geese coming down from Hudson Bay in Canada— 28 many community leaders, who admire her calm style in
are astonishing. dealing with problems.

28 31

A) NO CHANGE A) NO CHANGE
B) have been B) has been
C) were C) was
D) is D) would have been

One of the welding certificates on view 29 were


donated by Bethena Moore.

29

A) NO CHANGE
B) are
C) was
D) have been

CONTINUE
Subject-Verb Agreement 7

His photos of Memphis’s Beale Street jazz and blues Signs around the Mauna Lani lagoon 34 warn tourists
musicians 32 includes the likes of B. B. King, Aretha to keep their distance from the humped creatures that
Franklin, and Elvis Presley. look like gray-green boulders along the shoreline.

32 34

A) NO CHANGE A) NO CHANGE
B) does include B) warns
C) including C) is warning
D) include D) has warned

Though the 33 practice of bartering have ancient


origins, Time Dollars began in 1980 as a means of
encouraging and rewarding community involvement.

33

A) NO CHANGE
B) practice of bartering have had
C) practices of bartering has
D) practice of bartering has

CONTINUE
2 Pronouns

You know that this liquid in the turtles’ eyes has a Yet the wedding Mom dreams of performing will take
scientific explanation: 1 they keep they’re eyes moist place at the foot of a ski run. She imagines my brother
and clear of particles. and his bride skiing down the mountain to join 2 she

1 and the guests.

A) NO CHANGE 2
B) it keeps its A) NO CHANGE
C) it keeps their B) her and the guests.
D) they keep its C) their guests and she.
D) there guests and herself.

8
Pronouns 9

Whether performing Verdi at the Metropolitan Opera Tan also realized that, whether her work was published
or singing folk spirituals on 3 one of those concert or not 6 one could only learn about oneself by writing
tours, Anderson embraced her audiences with the same in her own, Chinese-American, voice.
largeness of spirit. 6
3 A) NO CHANGE
A) NO CHANGE B) one could only learn about herself
B) many of one’s C) she could only learn about herself
C) one out of her D) oneself could only be known
D) any one of those

7 Its impossible to predict which people will dig


History is not merely remembering the good that came into their pockets or open their purses, and I’ve stopped
before; 4 what’s nostalgia. trying to guess.

4 7

A) NO CHANGE A) NO CHANGE
B) its B) It’s
C) that’s C) Its’
D) thats D) That’s

A culture willing to confront its flaws can begin to find The Navajo language is complex, with a structure and
remedies 5 for it. sounds that 8 makes them unintelligible to anyone

5 without extensive exposure to it.

A) NO CHANGE 8
B) for them. A) NO CHANGE
C) of them. B) makes it
D) for themselves. C) make it
D) make them

CONTINUE
10 Pronouns

Joe is seven, living in those two or three years when 9 My parents seldom bothered to fold up or store away
they can manage to throw a baseball a few feet but when their errand cart in a closet. They kept the cart by the
what they’re really interested in are things closer at hand: front door, ready for when they needed to haul laundry
bugs, butterflies, dirt (if they’re in the infield), and grass down the long basement corridor to the washing
(if they’re in the outfield). machines or to carry flour, chocolate, and walnuts to my

9 grandmother’s Popham Avenue apartment, where my


sister preferred to bake. 12 They also use it to wheel
A) NO CHANGE
B) children books to and from the public library.
C) he 12
D) some of them
A) NO CHANGE
B) There
C) Their
Once they’ve picked it up, they look at it quizzically for D) They’d
a while and then look up to see what all the ruckus is
about. 10 She usually sees a bleacher section full of
parents, each adult frantically holding up a stiff arm. You walk into a clothing store and ask for a dress

10 shirt—a simple enough thing to do. But what if you are


a bilingual speaker in a bilingual city such as Miami, in
A) NO CHANGE
B) He usually sees which almost 35% of the population is Hispanic? How
C) They usually see would 13 they decide which language to use?
D) DELETE the underlined portion and begin the
sentence with a capital letter. 13

A) NO CHANGE
B) those
Quills fasten deeply, so the removal of a quill stub C) he
D) thats
before 11 it is ready to be naturally shed would be very
painful to the animal.

11

A) NO CHANGE
B) they are
C) those are
D) that is

CONTINUE
Pronouns 11

Hearing a Venezuelan singer’s latest recording being By her own account, she took the wrong approach to
played on 14 its sound system might prompt a bilingual gathering folklore when, in 1927, she first returned to
speaker to use Spanish. the South. When she asked 17 them if they knew any

14 folktales, they glanced at her nice car and “city” clothes,


shook their heads, and told her to look elsewhere.
A) NO CHANGE
B) a store’s 17
C) their
A) NO CHANGE
D) it’s
B) some of them
C) people
D) DELETE the underlined portion.
Before you can entertain any second thoughts, the plane
has taxied and taken off and is circling upward into the
huge cobalt sky. 15 Its headed to an altitude of three Other times, the only sound in the kitchen would be the
thousand feet. fan, rotating on 18 its’ squeaky hinge while sending a

15 breeze our way.

A) NO CHANGE 18
B) Its’ A) NO CHANGE
C) It is B) it’s squeakier
D) It was C) its squeaky
D) it’s squeaking

Behind the disk stands a granite wall bearing 16 their


inscription: “…UNTIL JUSTICE ROLLS DOWN LIKE This undertaking wasn’t their dream but perhaps 19
WATERS AND RIGHTEOUSNESS LIKE A MIGHTY their dream would lead to it.
STREAM.” 19
16 A) NO CHANGE
A) NO CHANGE B) they would lead to that.
B) which C) would lead it to that.
C) this D) would lead to it.
D) it’s

CONTINUE
12 Pronouns

One Saturday afternoon, 20 her and I transferred the 23 I enjoy spending evenings at a local cafe with my
bees—and their honey—to a wooden hive.
classmates. My classmates and I enjoy talking and
20
studying together.
A) NO CHANGE
23
B) she and I
C) me and her Which choice most effectively combines the underlined
D) she and me sentences?
A) My classmates and me enjoy spending evenings at a
local café, talking and studying together.
B) Enjoying talking and studying together, we spend
Furthermore, the uncluttered background directs the evenings at a local café.
C) I enjoy spending evenings, talking and studying
viewer to the woman herself as she looks off into the
together, at a local café, with my classmates.
distance as if 21 one is waiting for someone. D) I enjoy spending evenings at a local cafe with my
classmates, talking and studying together.
21

A) NO CHANGE
B) they were waiting
Most kayaks are made of rubberized cloth, molded
C) one waited
D) waiting plastic, or fiberglass. 24 It is covered except for the
opening in which the paddler or paddlers sit.

24
Some sixty years later, an elderly Frances Griffiths
A) NO CHANGE
publicly admitted that 22 her and her cousin had B) One is
staged the photographs as a practical joke. C) They are
D) Which are
22

A) NO CHANGE
B) her cousin and herself
C) she and her cousin
D) her cousin and her

CONTINUE
Pronouns 13

These days I enjoy hot peppers, but 25 one was still During World War II, a group of Navajo soldiers
surprised occasionally by their bite. developed a code that became one of the most successful

25 in U.S. military history. The Navajo language is complex,


with a structure and sounds that make it unintelligible to
A) NO CHANGE
B) one still is anyone without extensive exposure to it. Outside Navajo
C) you still are communities, such exposure is rare, which greatly
D) I am still contributed to 28 it’s success.

28

Still emphasizing self-help as a route to self-esteem, 26 A) NO CHANGE


B) that
her and her colleagues created a Department of
C) this
Commerce to coordinate future business development D) the Navajo code’s
with the needs of existing enterprises.

26
Although she narrowly lost bids for the U.S. Senate in
A) NO CHANGE
B) Mankiller and her colleagues created 1982 and 1986, Woods’s campaigns earned 29 their
C) created by Mankiller and her colleagues was national recognition.
D) Mankiller’s colleagues and her created
29

A) NO CHANGE
B) herself
Three of the children became successful artists, another a
C) them
composer, and 27 one who was a scientist and inventor. D) her
27

A) NO CHANGE
B) the other
C) one becoming
D) another being

CONTINUE
14 Pronouns

I simply feel better knowing a pen is handy. Who knows Banneker’s grandmother was an indentured servant
when 30 they will run dry? from England 33 who, after completing the term of her

30 contract, bought some land and then married a freed


slave.
A) NO CHANGE
B) the one I’m using 33
C) something
A) NO CHANGE
D) either one
B) whom,
C) which,
D) DELETE the underlined portion.
When Tran visited Vietnamese restaurants in the Los
Angeles area, he was impressed with both their décor
and their menus, but shocked to learn how much it costs After all, it’s not what we know that makes us smart, but
to rent a suitable building. He knew that 31 himself how 34 one chooses to use that knowledge.
and Maya would have to work a long time for the money 34
needed to sign a lease.
A) NO CHANGE
31 B) we choose
C) we chose
A) NO CHANGE
D) I chose
B) Maya and himself
C) Maya and him
D) he and Maya
The microlending institution, such as Women’s World
Banking, makes a small loan (as little as $5) to, for
Amil brought Stormy and Morgan, two enormous mares, example, an artisan or a food vendor 35 whom would
out of the stable. When all was ready, he told me to not qualify for a loan from a commercial bank.
get on 32 them. He showed me how by getting onto 35
Morgan.
A) NO CHANGE
32 B) whose
C) for whom
A) NO CHANGE
D) who
B) her.
C) Stormy.
D) the horse.

CONTINUE
Pronouns 15

In the year 2000, a Rosie the Riveter memorial was He didn’t think she had the strength to pass the
dedicated. At the height of the war, the workforce at the grueling physical tests required for police recruits. So,
shipyards grew to 100,000, and more than one-fourth Harvard, 38 whom held a master’s degree in urban
of laborers were women. About 200 of 36 them were government and administration, bet him that she could
able to attend the dedication ceremonies as the guests of pass the tests and then signed on as a police recruit.
honor. 38
36 A) NO CHANGE
A) NO CHANGE B) whom had
B) the females C) by having
C) which D) who held
D) those women

Undaunted, she thought of ways to improve the patrols


Beginning scuba divers should gain confidence before and 39 suggested it to her supervisors.
trying spearfishing so they won’t panic when 37 you 39
actually attract a barracuda.
A) NO CHANGE
37 B) it was suggested by her
C) they were suggested by her
A) NO CHANGE
D) suggested them
B) one actually attracts
C) they actually attract
D) us divers actually attract
Though some Nebraskans complain about the growing
number of 40 visitors whom come to observe the
annual crane migration, many focus their attention on
the birds themselves.

40

A) NO CHANGE
B) visitors
C) visitors who
D) visitors, whom

CONTINUE
16 Pronouns

Would it be easier to buy mushrooms at the store? Perhaps because of the depth of her understanding of
Absolutely. But it wouldn’t be as much fun. Once you football, my grandmother is not impressed by pampered
find your first morel, maybe by a dead elm or in an superstars and overpaid quarterbacks. Instead, her
old apple orchard, 41 a person will be even more loyalties lie with dedicated, hardworking athletes—team
determined to find the next one. And the next. And so players 44 whom, of course, play to win.
on. 44
41 A) NO CHANGE
A) NO CHANGE B) which
B) a morel hunter C) who,
C) you D) DELETE the underlined portion.
D) DELETE the underlined portion.

Just as the sun is behind you when you see a solar


The American craze for the toy began when the rainbow, the moon is behind 45 you when you see a
entrepreneur Donald Duncan saw a demonstration of lunar rainbow.
Flores’s new yo-yo. Noticing the large crowd 42 whom 45
watched, Duncan quickly realized the yo-yo’s potential.
A) NO CHANGE
42 B) it when it is
C) us if they see
A) NO CHANGE
D) you if they are
B) that
C) whose
D) who’s
My friends and 46 me, however, ignore the signs, wade
into the water, and struggle to flip a 150-pound turtle
On the way, I watched the shorebirds as 43 they hunted onto its back in the center of an oversized inner tube.
for their own treasures. 46
43 A) NO CHANGE
A) NO CHANGE B) I, however,
B) it C) I however,
C) these D) me, however
D) those
3 Tense, Mood, and Voice

You watch as the turtles each slowly dig a hole and fill it, Komiyama translates as “small mountain view.” No
one egg at a time, with over a hundred eggs. The eggs are other name 2 could of been more appropriate for Ms.
bright white and about two inches in diameter. You 1 Komiyama, who has lived alone on a mountaintop near
were watching as each turtle then slowly, laboriously, Tokyo for about fifty years.
buries the eggs by turning in circles, thus pushing sand 2
back into the holes with surprisingly efficient flippers.
A) NO CHANGE
1 B) could be
C) could been
A) NO CHANGE
D) would of been
B) watch
C) had watched
D) watched
The average performer is cast in only a tiny proportion of
the commercials 3 auditioning for.

A) NO CHANGE
B) to audition
C) auditioned
D) auditions

17
18 Tense, Mood, and Voice

Summer days have an endless quality. Everything 1 You sink into despair. The house you thought you owned
has seemed possible: people want to work or play all has turned on its master. Worse yet, there’s no going
night. back; the old hot water pipes have been ripped out, and a

4 jagged, four-foot hole in the roof 7 has turned into an


unplanned and precarious skylight.
A) NO CHANGE
B) was 7
C) seems
A) NO CHANGE
D) seemed to be
B) was turning
C) turned
D) would have turned
In the home of Amy Tan’s editor, a large blowup of a 1991
New York Times best-seller list 5 was hung on the
wall. The number one novel, Amy Tan’s The Kitchen God’s Wyeth was against formal education and taught his
Wife, is boldly circled. Tan is considered one of America’s children at home. Wyeth believed a richly textured
best authors. childhood, both disciplined and fanciful, 8 would lay

5 the foundation for productive adult years.

A) NO CHANGE 8
B) hangs A) NO CHANGE
C) hung B) were laying
D) was hanging C) had lain
D) DELETE the underlined portion.

Writing about those years 6 was given Tan a way of


understanding her past—and of coming closer to her She enjoyed singing throughout her childhood, however,
mother. she 9 chosen to study piano when she entered college.

6 9

A) NO CHANGE A) NO CHANGE
B) has gave B) had chose
C) had gave C) has chosen
D) gave D) chose

CONTINUE
Tense, Mood, and Voice 19

Along with more than 100,000 other Japanese Meanwhile, the runners continue to score. They score,
Americans, Ichiro was uprooted from his home that is, if 13 they were not to be distracted by the
and incarcerated after the bombing of Pearl Harbor grown-ups—or the butterflies.
unfairly 10 cast suspicion on an entire population. 13
10 A) NO CHANGE
A) NO CHANGE B) they would not have been
B) casted C) they’re not
C) had casted D) they’re not to be
D) has cast

What did my dad think a baby 14 could of done with


Fourteen years after the book’s publication, Okada died that huge ball?
without knowing that his work 11 is revived for an 14
enthusiastic audience.
A) NO CHANGE
11 B) would of done
C) would done
A) NO CHANGE
D) could do
B) has been
C) would be
D) DELETE the underlined portion.
Their first child was born in 1955, just three weeks before
Dr. King 15 has led the historic Montgomery bus
Before employing the Navajo code talkers, American boycott.
military officials 12 have been using cumbersome 15
machines to encode and relay information during battles.
A) NO CHANGE
12 B) leads
C) lead
A) NO CHANGE
D) led
B) had
C) would have
D) will have

CONTINUE
20 Tense, Mood, and Voice

Her first job was as a newspaper reporter; she moved to Lin continues to challenge herself in a career that
St. Louis to write for the Globe-Democrat. In 1953 she 16 combines an appreciation for sculpture, architecture, and
was to marry a fellow reporter and left the newspaper to the complexity of human experience. A public library in
raise their children. Ohio, a chapel in Pennsylvania, a train station in New

16 York—these 19 had been only some of the places that


have been influenced by Maya Lin.
A) NO CHANGE
B) would marry 19
C) married
A) NO CHANGE
D) would be married to
B) were
C) are
D) having been
It is common to hear 10 Spanish spoken in Miami’s
Little Havana, where many Cuban immigrants have
settled in recent decades. As people gathered at the store her first day there, a

17 wonderful outpouring of tales 20 begun as members of


the crowd vied for their turn to tell stories.
A) NO CHANGE
B) Spanish, spoken 20
C) Spanish, spoke
A) NO CHANGE
D) Spanish spoke
B) would begun
C) had began
D) began
The Globe audience heard hundreds of other expressions
that are now household words. In Henry V, Shakespeare
used this very term to describe how the names of King
Henry’s war heroes 18 becomes as well known to
English people as simple words used in any household.

18

A) NO CHANGE
B) had become
C) have became
D) becoming

CONTINUE
Tense, Mood, and Voice 21

Sometimes we sang along with the radio, frequently In the early twentieth century, as cameras became easier
substituting the word tomato for words in the song. to use and more affordable, amateur photographers
Other times, the only sound in the kitchen would be the began capturing scenes and subjects that 24 had
fan, rotating on its squeaky hinge while sending a breeze previously been neglected.
our way. Patches of salt 21 forming on our faces and 24
arms as our sweat evaporated into the August air.
A) NO CHANGE
21 B) have previously been
C) are previously being
A) NO CHANGE
D) are previously
B) would form
C) form
D) DELETE the underlined portion.
Little is known of Earl Mo Williams except that he
worked as an elevator operator at a department store
The successful bidder 22 donating the sack of flour and later as a photofinisher. Fortunately, he 25 spends
back to Gridley to be auctioned off again. his spare time among Lincoln’s residents, capturing

22 moments on film that remain vivid today.

A) NO CHANGE 25
B) could of donated A) NO CHANGE
C) can donate B) is spending
D) donated C) spent
D) has spent

McLean didn’t play according to the rules of the popular


music 23 I was growing up listening to on the radio.

23

A) NO CHANGE
B) I had grown up listening to
C) growing up and listening to
D) I grew up and listened

CONTINUE
22 Tense, Mood, and Voice

Scientific curiosity is often motivated by observable At first, I was alarmed, but the horse was moving slowly,
phenomena. Sometimes though, as with the research of and I realized that I 28 would of been able to hang on.
entomologist Dr. Lanna Cheng, that curiosity is fueled by 28
what 26 seem to be absent.
A) NO CHANGE
26 B) would be
C) will be
A) NO CHANGE
D) should of been
B) seems
C) seemed
D) have seemed
There’s increasing agreement with the view that “good
neighbors keep their noise to themselves.” Of course,
Through her research on Halobates and other insects, many people 29 had insisted that they have a right to
Dr. Cheng has considered—and then dismissed—three make noise on their own property. But do they?
hypotheses concerning the scarcity of marine insects. 29
The first 27 offering the ocean’s high salt content as an
A) NO CHANGE
explanation, but Dr. Cheng has found species of flies that B) insisted
can tolerate more than three times the salt content of the C) were insisting
sea. D) will insist

27

A) NO CHANGE Many municipalities have 30 begun to adopt


B) has been offered
ordinances that limit levels and kinds of noise.
C) offers
D) will offer 30

A) NO CHANGE
B) began
C) beginning
D) begin

CONTINUE
Tense, Mood, and Voice 23

As the dancers step to the music, they 31 were also Sidestepped by progress and 34 frozen in time, Virginia
stepping in time to a sound that embodies their unique City is now a living historical museum, preserving a slice
history and suggests the influence of outside cultures on of frontier life.
their music. 34
31 A) NO CHANGE
A) NO CHANGE B) froze
B) are also stepping C) freezed
C) have also stepped D) frozed
D) would also step

During the Moon Festival, I would show off the lanterns


I usually carry a fistful of pens in my backpack. I simply my mother had made, eat mooncake, and drink tea with
feel better 32 to know a pen is handy. my grandparents. In the summer, we walked together

32 on the beach. I looked forward to the ocean air and the


sound of the waves. My mother 35 walks with me on
A) NO CHANGE
B) to know that the wet sand, often buying me something from one of the
C) known that shops along the beach.
D) knowing
35

A) NO CHANGE
B) would walk
For six or seven decades, this is the way 33 things
C) has walked
would have worked. Garbage trucks squeezed through D) was walking
the back alleys, collecting trash.

33

A) NO CHANGE
B) things were to have worked.
C) that things work.
D) things worked.

CONTINUE
24 Tense, Mood, and Voice

The miscommunication that occurred between my My aunt said that she 39 would have checked with
neighbor and the waiter 36 was not related to grammar, my parents, who each agreed that if I wanted to take
pronunciation, or vocabulary. responsibility for Rosie, I could take her home with me.

36 39

A) NO CHANGE A) NO CHANGE
B) not being related B) had checked
C) relates not C) would check
D) relating not D) must check

37 Walking through any one of an increasing number Based on those drawings, Banneker designed the works
of vegetable gardens these days and you’re likely to shake for his own clock and carved the gears, wheels, and other
your head in disbelief. moving parts. The clock 40 keeps precise time for—can

37 you believe it?—over forty years.

A) NO CHANGE 40
B) Having walked A) NO CHANGE
C) To walk B) was keeping
D) Walk C) kept
D) still keeps

Seed savers point out that the world’s food supply


relies on fewer than two dozen species of plants, nearly Early in the semester, I opened the bag of battered
all of which are being reduced to only a few varieties negatives and 41 began to examine them.
through the use of hybrids and plant genetics. That lack 41
of variety 38 had made the plants more vulnerable to
A) NO CHANGE
rapid climate change, insects, and disease. B) had began
38 C) begun
D) begin
A) NO CHANGE
B) was making
C) makes
D) made

CONTINUE
Tense, Mood, and Voice 25

In 2002, Minnesota writer and editor Mai Neng Moua My mother’s faint handwriting covered the outside of the
created a new literary tradition with the publication of package. Fragile, she 44 writes next to my address and
the first-ever anthology of Hmong American writers. along each of the sides.
This collection, Bamboo Among the Oaks, 42 features 44
the fiction, nonfiction, poetry, and drama of twenty-two
A) NO CHANGE
Hmong Americans. B) were to write
42 C) had it written
D) had written
A) NO CHANGE
B) feature
C) featuring
D) featured: After high school graduation, with ten shillings 45
given to her by her aunt, she bought oranges and made
twelve jars of marmalade, which she then sold on the
The school flourishes today, with two campuses—one in street at a nice profit.
Spring Green, Wisconsin, and the other near Scottsdale, 45
Arizona, in the Sonoran Desert. The students 43
A) NO CHANGE
learning about the art and science of architecture, and B) were given
they perform the school’s daily maintenance tasks such C) gave
as cooking, serving, and cleaning up in the school’s D) were gave

kitchen.

43
So, at the 1975 Mexico City gathering that opened
A) NO CHANGE the United Nations Decade for Women, Ocloo, with
B) learned
like-minded women, 46 will have developed a
C) learn
D) who learn “microlending” plan.

46

A) NO CHANGE
B) developed
C) will develop
D) have developed

CONTINUE
26 Tense, Mood, and Voice

Ornette Coleman had something new to offer. His Many monarchs west of the Rocky Mountains migrate
approach to jazz 47 involved improvisation guided by to the southern California coast, where they 50
a personal expression of emotions and a freedom from overwintered in eucalyptus groves.
fixed harmonic chord structures and tempos. 50
47 A) NO CHANGE
A) NO CHANGE B) were overwintering
B) was involving C) overwinter
C) had involved D) overwinters
D) involves

In the 1920s Pedro Flores, a Filipino immigrant,


When farms were 48 first settling, it was the building started a yo-yo manufacturing company in California.
of a barn that announced to the world that the builders The American craze for the toy 51 began when the
were planted on a piece of land and intended to stay. entrepreneur Donald Duncan saw a demonstration of

48 Flores’s new yo-yo.

A) NO CHANGE 51
B) first settled, A) NO CHANGE
C) settling, B) begins
D) having first settlers, C) begun
D) had began

Between 100,000 and 500,000 birds gather here for a six-


week rest during their long journey from Texas and New Yo-yo technology made a leap forward in the 1970s when
Mexico to the Arctic Circle. This gathering 49 brought designers added weighted rims so the toy would spin for
together the largest concentration of cranes to be found a longer time. In 1985, this most ancient of 52 toys,
anywhere on the globe. went into space.

49 52

A) NO CHANGE A) NO CHANGE
B) had brought B) toys went
C) would bring C) toys had went
D) brings D) toys, had gone

CONTINUE
Tense, Mood, and Voice 27

Immersed in the icy water off the Antarctic Peninsula, After we pull the heavy turtle back to the water, I don
Lynne Cox wasn’t sure if she could accomplish her fins, a mask, and a snorkel in order to watch the turtle
goal to be the first person to swim a mile through the after it’s released. Kicking my fins, 55 I follow the turtle
glacier-strewn sea. At forty-five, she 53 would of been as it beats its flippers like wings and soars through the
training for two years for this event, and she hoped her canyon reefs toward the deep water beyond.
preparations would pay off. 55
53 A) NO CHANGE
A) NO CHANGE B) I followed
B) had C) were I to follow
C) have D) if I follow
D) had to of

Before retiring, she 54 has worked for many years as a


waitress.

54

A) NO CHANGE
B) has been working
C) would have worked
D) worked
4 Possessive Determiners

You sit in the bright silver moonlight on a beach 1 A culture willing to confront 3 its flaws, can begin to
where its 10,000 miles from home. find remedies for them.

1 3

A) NO CHANGE A) NO CHANGE
B) and its B) it’s flaws
C) it’s C) it’s flaws,
D) DELETE the underlined portion. D) its flaws

You notice that the 2 turtles’ eyes are covered with a The same goes for the 4 abolition, women’s rights, and
shiny liquid. It looks as if they’ve been crying. civil rights movements.

2 4

A) NO CHANGE A) NO CHANGE
B) turtle’s eyes B) abolition, womens’ rights,
C) turtles eyes C) abolition women’s rights
D) turtles’ eye’s D) abolition, womens rights

28
Possessive Determiners 29

With all that natural beauty, 5 it isn’t hard to Sandra Cisneros, perhaps the best known Latina author
understand what has kept Taeko Komiyama there for all in the United States, writes poems and stories whose
these years. titles alone—“Barbie-Q,” “My Lucy Friend Who Smells

5 Like Corn,” “Woman Hollering Creek”—engage 8


potential readers’ curiosity.
A) NO CHANGE
B) its not 8
C) its’ not
A) NO CHANGE
D) there isn’t
B) potential, reader’s
C) potential, readers
D) potential readers
The words of a novel or poem are literally “bound”
within 6 there pages.

6 While my friend Sam and I were on a trip to Bombay,


we stopped at a small factory, where we had a very
A) NO CHANGE
B) its interesting tour. During that time, I talked to the
C) one’s workers—with my 9 friend’s help—in Hindi.
D) these
9

A) NO CHANGE
B) friends help—
The Fleer Corporation 7 named it’s new product
C) friends’ help—
Dubble Bubble. D) friends help
7

A) NO CHANGE
B) named its Some desperate parents line their 10 childrens
C) called they’re bedroom window’s with aluminum foil to convince them
D) called it’s that it is dark outside, and time to go to bed.

10

A) NO CHANGE
B) children’s bedroom windows
C) childrens bedroom’s windows
D) childrens’ bedroom windows’

CONTINUE
30 Possessive Determiners

On 11 Katmandus busiest commercial avenue, under When she was just four years old, she would tune the 14
billboard advertisements for toothpaste and movies, a families radio to classical music and then listen intently
sidewalk artist chalked heroes from the country’s oldest to every note.
myths. 14
11 A) NO CHANGE
A) NO CHANGE B) family’s radio,
B) Katmandus’ C) families radio,
C) Katmandu’s D) family’s radio
D) Katmandu’s most

Giant ferns, 15 who’s lush foliage I previously


Katmandu has been influenced by change from outside, associated with impenetrable jungles, grow in all my
but 12 its Nepalese culture thrives. neighbors’ yards.

12 15

A) NO CHANGE A) NO CHANGE
B) it’s B) whose
C) their C) of who’s
D) its’ D) of whose

The dried-up 13 roses also led me to philosophize with Since the house I rent contains a glassed-in atrium with
a metaphor: life is a pale rose, brief and beautiful. a little goldfish pond and many plants, I particularly

13 value the 16 lizards ability to control the mosquito


population.
A) NO CHANGE
B) rose’s also 16
C) roses also,
A) NO CHANGE
D) roses, also
B) lizards’ ability
C) lizards ability,
D) lizards’ ability,

CONTINUE
Possessive Determiners 31

Today is 17 my mother’s birthday. Many bilingual teenagers in Miami feel more

17 comfortable speaking English than Spanish; the reverse


is often true of their parents. Consequently, teenagers
A) NO CHANGE
B) her birthday. may use English if the salesperson is their own age but
C) mine mother’s birthday. Spanish (as a sign of respect) if the salesperson belongs
D) my mothers birthday. to 20 their parents generation.

20

Eventually, it will be shed, allowing a new, complete A) NO CHANGE


B) their parents’
feather to 18 grow in it’s place.
C) there parents’
18 D) there parent’s

A) NO CHANGE
B) grow in its
C) have grow in its While the harmonica was originally marketed as a 21
D) have grow in it’s child’s toy, it eventually became a valued portable
instrument for folk and blued musicians.

21
After that, she was elected lieutenant governor, becoming
the first 19 woman in Missouri history, to hold a A) NO CHANGE
B) child’s toy;
statewide office.
C) childs’ toy
19 D) childs toy, that

A) NO CHANGE
B) woman in Missouri’s history
C) woman, in Missouri’s history Iago falsely hints that Desdemona, the wife of Othello, is
D) woman, in Missouri history in love with 22 one of Othellos’ officers.

22

A) NO CHANGE
B) one of Othellos
C) one, of Othellos’
D) one of Othello’s

CONTINUE
32 Possessive Determiners

To save the trees and still provide for the 23 patient’s In one portrait, a 26 porch’s railings and columns frame
needs, scientists began to produce taxol synthetically in a woman on one side, and a house frames the other.
the laboratory from the tree’s leaves. 26
23 A) NO CHANGE
A) NO CHANGE B) porches’
B) needs’ of the patients, C) porches
C) patients’ needs, D) porch
D) patients needs,

The first theory offers the 27 oceans high, salt content


Mr. Reuel Gridley, a grocer, entered into a friendly as an explanation, but Dr. Cheng has found species of
wager on the election’s outcome with a Dr. Buel Herrick. flies that can tolerate more than three times the salt
The 24 bets terms stated that the loser of the wager, content of the sea.
escorted by a marching band, must carry a fifty-pound 27
sack of flour one mile along Austin’s Main Street.
A) NO CHANGE
24 B) oceans high
C) ocean’s high
A) NO CHANGE
D) ocean’s high,
B) bets terms’
C) bet’s terms
D) bets’ terms
The second hypothesis states that the ocean is too deep
and underwater pressure too great for insect larvae to
For my friends Tran and Maya Nguyen, the distance complete 28 it’s life cycle.
between Vietnam and California is more than 25 it’s 28
measure in miles.
A) NO CHANGE
25 B) there
C) they’re
A) NO CHANGE
D) their
B) its’
C) its
D) his

CONTINUE
Possessive Determiners 33

I had followed 29 Amil’s initial instructions only to Furthermore, the board offered no evidence to back
realize he had not told me what to do next. up 32 it’s claim that informal clothing created a

29 negative educational environment.

A) NO CHANGE 32
B) Amils’ initial instructions, A) NO CHANGE
C) Amils initial instructions B) they’re
D) Amils’ initial instructions C) its
D) ones

As the dancers step to the music, they are also stepping


in time to a sound that embodies 30 their unique One day, Elsie Wright and Frances Griffiths returned
history and suggests the influence of outside cultures on from a walk in the English countryside with news that
their music. they had seen fairies. They had even taken photographs

30 that showed several of the tiny sprites, some dancing in a


ring in the grass, some fluttering in front of the 33 girl’s
A) NO CHANGE
B) they’re faces.
C) it’s 33
D) its’
A) NO CHANGE
B) girls’ faces.
C) girls faces.
The practice of advancing a 31 person’s age seems to D) girls face’s.
me to reflect the value a society places on life experience
and longevity.

31 Amalia began studying under Mexican folklorist Luis


Felipe Obregon. She learned that her 34 countrys
A) NO CHANGE
B) persons’ age folklore was a fusion of Aztec influences.
C) persons age 34
D) people’s age
A) NO CHANGE
B) countries’
C) country’s
D) countries

CONTINUE
34 Possessive Determiners

I often find myself in this situation—sheepishly handing An avid golfer, Dr. Grant’s invention of the golf tee in
back some one 35 else’s pen. 1884 was revolutionary. Before 38 that invention,

35 golfers preparing to tee off (hit their first shot) had to


shape a mound of moist dirt or sand to raise the golf ball
A) NO CHANGE
B) elses’ off the turf in order to hit it cleanly.
C) elses 38
D) else
A) NO CHANGE
B) their
C) its’
My pen habit, like most habits, is not without 36 it’s D) it’s
negative consequences.

36
Those constructed according to 39 Grants design fit
A) NO CHANGE
B) there so well and so comfortably that the general health of his
C) their patient was measurably improved.
D) its
39

A) NO CHANGE
B) Grant’s design
In the 1960s, this became reality when the U.S. Navy
C) Grant’s design,
deployed 37 its first undersea habitat, SeaLab. D) Grants design,
37

A) NO CHANGE
B) there By December, the population had skyrocketed to
C) their hundreds of thousands of residents, the Montana Territory
D) they’re was established, and nearby Virginia City became 40 it’s
first incorporated town and territorial capital.

40

A) NO CHANGE
B) its
C) its’
D) their

CONTINUE
Possessive Determiners 35

My neighbor was surprised at the reaction. The waiter Many of the plants are heirloom varieties 44 whose
had interpreted my 41 neighbors question as a simple seeds have been privately passed from one generation to
request for information, not as a request to bring the the next.
dish. 44
41 A) NO CHANGE
A) NO CHANGE B) who’s
B) neighbors’ question C) whom
C) neighbor’s question D) who
D) neighbors question,

I learned that I should build plywood enclosures in each


Both prairie and woods were dear to me and important of my 45 parents’ backyards so that Rosie would be safe
to the development of 42 one’s interactions with the year-round.
world. 45
42 A) NO CHANGE
A) NO CHANGE B) parent’s backyards
B) their C) parents backyards
C) your D) parents backyards,
D) my

Let us study an 46 outfielders face, without the aid


My 43 tastes’ are similarly diverse, whether in art, of binoculars, when he’s sprinting toward the infield to
literature, music, movies, TV shows, or long walks. catch a short fly ball.

43 46

A) NO CHANGE A) NO CHANGE
B) taste’s B) outfielder’s face,
C) tastes C) outfielders’ face,
D) tastes, D) outfielders faces,

CONTINUE
36 Possessive Determiners

We would burn paper money for good luck and go from Soon, the 50 faces’ of my mother’s family began to
house to house wishing 47 one’s relatives health and appear on the photographic paper.
happiness. 50
47 A) NO CHANGE
A) NO CHANGE B) faces of my mother’s
B) our C) faces’ of my mothers
C) ours D) faces of my mothers
D) their

I recalled my mother’s description of the long, hard


Benjamin Banneker, African American inventor and hours spent working in the migrant camps and the peach
astronomer, grew up on his 48 familys’ farm in colonial and apricot fuzz that prickled 51 it’s skin.
Maryland. 51
48 A) NO CHANGE
A) NO CHANGE B) there
B) family’s C) her
C) families’ D) its
D) families

The sound of the distant honking of these majestic 52


He had dismantled a pocket watch borrowed from a birds always makes me look up.
traveling merchant, made detailed drawings of 49 it’s 52
components, and returned it—fully functioning—to the
A) NO CHANGE
merchant. B) birds,
49 C) bird’s
D) birds’
A) NO CHANGE
B) its’
C) its
D) their

CONTINUE
Possessive Determiners 37

Coleman’s approach involved improvisation guided Because people in each region of the country and each
by a personal expression of emotions and a freedom ethnic group have tended to build different types of
from fixed harmonic chord structures and tempos. barns, these structures help define a landscape and
Eventually, 53 their style caught the attention of other tell 55 its story.
adventurous young players. 55
53 A) NO CHANGE
A) NO CHANGE B) it’s
B) his C) there
C) theirs D) the
D) they’re

Scuba diving is one of the 56 country’s fastest-growing


The memorial was designed by landscape sports.
architect Cheryl Barton and photographer Susan 56
Schartzenberg. 54 Its’ various features create a sense
A) NO CHANGE
of the size and shape of the cargo vessels, called Liberty B) country’s fastly growing
Ships, that were built in Richmond. C) countries fastest-growing
D) countries’ fastest-growing
54

A) NO CHANGE
B) Its
C) It’s
D) Their

CONTINUE
38 Possessive Determiners

Penfield had discovered a circular pattern resembling an Smaller and paler than the average pinecone, a morel
impact crater in 57 rocks lying one mile underground. blends perfectly into 59 its natural surroundings.

57 59

A) NO CHANGE A) NO CHANGE
B) rocks, laying B) it’s
C) rocks’ laying C) their
D) rocks’ lying D) there

Beverly Harvard—the chief of police in Atlanta and the When she was fifteen, she broke the men’s and
first African American woman to lead a major 58 citys women’s 60 record’s for swimming the English Channel
police department—began her career in law enforcement by finishing the twenty-seven-mile swim in fewer than
in an unusual way: she answered a challenge made by her ten hours.
husband. 60
58 A) NO CHANGE
A) NO CHANGE B) records
B) cities C) records’
C) cities’ D) records,
D) city’s

CONTINUE
Possessive Determiners 39

Eventually, Rogers became a success in New York When a moonbow appears in front of you, 63 its center
as a lasso-twirling 61 comedian who’s droll, folksy lies in the direction exactly opposite that of the moon.
monologues amused large audiences. 63
61 A) NO CHANGE
A) NO CHANGE B) it’s
B) comedian whose C) its’
C) comedian, who’s D) at it’s
D) comedian who

I open the 64 turtle’s mouth so I can document what it


62 Although Will’s father wanted his son to stay on the has been eating, and wrinkle my nose.
family ranch, Will wanted to experience other ways of 64
life.
A) NO CHANGE
62 B) turtles’
C) turtles
A) NO CHANGE
D) turtles’s
B) Although, Will’s
C) Although, Wills
D) Although Wills
5 Word Questions

There is perhaps no 1 competent example of her Interested students can sign up for half-hour shifts of
ability to reach out to an audience than the concert she reading aloud, and many students come—bringing
gave on Easter Sunday in 1939. pillows and pizzas—and 2 manage to stay around to

1 hear the entire book.

A) NO CHANGE 2
B) more A) NO CHANGE
C) better B) handle
D) good C) direct
D) regulate

40
Word Questions 41

Diemer had no knowledge of chemistry but had been The odd green shade of artificial turf is a poor 6 relief
working by trial and error for more than a year. Four for green grass.
months after his discovery, the 3 formation was nearly 6
perfected and the first large batch of bubble gum was
A) NO CHANGE
made. B) substitute
3 C) cover
D) reserve
A) NO CHANGE
B) forming
C) founding
D) formula For the patient, persistent, and lucky performer, the
financial rewards more than 7 compensate for the
many disappointments.
Despite the obstacles that faced her in school, Cisneros 7
completed not only high school, but also college. Her
A) NO CHANGE
persistence paid off in her twenties, when Cisneros was B) restore
admitted to the 4 prestigious Writers’ Workshop at C) relieve
the University of Iowa. D) repair

A) NO CHANGE As the sun comes up later and later each winter morning,
B) prodigal
many people’s 7 initiation lowers.
C) prodigious
D) promissory 8

A) NO CHANGE
B) initiate
Cisneros felt 5 decided out of place. C) initiative
D) initiator
5

A) NO CHANGE
B) deciding
C) decidedly
D) decidedly and

CONTINUE
42 Word Questions

Not surprisingly, there is no satisfactory method for Unfortunately, Mrs. Okada had burned the unpublished
governing rock music. Parents can best 9 expel manuscript after her 12 offer to donate her husband’s
responsibility for their children’s listening habits by papers to the University of California in Los Angeles was
communicating their concerns. rejected.

9 12

A) NO CHANGE A) NO CHANGE
B) exert B) exchange
C) exclude C) assumption
D) exceed D) advice

Originality was not blessing my thoughts as I gardened. For many centuries, fireworks came in only two colors,
My little experiment was producing a neat yard 10 or amber and silver, which were emitted by burning black
bad poetry. powder. Contrast that with the kaleidoscope of hues

10 produced by today’s shells, including crimson, orange,


emerald, turquoise, and gold. Just as 13 gifted have
A) NO CHANGE
B) although been advances in the design of fireworks.
C) so 13
D) but
A) NO CHANGE
B) stimulating
C) unnerving
11 Irritated by this desire for adventure, Leon moved D) impressive
to New York City in her early twenties.

11

A) NO CHANGE
B) Prompted
C) Obliged
D) Annoyed

CONTINUE
Word Questions 43

Today is my mother’s birthday. I decided to 14 stupefy In a T-ball league, one needs to do something to keep
her with lilac blooms from my backyard, so this the score from reaching triple digits in the early going.
morning, carrying a shopping bag full of the flowers, I There’s a rule, 17 therefore, that says the runner must
boarded a crowded “E” train and rode it to the very last stop when any fielder from the other team picks up the
stop in the Bronx. ball and holds it aloft.

14 17

A) NO CHANGE A) NO CHANGE
B) surprise B) instead,
C) confound C) likewise,
D) befuddle D) meanwhile,

Before Navajo code talkers were employed, American If a feather breaks off with the stub of its hollow quill
military officials had been using 15 cumbersome shaft still in place, the bird’s body mistakenly believes the
machines to encode and relay information during battles. feather is 18 solid.

15 18

A) NO CHANGE A) NO CHANGE
B) thorny B) integral.
C) strenuous C) total.
D) gawky D) intact.

Though Smith has been interested in theater since her Alligator jaws 19 contain seventy to eighty conical
youth, her 16 certification of her own talent grew teeth and have a closing power of up to 1,200 pounds per
gradually. square inch.

16 19

A) NO CHANGE A) NO CHANGE
B) endorsement B) control
C) recognition C) embrace
D) recollection D) encompass

CONTINUE
44 Word Questions

She tunes in her favorite talk show and 20 gets to work. It’s often hard to determine who first 23 uttered a given

20 word and almost as hard to know who first wrote it


down.
A) NO CHANGE
B) establishes 23
C) complies
A) NO CHANGE
D) institutes
B) said aloud audibly
C) said verbally
D) gave vocalization to
My dad insists that the endless hours he spent kicking
the ball to me when I was five or six helped me to 21
develop my soccer skills. It was while she was studying anthropology at Barnard

21 College in New York that Hurston decided to 24


reattach with the customs, songs, stories, and games she
A) NO CHANGE
B) progress had learned as a child.
C) emerge 24
D) flourish
A) NO CHANGE
B) reconnect
C) reestablish
Because of Woods’s history of public outreach, she D) adhere
was 22 named president of the National Women’s
Political Caucus, an organization that seeks to bring
women into the political process. Mr. Reuel Gridley, a grocer, entered into a friendly wager

22 on the election’s outcome with a Dr. Buel Herrick. That


wager would 25 benefit many Civil War soldiers in an
A) NO CHANGE
B) titled unforeseen way.
C) presumed 25
D) graced
A) NO CHANGE
B) further
C) boost
D) heal

CONTINUE
Word Questions 45

If I walked, it would take longer to get to my destination Many people might be surprised to learn that the
when running errands, but I consoled myself with 26 American way of computing a person’s age differs from
reassuring thoughts of how my body could benefit from the traditional Korean system, in which a person is
the effort. considered to be already one year old at the time of his or

26 her birth. As a child growing up in two cultures, I found


this 29 contest a bit confusing.
A) NO CHANGE
B) affectionate 29
C) soft
A) NO CHANGE
D) protective
B) change
C) dispute
D) difference
I’ve thought many times of the 27 instinctive way our
bodies respond to the beat of music by wanting to get
up and dance, or how a parent quiets a crying baby by Perhaps the celebration of New Year’s Day in Korean
walking back and forth. culture is 30 heightened because it is thought of as

27 everyone’s birthday party.

A) NO CHANGE 30
B) arbitrary A) NO CHANGE
C) prescribed B) raised
D) calculated C) lifted
D) lighted

Over the years, I have 28 gained a reputation among


family and friends as a honeybee enthusiast. Even something as 31 visibly simple or natural as

28 computing a person’s age can prove to be not so clear-cut.

A) NO CHANGE 31
B) elevated A) NO CHANGE
C) compelled B) apparently
D) assisted C) entirely
D) fully

CONTINUE
46 Word Questions

The school board members believed that wearing “play Hidden under the waves, Aquarius is a bright yellow,
clothes” to school made the students 32 inefficient forty-three-foot-long cylinder. Inside, scientists live
toward their school work, while more formal attire in homey but cramped quarters, with bunks, a shower
established a positive educational climate. and toilet, a microwave, and 35 an array of scientific

32 instruments.

A) NO CHANGE 35
B) lazy A) NO CHANGE
C) obstinate B) a classification
D) indifferent C) an arrearage
D) a taxonomy

She learned that her country’s folklore was a 33 fusion


of Aztec influences. Unfortunately, a golf ball would often roll off a handmade

33 mound before the golfer could complete his or her swing.


By both raising and stabilizing the ball, Grant’s invention
A) NO CHANGE
B) grouping of a tee 36 remedied this problem.
C) association 36
D) cluster
A) NO CHANGE
B) allocated
C) resorted
I usually carry a fistful of pens in my backpack. Maybe D) rescued
I’m afraid an important thought will come to me and I’ll
have no way to 34 record it.

34

A) NO CHANGE
B) reflect
C) refer
D) review

CONTINUE
Word Questions 47

Garbage cans must now be placed on the front sidewalk The hot, dry wind blowing off the desert seemed to 40
and collected from the main street. What was once melt right through me. I suddenly wished I could duck
hidden away behind the houses is now out in the open back inside the air-conditioned 747 and head back home.
for all to see. How disappointed those early town leaders 40
would be to see this public 37 display of trash.
A) NO CHANGE
37 B) simmer
C) blast
A) NO CHANGE
D) ignite
B) attraction
C) production
D) presentation
When I remember that human beings share many of
the same hopes and fears, dreams and disappointments,
Many seed savers order endangered plants through a then I can more easily form new relationships and fortify
small but growing number of organizations that 38 old ones. Perhaps this is a small thing, but no less 41
standardize in rare seeds. significant than mastering a new skill or suddenly

38 understanding a concept.

A) NO CHANGE 41
B) specialize A) NO CHANGE
C) concentrate B) solid
D) confine C) sensitive
D) susceptible

Poisonous waters at temperatures up to 360 degrees


Celsius gush from cracks in the ocean floor, and A 42 common theme in Moua’s collection of literary
thermophiles 39 convert the toxic chemicals into food works is the search for identity.
for crabs, giant worms, and other deep-sea life. 42
39 A) NO CHANGE
A) NO CHANGE B) collaborating
B) revamp C) habitual
C) confiscate D) colluding
D) renovate

CONTINUE
48 Word Questions

As a uniquely African American musical form, jazz 43 Providing a festive 46 conclusion to a perfect day,
impressed ready to play a role in the struggle for freedom a troupe of flamenco dancers was performing in the
and equality. amphitheater at the edge of town.

43 46

A) NO CHANGE A) NO CHANGE
B) roused B) destiny
C) appeared C) fate
D) performed D) termination

Early each spring, a ribbon of land along the Platte River By donating their time and talents to others through
in Nebraska is a temporary home to one of the world’s nonprofit Time Dollar Exchange programs, participants
most spectacular wildlife displays: the migration of are contributing to their communities and 47 fostering
the sandhill cranes. This gathering brings together the interaction.
largest 44 heap of cranes to be found anywhere on the 47
globe.
A) NO CHANGE
44 B) upgrading
C) reforming
A) NO CHANGE
D) sponsoring
B) concentration
C) bunch
D) supply
We float our latest turtle ashore. Flippers flapping and
beak snapping, the turtle resists as we hoist it onto the
That’s a crucial part of the preparation for morel hunting, examination table that we’ve set up beneath an awning
because often the same woods that 45 yield morels on the beach. It’s my turn to 48 enact the exam.
produce poisonous mushrooms, too. 48
45 A) NO CHANGE
A) NO CHANGE B) act out
B) capitulate C) perform
C) submit D) operate
D) relent
6 Conventional Expressions

My mother is a justice of the peace; that means she has When my sons and I visit the ballpark we are 3
the power to 1 perform weddings. essential on a picnic.

1 3

A) NO CHANGE A) NO CHANGE
B) put on the ball and chain. B) more essentially on
C) hook couples up in wedlock. C) more essential to
D) do the nuptial thing. D) essentially on

It has been said that Marian Anderson’s concerts were 2 My friend, a young man who had lived in Bombay his
much like communal celebrations than singing events. entire life 4 except for a few years spent studying in

2 the United States, walked with me to the post office.

A) NO CHANGE 4
B) more as A) NO CHANGE
C) more like B) but for the exception of
D) DELETE the underlined portion. C) with the accepting of
D) yet for

49
50 Conventional Expressions

Despair, however, soon gives 5 up on adaptation. You Children of that age still think nothing of doing little
pound a few nails, maybe patch over that hole in the dances in the outfield, often with their backs to home
roof. plate and, consequently, the batter. It’s not 8 as if the

5 outfielders’ positions matter much, though—the ball


never gets hit hard enough to reach there.
A) NO CHANGE
B) away 8
C) out of
A) NO CHANGE
D) way to
B) if
C) whether
D) as to whether
I recognized, however, that her advice was prompted by
my complaints 6 about writer’s block.

6 It has been 9 for ten years since those days, but I can
still remember how good I felt every time I won the ball
A) NO CHANGE
B) by from Max.
C) from 9
D) to
A) NO CHANGE
B) over
C) more then
A teenager who’s holding a shoebox containing a kitten D) passed
as tiny as a gingersnap smiles 7 even if a line of girls in
frilly white communion dresses file by.

7 Every day I feel a spindly, ringing connection 10 along


with my parents as I wheel the vegalah over the pavement
A) NO CHANGE
B) as to my new home.
C) whereas 10
D) such that
A) NO CHANGE
B) for
C) with
D) of

CONTINUE
Conventional Expressions 51

Each afternoon, trucks drove over a loose utility-hole We also find 14 illusions of Shakespeare’s words in film.
cover, and the clanking noise awoke the boys. Woods 14
appealed 11 on her city council to close the street to
A) NO CHANGE
trucks, but the council stalled. B) illusions to
11 C) allusions of
D) allusions to
A) NO CHANGE
B) by
C) in
D) to The Harlem Renaissance author Zora Neale Hurston
is best known for her novels, especially Their Eyes
Were Watching God, however, she was also a devoted
As a result of her success 12 to bring community chronicler 15 of African American folklore.
members into the legislative process, Woods was asked to 15
fill a vacant seat on the city council.
A) NO CHANGE
12 B) with
C) for
A) NO CHANGE
D) on
B) and bringing
C) bringing
D) in bringing
Hurston also visited Polk Country, Florida, where she
sponsored contests for the most imaginative tall tale.
13 You now can entertain any second thoughts, the The winners received prizes, and Hurston collected a 16
plane has taxied and taken off and is circling upward into wide vastness of folklore.
the huge cobalt sky. 16
13 A) NO CHANGE
A) NO CHANGE B) tremendous prosperity
B) You C) major chunk
C) Then you D) wealth
D) Before you

CONTINUE
52 Conventional Expressions

On special occasions, I reach for a can of tomatoes from My attention focused 20 first with Higgins, who played
my mother, twist off the top, and let the smell of summer with his head tipped back, smiling and nodding.
drift back 17 to me from my childhood. 20
17 A) NO CHANGE
A) NO CHANGE B) initially with
B) with C) first on
C) before D) initially
D) along

When I process text at my computer, I remember


In April 1864, during the Civil War, the Nevada mining my father at his Linotype machine. Compared to his
town of Austin held its first mayoral election. Mr. Reuel workplace, my office is quiet and clean. But in essence,
Gridley, a grocer, entered into a friendly wager 18 for my father’s work and mine are 21 the most alike.
the election’s outcome with a Dr. Buel Herrick. 21
18 A) NO CHANGE
A) NO CHANGE B) the same.
B) of C) more of the same.
C) on D) like the same.
D) at

In spring, I’d rejoice in the first crocuses emerging 22


For my friends Tran and Maya Nguyen, the distance 19 under the snow, the faint misting of green on tree
between Vietnam and California is more than its branches just beginning to bud, and the colorful
measure in miles. explosion of tulips and daffodils.

19 22

A) NO CHANGE A) NO CHANGE
B) among B) from under
C) from C) outside
D) to D) inside

CONTINUE
Conventional Expressions 53

So now when I want to escape the busyness of my life, The court agreed with Kevin that a person’s right 10 for
I take a walk. No matter how far I go, I know that I will wearing clothing of his or her own choosing is, in fact,
return 23 refreshed in both body and mind. protected by the Fourteenth Amendment.

23 26

A) NO CHANGE A) NO CHANGE
B) refreshing in B) of wearing
C) a refreshment of C) to wear
D) refreshed with D) wearing

In the meantime, I told her, I would be happy to come by Kevin’s challenge initiated a review of students’ rights and
and size 24 up to her situation. administrative responsibility 27 in public education.

24 27

A) NO CHANGE A) NO CHANGE
B) up B) on
C) the extent C) with
D) DELETE the underlined portion. D) about

Little is known of Earl Mo Williams 25 accept that he Her father, a prominent rancher and politician, did
worked as an elevator operator at a department store and not approve of his daughter dancing. But Amalia was
later as a photofinisher. persistent. 28 Despite this, her father acquiesced,

25 allowing, even helping, her to pursue her dream.

A) NO CHANGE 28
B) except that A) NO CHANGE
C) except for B) Finally,
D) accepting C) In fact,
D) On the other hand,

CONTINUE
54 Conventional Expressions

Because the pressure inside the habitat is matched to the Judge Murphy had been called to serve 32 as the
pressure of the ocean depth 29 outside, divers can enter magistrate of Virginia City.
and exit at will without fear of decompression sickness— 32
called “bends.”
A) NO CHANGE
29 B) for
C) like
A) NO CHANGE
D) to
B) at the exterior,
C) on the reverse side,
D) beyond,
Sarah found work as a hotel chambermaid. In 1873,
she married William Brown, but her husband passed
Before that invention, golfers preparing to tee off (hit away 33 less than a decade later.
their first shot) had to shape a mound of moist dirt or 33
sand to raise the golf ball off the turf in order 30 to hit
A) NO CHANGE
it cleanly. B) less then
30 C) fewer than
D) fewer then
A) NO CHANGE
B) for hitting
C) to hitting
D) that hit My neighbor did not understand that requests are
communicated 34 different from other cultures.

34
Grant recognized a need and focused his skills,
A) NO CHANGE
imagination, and intelligence 31 to devising a way to B) different than
fill that need. C) differently in
D) differently than
31

A) NO CHANGE
B) at
C) on
D) for

CONTINUE
Conventional Expressions 55

Holm claims 35 at having a prairie eye, feeling My friend Ellen is typical 37 of many more potters in
uncomfortable in enclosed, mysterious spaces like the that she shapes some pieces on a spinning potter’s wheel
woods; he prefers the “magnitude and delicacy” of the and builds others on a work table from coils or slabs of
open prairie. clay.

35 37

A) NO CHANGE A) NO CHANGE
B) to having B) of many
C) to have C) mostly of
D) by having D) for most

Seed savers point out that the world’s food supply relies Although I love my aunt, I wasn’t happy about the
on 36 fewer than two dozen species of plants, nearly prospect of three days at her house with nothing to do.
all of which are being reduced to only a few varieties It turns out I was in for a surprise. 38 Presently after I
through the use of hybrids and plant genetics. arrived, my aunt said she had a gift for me.

36 38

A) NO CHANGE A) NO CHANGE
B) lesser than B) Short while
C) fewer then C) As soon
D) less then D) Soon

My woods eye and prairie eye provide varied reactions to


more than just landscapes; these perspectives apply 39
to my other interests as well.

39

A) NO CHANGE
B) themselves for
C) myself to
D) for

CONTINUE
56 Conventional Expressions

I’d not only helped people in Morocco but also learned Frank Lloyd Wright, one of the twentieth century’s
Arabic, developed lifelong friendships, celebrated Islamic most creative and innovative architects, based most of
holidays, and come to understand and appreciate a his designs on two theories. One is his mentor Louis
culture 40 different from my own. Sullivan’s idea that “form follows function”— 43 this is

40 a building’s use determines its design.

A) NO CHANGE 43
B) differently from A) NO CHANGE
C) differently than B) that is,
D) different then C) this is,
D) that is

She strongly believed 41 if humans were to benefit


immeasurably from beauty in their environment. Here’s how it works: A large lending institution lends

41 the initial money, $5 million or more, to a microlending


institution. Then the microlending institution, such as
A) NO CHANGE
B) because if humans Women’s World Banking, makes a small loan (as little as
C) that if humans $5) to, 44 in contrast, an artisan or a food vendor who
D) that humans would not qualify for a loan from a commercial bank.

44

She is pleased that, 42 within her anthology, readers A) NO CHANGE


B) for example,
nationwide are being introduced to the writing of
C) as a result,
Hmong Americans. D) instead,
42

A) NO CHANGE
B) throughout
C) through
D) for

CONTINUE
Conventional Expressions 57

Next came Coleman’s controversial but successful 1959 This mass extinction may also have been the key 48 to
debut with his quartet at a New York club called the Five the evolution in another life form: mammals.
Spot. This engagement—coupled 45 for the release of 48
two more albums (The Shape of Jazz to Come and The
A) NO CHANGE
Change of the Century)—reversed Coleman’s fortunes. B) for the evolution to
45 C) to the evolution of
D) of the evolution to
A) NO CHANGE
B) with
C) up
D) DELETE the underlined portion. Such praise does not come 49 any easier to any big-city
police chief, but Harvard applies her talents and works
twelve- to sixteen-hour days to earn it.
A 46 growth number of people are working to reverse 49
that trend.
A) NO CHANGE
46 B) easily
C) more easy
A) NO CHANGE
D) easier
B) grown
C) growing
D) to grow
When I close my eyes, I see them. They pop up through
dead leaves, emerge from under fallen logs, and sprout
These streamlined predators do not ordinarily attack next to tree stumps. Even indoors, I think I spot them
humans, but many have 47 learned, by experience that out of the corner of my eye. Basically, I spend 50 every
a fish wounded by a diver’s spear is easy prey. free moment in search of them.

47 50

A) NO CHANGE A) NO CHANGE
B) learned B) appropriate vacation time slots
C) learned; C) every moment of my leisure time
D) learned, and D) whatever time I can set aside

CONTINUE
58 Conventional Expressions

In fact, the name yo-yo is a Tagalog word that In today’s world, Will Rogers would 53 of probably
translates 51 as “come back.” become a talk show host.

51 53

A) NO CHANGE A) NO CHANGE
B) by B) have probably became
C) with C) have probably become
D) from D) of probably became

Ernest C. Withers has been recording history with his In 1637 Descartes correctly 54 hypothesized that
camera for more than sixty years. Withers started taking rainbows occur when suspended water droplets refract
pictures 52 in his youth. light, separating it into an arc-shaped spectrum of colors.

52 54

A) NO CHANGE A) NO CHANGE
B) with B) hypothesized, that
C) of C) hypothesized; that
D) at D) hypothesized that,

Service opportunities range 55 between home repair


and meal preparation to ride sharing and reading aloud.

55

A) NO CHANGE
B) among
C) from
D) across
7 Sentence Boundaries

Attending school six days a week, 1 taking as many as Such is the ceremonial power we give to spoken text that
nine courses during a term. we just don’t think of each other as properly married

1 or 3 buried; until the right words from a book are


read.
A) NO CHANGE
B) as many as nine courses may be taken 3
C) they take as many as nine courses
A) NO CHANGE
D) nine courses are taken by as many as possible
B) buried until
C) buried. Until when
D) buried if
While clouds mist up from the sea, 2 sunlight
shimmering in the distance on the snowy slopes of
Mt. Fuji.

A) NO CHANGE
B) shimmering sunlight
C) sunlight shimmers
D) and sunlight shimmers

59
60 Sentence Boundaries

Although Walter Diemer, now a retired senior vice Her 7 voice, which by being one of a Latina living
president of the Fleer Corporation, never patented his outside the mainstream, found a large and attentive
invention, 4 while he had hoped to keep his recipe a audience in 1984 with the publication of her first short
secret from other gum manufacturers. story collection, The House on Mango Street.

4 7

A) NO CHANGE A) NO CHANGE
B) and B) voice—that of a Latina living outside the
C) so mainstream—
D) DELETE the underlined portion. C) voice, being one of a Latina living outside the
mainstream, it
D) voice—in which it was a Latina living outside the
mainstream—
A finger jabbed my arm as I waited for my order at a taco
stand in Santa Fe. “You don’t want to order it hot,” a man
I had never met insisted, 5 he flags down the stand Cisneros uses her influence as a successful writer to help

owner. other Latino writers get their works published. But 8


having made the argument that, in order for large
5
numbers of young Latinos to achieve literary success, the
A) NO CHANGE
educational system itself must change.
B) flagging
C) flags 8
D) he is flagging
A) NO CHANGE
B) she argues that,
C) arguing that,
Even the parts of a chili 6 vary the seeds and veins of a D) she argues that, when
pepper are hotter than the flesh.

A) NO CHANGE
B) vary;
C) vary,
D) vary but

CONTINUE
Sentence Boundaries 61

My package was then passed to the next person in line, Her interest in this theme grew directly from her
who, with equal 9 care, he sealed its edges with small once difficult relationship with her 12 mother and
circles of melted red wax, which quickly hardened in Tan describes her teen-aged self as “every mother’s
place. nightmare.”

9 12

A) NO CHANGE A) NO CHANGE
B) care to seal B) mother; Tan
C) care, sealing C) mother, and she also
D) care, sealed D) mother, Tan

According to Wilma P. Mankiller, 10 whose job was You sink into 13 despair, the house you thought you
like running a large company. owned has turned on its master.

10 13

A) NO CHANGE A) NO CHANGE
B) her job was B) despair. The
C) who’s job was C) despair the
D) the job being D) despair and the

Now 11 having lived in central Michigan where winter Take a long look at your shiny new bathroom, 14 settle
days are still considerably shorter than summer days, but into an easy chair, and enjoy a well-deserved rest.
the rate of change is much slower and the difference is 14
less extreme.
A) NO CHANGE
11 B) settling into an easy chair and enjoying
C) you settle into an easy chair and enjoy
A) NO CHANGE
D) settling into an easy chair, you enjoy
B) while I live
C) living
D) I live

CONTINUE
62 Sentence Boundaries

N.C. 15 Wyeth, gifted artist and illustrator, was in the Having recently moved to Florida, 17 and finding
early twentieth century. myself living in a subtropical environment for the first

15 time.

A) NO CHANGE 17
B) Wyeth was a gifted artist and illustrator A) NO CHANGE
C) Wyeth was a gifted artist, and illustrator B) having found
D) Wyeth, a gifted artist and illustrator, was C) I find
D) finding

16 Leon was also fascinated by faraway places. She


covered her bedroom walls with postcards of famous One writer who did publish back then, John 18 Okada.
sites. He would have been forgotten were it not for a group

16 of young Asian American writers who rediscovered


his novel, No-No Boy, and republished it for a new
Which choice most effectively combines the two
sentences? generation.
A) Leon was also fascinated by faraway places: and
she covered her bedroom walls with postcards of 18
famous sites.
A) NO CHANGE
B) Leon was also fascinated by faraway places, so
her bedroom walls, she covered with postcards of B) Okada; he
famous sites. C) Okada,
C) Leon was also fascinated by faraway places and D) Okada, who
covered her bedroom walls with postcards of
famous sites.
D) Leon was also fascinated by faraway places;
covering her bedroom walls with postcards of All along, she was telling stories—everything from social
famous sites.
satire to her retelling of “Cinderella” as Cindy 19 Ellie,
a poor African American girl whose rags are transformed
into magnificent African-style gowns.

19

A) NO CHANGE
B) Ellie. A
C) Ellie; a
D) Ellie, she was a

CONTINUE
Sentence Boundaries 63

Smith realized the extent of her gift when her friend Alex If you ask my mother what she remembers most vividly
Haley, who had gathered essential material for writing about my birth, she probably would tell you about the
his best-selling novel Roots from a griot in Gambia, gift my father brought me while 23 I was still in the
began to refer to her as “my American 20 griot,” this hospital. He brought a soccer ball.
was a revelation to Smith. 23
20 Which choice most effectively combines the two
sentences at the underlined portion?
A) NO CHANGE
A) I was still in the hospital, he brought a soccer ball.
B) griot” that
B) he brought a soccer ball in the hospital.
C) griot.” This
C) I, still in the hospital, brought a soccer ball.
D) griot,”
D) I was still in the hospital: a soccer ball.

The average alligator male is thirteen feet long and


My dad insists that 24 it was the endless hours he spent
weighs 600 21 pounds, females are slightly smaller.
kicking the ball to me when I was five or six helped me to
21 develop my soccer skills.

A) NO CHANGE 24
B) pounds females being
C) pounds; females are A) NO CHANGE
D) pounds; females being B) because of
C) being
D) DELETE the underlined portion.

Less than 9 percent of all hatchlings survive their first


two and a half 22 years predators find them easy
These cues may operate independently, or several may
quarry.
act 25 together. So that they signal language choice.
22
25
A) NO CHANGE
A) NO CHANGE
B) years. Predators
B) together. In order to signal
C) years, predators
C) together; signaling
D) year’s predators
D) together, signaling

CONTINUE
64 Sentence Boundaries

I returned home determined to learn to play the These bidders added $170,000 to the Sanitary
harmonica well. 26 I worked to find the right mouth Commission’s 29 fund. Within twelve months,
holes. I tried to bend the right notes. Gridley’s sack of flour had earned $275,000.

26 29

Which choice most effectively combines the two A) NO CHANGE


sentences? B) fund. Likewise, within
A) I worked to find the right mouth holes and bend C) fund, because within
the right notes.
D) fund, while within
B) I worked to find the right mouth holes and trying
to bend the right notes.
C) I worked, finding the right mouth holes to thus
bend the right notes.
Recently, Maya saw what seemed to her the perfect
D) I worked to find the right mouth holes; and tried to
bend the right notes. building for rent. The neighborhood was friendly, with
lots of foot 30 traffic. She envisioned their restaurant
fitting in nicely here.
27 To produce enough taxol to provide for the needs of
30
all patients, chemists estimated that they would need the
A) NO CHANGE
bark from about 40,000 Pacific yews each year.
B) traffic, that she
27 C) traffic, she
D) traffic she
A) NO CHANGE
B) Enough taxol production to provide
C) Producing enough taxol in providing
D) They wanted to produce enough taxol to provide Soon McLean stepped onstage with his 31 sax yet I
wasn’t prepared for the impact.

31
28 While voters elected the candidate that Dr. Herrick
A) NO CHANGE
supported.
B) sax.
28 C) sax that
D) sax, which
A) NO CHANGE
B) Since voters
C) When voters
D) Voters

CONTINUE
Sentence Boundaries 65

As he typed, the machine melted chunks of metal and Houdini, who devoted considerable effort to exposing
then formed the molten mass into lines of metal 32 hoaxes involving 35 spiritualism and was skeptical
type, bright and burning hot, these lines of type— about the existence of supernatural beings.
headlines, sentences, phrases—slid out of the machine. 35
32 A) NO CHANGE
A) NO CHANGE B) spiritualism, being
B) type, then bright C) spiritualism, was
C) type, they were bright D) spiritualism and
D) type. Bright

Amalia Hernandez bowed gracefully before the 36


Finally, I made a deal with myself. Each day I would try crowd, many viewers stood and applauded, showing
to find time for a half-hour 33 walk. Fitting it into my their appreciation for the performance they had just
schedule whenever I could. witnessed.

33 36

A) NO CHANGE A) NO CHANGE
B) walk. Which fit B) crowd many
C) walk; with fitting C) crowd. Many
D) walk, fitting D) crowd. While many

I watched him gently nudge his heels into Morgan’s 34 For example, at a 37 depth of sixty feet, scientists
side, which she quickly responded and trotted down the starting from the surface could only work one hour per
path. day.

34 37

A) NO CHANGE A) NO CHANGE
B) side, which it B) depth, of sixty feet: scientists
C) side. She C) depth of sixty feet; scientists
D) side, where she D) depth of sixty feet. Scientists

CONTINUE
66 Sentence Boundaries

I have 38 as many happy memories of my childhood in In California, poppies spread in dazzling carpets
Vietnam. of orange. In Virginia, pink cosmos accent the 41

38 landscape, occasionally, these scenic expanses come


about with no help from humans.
A) NO CHANGE
B) just as 41
C) that
A) NO CHANGE
D) DELETE the underlined portion.
B) landscape occasionally
C) landscape occasionally,
D) landscape. Occasionally,
Benjamin’s grandmother taught him to 39 read, he
attended a one-room Quaker school when the farm work
slowed down during the winter. Frank Lloyd Wright, one of the twentieth century’s most

39 creative and innovative 42 architects basing most of his


designs on two theories.
A) NO CHANGE
B) read, and he 42
C) read,
A) NO CHANGE
D) read;
B) architects. Based
C) architects, basing
D) architects, based
Having enrolled in a photography class for my first
semester, I had offered to preserve some of our
family’s 40 memories before the film deteriorated or Fragile, she had written next to my address and along
got lost. each of the 43 sides, I wasn’t sure, for a moment,

40 whether she meant to warn the postal workers or me.

A) NO CHANGE 43
B) memories. Before the film was A) NO CHANGE
C) memories; before the film had B) sides
D) memories before: the film C) sides as
D) sides.

CONTINUE
Sentence Boundaries 67

Ocloo knew that her countrywomen would need to have Every spring, there’s a contest where I live in northern
a steady source of 44 income. In order to gain access to Minnesota to see who can find the most 47 morels, this
education and health care. year, I’m going to enter.

44 47

A) NO CHANGE A) NO CHANGE
B) income in B) morels this
C) income; in C) morels. This
D) income: in D) morels, because this

One of the welding certificates on view was donated by The orange and black monarch 48 butterfly, which is
Bethena 45 Moore, operating a torch that could heat the most easily recognized and striking butterfly species
metal to 250°F, she helped make Kaiser one of the most in North America.
productive shipyards of the wartime era. 48
45 A) NO CHANGE
A) NO CHANGE B) butterfly
B) Moore operating C) butterfly that
C) Moore, who operated D) butterfly,
D) Moore. Operating

In November 2002, 49 a crew of physicians, sailors, and


46 Thoughts of San Francisco without its Victorian expedition experts, Cox headed for Neko Harbor on the
row houses or the New England coast without its Antarctic Peninsula.
lighthouses. 49
46 A) NO CHANGE
A) NO CHANGE B) a crew made up of
B) When thinking C) with a crew of
C) Think D) DELETE the underlined portion.
D) Thinking

CONTINUE
68 Sentence Boundaries

Using his hometown as his 50 base and documenting As the game unfolds, she jumps to her 53 feet, waving
the key people and events of the Memphis music the remote control and yelling, “That was clipping!” or
scene, baseball’s Negro Leagues, and the Civil Rights “Touchdown!”
Movement. 53
50 A) NO CHANGE
A) NO CHANGE B) feet! Waving
B) base. Withers documented C) feet waving,
C) base, Withers documented D) feet; waving
D) base, documenting

By donating their time and talents to others through


He was a 51 humorist who cloaked his insights in the nonprofit Time Dollar Exchange 54 programs are
homely garb of folk wit. contributing to their communities and fostering

51 interaction.

A) NO CHANGE 54
B) humorist A) NO CHANGE
C) humorist; B) programs now
D) humorist, C) programs, participants
D) programs, and

On Sundays, she settles on the sofa for the pregame 52


show. She doesn’t sit still or stay quiet for long. A program in Brooklyn, New York, that focuses on

52 services for senior citizens is one of the longest-


running 55 projects since 1987, more than 5,000
Which choice most effectively combines the two
sentences at the underlined portion? participants have tallied up 115,000 Time Dollars.
A) show she
55
B) show, but she
C) show, A) NO CHANGE
D) show; however, B) projects. Since
C) projects and since
D) projects;
8 Subordination
and Coordination

When Mom arrived, the guests had already been ferried In their effort to dwell on only the upbeat aspects of
out to a leaky, flat-bottomed 1 boat made festive with history, the people peddling in nostalgia are distorting
flowers and pink balloons. the past, 3 and remembering it.

1 3

A) NO CHANGE A) NO CHANGE
B) boat for it has been made B) nor
C) boat, which it was made C) as their
D) boat, making it D) not

Mom appears to be blessing the bride, 2 that a lightly Japanese students observe a rigorous annual schedule
harried expression is disclosed by the wind. that begins in the second week of 4 April and

2 extending through the following March.

A) NO CHANGE 4
B) which a lightly harried expression A) NO CHANGE
C) a slightly harried expression of whom B) April, which extends the school year
D) whose slightly harried expression C) April, their school year extends
D) April and extends

69
70 Subordination and Coordination

Taeko Komiyama has one of the most common family The dome should go. 8 While it’s true that the dome
names in Japan, 5 so in her case, it is especially fitting. blocks out rain, it also blocks out the sun and sky.

5 8

A) NO CHANGE A) NO CHANGE
B) but B) Since it’s
C) it’s C) For it’s
D) DELETE the underlined portion. D) It’s

6 While most vegetables taste similar from one My friend, a young man who had lived in Bombay his
species of the plant to another, the chili pepper, backbone entire life except for a few years spent studying in the
of the incendiary cuisine of the Southwest, varies United States, 9 if he were to walk with me to the post
immensely. office.

6 9

A) NO CHANGE A) NO CHANGE
B) Once B) walked
C) Certainly C) if he would walk
D) Because D) is walking

Why gather around a carpet, 7 in which we can do During that time, 10 as I talked to the workers—with
just as well at home, to enjoy our hot dogs, peanuts, and my friend’s help—in Hindi.
sodas? 10
7 A) NO CHANGE
A) NO CHANGE B) when
B) in being C) that
C) which D) DELETE the underlined portion.
D) DELETE the underlined portion.

CONTINUE
Subordination and Coordination 71

She sought start-up funding for the Cherokee 11 One writer who did publish back then, John Okada,
Gardens being a commercial gardening operation. would have been forgotten were it not for a group of

11 young Asian American writers who rediscovered his


novel, No-No Boy, 14 and republished it for a new
A) NO CHANGE
B) Gardens that was generation.
C) Gardens, it was 14
D) Gardens,
A) NO CHANGE
B) who republished it
C) having republished it
Unless we’re willing to deputize every sales clerk in every D) and it was republished
music 12 store, or music ratings won’t be enforced.

12
This 15 group, known as the Navajo code talkers, took
A) NO CHANGE
B) store, part in every assault the U.S. Marines conducted in the
C) store so Pacific from 1942 to 1945, transmitting information
D) store, thus on tactics, troop movements, orders, and other vital
communications over telephones and radios.

15
As I pushed the lawnmower back and forth, I composed
a couplet: “The graceful grass beneath my feet /Is crushed A) NO CHANGE
B) group which was
and trimmed in one big sweep.” Chuckling under my
C) group was
breath at my little poem, 13 the lawnmower picked up D) group
speed and soon finished mowing the lawn.

13

A) NO CHANGE
B) I picked up speed
C) speed started to pick up
D) speed at last was picking up

CONTINUE
72 Subordination and Coordination

The Navajo code remained classified after the war 16 Flaps in their nose, throat, and ears close to keep water
that was later used, along with codes made from other out, and transparent membranes protect their eyes, 19
American Indian languages, in the Korean Conflict and when it permitted them to see or hunt underwater as
the Vietnam War. well as on the surface.

16 19

A) NO CHANGE A) NO CHANGE
B) and which B) permitting
C) and C) and it permits
D) DELETE the underlined portion. D) permit

17 Since there’s not much chance that a seven-year- Flaps in their nose, throat, and ears close to keep water
old just learning the game can hit a pitched baseball, the out, 20 so that transparent membranes protect their
umpire puts the ball on top of a stationary tee, a piece of eyes, permitting them to see or hunt underwater as well
flexible tubing adjusted to each batter’s height. as on the surface.

17 20

A) NO CHANGE A) NO CHANGE
B) While B) in which
C) Although C) and
D) Unless D) since

The raptor now has a rebuilt, functional 18 feather. The way Renee cornrows my hair is soothing, like a
Eventually, it will be shed, allowing a new, complete scalp massage. 21 While the loose ends of each row
feather to grow in its place. are brought together into one final plait, which is tucked

18 under at the back of my head.

Which choice most effectively combines the two 21


sentences at the underlined portion?
A) NO CHANGE
A) feather, eventually it
B) After the
B) feather that, eventually,
C) When the
C) feather, which eventually it
D) The
D) feather; eventually

CONTINUE
Subordination and Coordination 73

Each synthetic braid is tied to my natural hair with a The vegalah I bought is a bright wire model with sturdy
tight double knot, which Renee guarantees 22 it will black wheels. However, new carts are like new 25
not unravel. sneakers: they lack the character that comes from use.

22 25

A) NO CHANGE A) NO CHANGE
B) that it B) sneakers, they
C) they C) sneakers, but
D) DELETE the underlined portion. D) sneakers

They kept the cart by the front door, ready for when they As a child, Coretta Scott loved to sing and play the piano.
needed to haul laundry down the long basement corridor Born in 1927 in Marion, Alabama, the daughter of a
to the washing machines 23 even to carry flour, successful farmer, 26 she became a top student in grade
chocolate, and walnuts to my grandmother’s Popham school and often led class songs.
Avenue apartment, where my sister preferred to bake. 26
23 A) NO CHANGE
A) NO CHANGE B) who became
B) or C) and
C) if D) DELETE the underlined portion.
D) DELETE the underlined portion.

Then, at age fifteen, I decided to enroll in a blues


And then there are the copies of my own vegalah that I harmonica class. On the first night, the instructor, 27
see coming and 24 going, pulling by a number of area whom had performed in local rock bands for years,
residents, who may, like me, wish their carts had more guided the class through a tortured version of “Mary Had
individuality. a Little Lamb.”

24 27

A) NO CHANGE A) NO CHANGE
B) going are pulled B) whom once
C) going, they are pulled C) who had
D) going, pulled D) who did

CONTINUE
74 Subordination and Coordination

28 To the surprise of many, it was a college student, The Harlem Renaissance author Zora Neale Hurston is
Maya Lin, who won the competition to design the best known for her novels, especially Their Eyes Were
Vietnam Veterans Memorial. Watching God, 30 however, she was also a devoted

28 chronicler of African American folklore.

A) NO CHANGE 30
B) She really surprised many, A) NO CHANGE
C) It was a surprise, B) even so,
D) It came as a surprise to many, C) despite
D) DELETE the underlined portion.

29 Some sculptors suggest the magnitude of an event


by presenting larger-than-life figures. Lin, on the other 31 While she was studying anthropology at Barnard
hand, relies on more understated methods to suggest College in New York that Hurston decided to reconnect
matters of great significance. with the customs, songs, stories, and games she had

29 learned as a child.

Which choice most effectively combines the two 31


sentences?
A) NO CHANGE
A) While some sculptors suggest the magnitude of
an event by presenting larger-than-life figures, Lin B) During
relies on more understated methods to suggest C) It was while
matters of great significance. D) While
B) Some sculptors suggest the magnitude of an event
by presenting larger-than-life figures; likewise, Lin
relies on more understated methods to suggest
matters of great significance.
In the first century A.D., the Greek biographer
C) Though some sculptors suggest the magnitude of
an event by presenting larger-than-life figures, she Plutarch 32 warned of sleeping in the shade of a yew
does not.
was a deadly practice.
D) Because some sculptors suggest an event’s
magnitude with larger-than-life figures, Lin relies 32
on understated methods to suggest matters of great
significance. A) NO CHANGE
B) warned people about
C) having warned that
D) warned that

CONTINUE
Subordination and Coordination 75

33 This process is more costly than harvesting Occasionally McLean would stroll offstage. Then one of
the bark, growing the foliage in a nursery provides a the other musicians 35 taking the lead, pulling strange
renewable resource. patterns from his instrument, the equivalent in sound of

33 someone dancing under a blinking strobe light.

A) NO CHANGE 35
B) This A) NO CHANGE
C) As this process B) would take the lead, pulling
D) Although this process C) took the lead, he pulled
D) taking the lead by pulling

34 Bidding became brisk. Townspeople felt


compassion for the soldiers fighting in the eastern United 36 Suddenly the show was over. The musicians walked
States. offstage.

34 36

Which choice most effectively combines the underlined Which choice most effectively combines the two
sentences? sentences?
A) Bidding became brisk even though townspeople A) Suddenly the show was over; walking offstage went
felt compassion for the soldiers fighting in the the musicians.
eastern United States. B) Suddenly the show was over and the musicians
B) Townspeople, because they were bidding briskly, walked offstage.
felt compassion for the soldiers fighting in the C) Suddenly, the musicians walked offstage, the show
eastern United States. was over.
C) Bidding became brisk because townspeople felt D) Suddenly the show was over, the musicians walked
compassion for the soldiers fighting in the eastern offstage.
United States.
D) For the soldiers fighting in the eastern United
States, bidding became brisk by the townspeople.
Over the years, I have gained a reputation 37 that
among family and friends as a honeybee enthusiast.

37

A) NO CHANGE
B) which is
C) that is
D) DELETE the underlined portion.

CONTINUE
76 Subordination and Coordination

38 Although antique photographs of African I had no experience with 41 horses when I had always
American life in major cities like Chicago and New York been too scared to ride them.
are not uncommon, few records of life in towns like 41
Lincoln have survived.
A) NO CHANGE
38 B) horses, which
C) horses;
A) NO CHANGE
D) horses, however,
B) Whether
C) Unless
D) Besides
These steps suggest that 42 except the debate about
noise controls is loud, we may eventually end up with a
Early in her career, Dr. Cheng became intrigued by quieter environment.
the fact that 39 because insects are the earth’s most 42
abundant and diverse organisms, they are virtually
A) NO CHANGE
absent from the world’s largest habitat, the ocean. B) even if
39 C) besides
D) despite
A) NO CHANGE
B) though
C) since
D) DELETE the underlined portion. Waila is a social 43 music that is performed at
weddings, birthday parties, and feasts.

43
When I moved from New York City to rural Nebraska, I
A) NO CHANGE
became friends with a 40 rancher, whose Amil. B) music in which it is performed
40 C) music, performing
D) music; performances
A) NO CHANGE
B) rancher; named
C) rancher named
D) rancher named,

CONTINUE
Subordination and Coordination 77

By contrast, American society has often been described 46 Conan Doyle sent a copy of one of the photographs
as one 44 that values the vibrant energy of youth over to his friend Harry Houdini. Harry Houdini was a
the wisdom and experience gained with age. famous magician and escape artist.

44 46

A) NO CHANGE Which choice most effectively combines the two


B) whose sentences?
C) this A) Conan Doyle sent a copy of one of the photographs
to his friend Harry Houdini, who was a famous
D) whom magician and escape artist.
B) Conan Doyle sent a copy to his friend Harry
Houdini, a famous magician and escape artist, of
one of the photographs.
Doyle then wrote an enthusiastic article for Strand C) Conan Doyle’s friend Harry Houdini, a famous
magazine, 45 being the place in which most of his magician and escape artist, sent a copy of one of
the photographs to him.
Sherlock Holmes stories had first appeared, and later D) Conan Doyle sent Harry Houdini a copy of one of
wrote a book on the subject titled The Coming of the the photographs, who was a famous magician and
escape artist.
Fairies.

45

A) NO CHANGE When Houdini remained unconvinced by the


B) in which the magazine where evidence, Conan Doyle became angry. Though the two
C) in which
remained 47 cordial, but their friendship was damaged
D) being where
by the disagreement.

47

A) NO CHANGE
B) cordial and
C) cordial that
D) cordial,

CONTINUE
78 Subordination and Coordination

48 In 1917, our house was built. In 1917, city planners They replaced the hollowed-out logs used to transport
developed an orderly scheme for the collection of trash. water with 50 metal pipes, in those allowed for the

48 introduction of indoor plumbing.

Which choice most effectively combines the underlined 50


sentences?
A) NO CHANGE
A) In 1917 when our house was built, city planners
developed an orderly scheme for the collection of B) pipes, which
trash. C) pipes in which
B) In 1917, our house was built, and in the same year, D) pipes those
1917, city planners developed an orderly scheme
for the collection of trash.
C) Our house was built in 1917 as part of a city
planner’s orderly scheme for the collection of trash. Because vegetable seeds must be planted every three or
D) In 1917, our house was built, so city planners
developed an orderly scheme for the collection of four years to maintain their viability, these rare varieties
trash. will be lost forever 51 unless they are not kept alive by a
network of dedicated gardeners.

51
49 The Bickfords were an enterprising couple.
They purchased two-thirds of the Virginia City Water A) NO CHANGE
B) that
Company in 1888.
C) while
49 D) if

Which choice most effectively combines the underlined


sentences?
A) The Bickfords were an enterprising couple in that Indeed, she identifies strongly with the griots of West
they would purchase two-thirds of the Virginia
City Water Company in 1888. Africa—those village storytellers 52 where they use
B) The Bickfords, an enterprising couple, they songs, poems, and narration to help preserve and
purchased two-thirds of the Virginia City Water
Company in 1888. transmit culture and history.
C) The Bickfords purchased two-thirds of the Virginia
52
City Water Company in 1888, an enterprising
couple.
A) NO CHANGE
D) The Bickfords were an enterprising couple and
B) who
purchased two-thirds of the Virginia City Water
Company in 1888. C) whom
D) that they

CONTINUE
Subordination and Coordination 79

My dad is a serious soccer player 53 who’s early Kayakers use a short, double-bladed 56 paddle, an
ambition had been to play professionally in his native elasticized sprayskirt fits snugly around the waist of the
country, Mexico. seated paddler to keep water out of the boat.

53 56

A) NO CHANGE A) NO CHANGE
B) that’s B) paddle, and
C) whose C) paddle, so
D) which D) paddle

For example, in 1988, Lin designed a memorial to honor In one shot, my young grandparents toted crates filled
individuals killed in the Civil Rights Movement. Located with ripe fruit, and my mother sat on a 57 branch,
in Montgomery, Alabama, 54 that work consist of peering down at the camera.
a low granite disk on which the names of the dead 57
appear along with references to landmark events of the
A) NO CHANGE
movement. B) branch, which peered
54 C) branch that peered
D) branch peering
A) NO CHANGE
B) this work consists
C) she consists
D) it consisted After much too long a flight, we landed. 58 Being that I
fully stretched my legs for the first time in twelve hours,
I poked my head out the bulkhead door and exclaimed,
Garbage cans would be picked up from the narrow “This place is hot.”
alleys 55 that ran behind the rows of houses. 58
55 A) NO CHANGE
A) NO CHANGE B) Because I fully stretched
B) being that they C) Although fully stretching
C) so that they D) Fully stretching
D) since they

CONTINUE
80 Subordination and Coordination

59 Texas was one of the early leaders in this The students learn about the art and science of
movement, established a wildflower program in the architecture, and they perform the school’s daily
1930s, thanks to the initiative of Gibb Gilchrist, the maintenance tasks such as cooking, serving, and cleaning
stated highway engineer at the time. up in the school’s kitchen. Those tasks are part of the

59 school’s curriculum, 61 although Wright believed that


only a person familiar with every room’s function could
A) NO CHANGE
B) Texas, where design a well-formed house.
C) In Texas, 61
D) Texas,
A) NO CHANGE
B) since
C) while
60 Students being taught these theories and their D) furthermore
practice, Wright founded the Frank Lloyd Wright School
of Architecture in 1932.

60 The contents of this package were indeed 62 fragile. I


thought that the reason it was fragile was not in any of
A) NO CHANGE
B) To teach students the expected ways.
C) Students learning 62
D) As students learn
Which choice most effectively combines the two
sentences at the underlined portion?
A) fragile: not
B) fragile; but not
C) fragile, I thought, but not
D) fragile. I did not think

CONTINUE
Subordination and Coordination 81

However, it was when I began pronouncing the words But many people felt that jazz was stagnating. Rock ’n’
to 63 myself; that I felt the true weight of the book. roll had become the most popular music 66 of the era.

63 No major wave of inspiration had swept through jazz


since the bebop revolution of the early forties.
A) NO CHANGE
B) myself 66
C) myself−
Which choice best combines the two sentences at the
D) myself. underlined portion?
A) of the era, similarly, no major
B) of the era no major
C) of the era; thus no major
After he died, it was a language I never heard again 64
D) of the era, and no major
nonetheless, until I heard myself struggling with the
words he had used so easily all those years ago, his voice
coming back as vividly as if he were next to me once
The labor force that shrank as men joined the military
again, pointing to the page, guiding me.
forces 67 but grew as women flooded into jobs newly
64 open to them.

A) NO CHANGE 67
B) furthermore,
C) because, A) NO CHANGE
D) that is, B) though it
C) and yet it
D) DELETE the underlined portion.

Often, when I feel a need for tranquility, I try to summon


images of the tall grasses and furry cattails and the
An existing barn can become part of a modern farming
mallards, herons, and geese 65 whom are found in the
operation, 68 and it is converted for use as a store or
32,000-acre marsh.
residence.
65
68
A) NO CHANGE
A) NO CHANGE
B) whom can be found
B) or it can be
C) that can be found
C) and that is why it can be
D) that are finding
D) because they are

CONTINUE
82 Subordination and Coordination

Many of the visitors try scuba 69 diving. Scuba diving These discoveries provided strong evidence that a large
is one of the country’s fastest-growing sports. asteroid crashed into Earth about 65 million years 71

69 ago. The crash created what is known as the Chicxulub


crater.
Which choice most effectively combines the two
sentences at the underlined portion? 71
A) diving that is one
B) diving, one Which choice most effectively combines the two
sentences at the underlined portion?
C) diving; which is one
A) ago, to create
D) diving and one
B) ago, in order to create
C) ago, creating
D) ago; creating
A barracuda’s elongated head, with its sharp teeth and
intense stare, might intimidate you, but if you turn and
point a finger at the fish, you will usually scare it away. Beverly Harvard—the chief of police in Atlanta and the
It holds the world’s record for the underwater 70 dash, first African American woman to lead a major city’s
it’s probably as wary of your unfamiliar size, shape, and police department—began her career in law enforcement
smell as you are of its speed. in an unusual 72 way: she answered a challenge made

70 by her husband.

A) NO CHANGE 72
B) dash A) NO CHANGE
C) dash that B) way; to answer
D) dash, but C) way to answer
D) way of winning

CONTINUE
Subordination and Coordination 83

Her dedication opened doors for her within the 73 Monarchs are particularly fascinating 76 because they
department. Each door led to a higher level of command. are one of the main migratory butterfly species in North

73 America.

Which choice most effectively combines the two 76


sentences at the underlined portion?
A) NO CHANGE
A) department, led
B) therefore
B) department, that led
C) despite
C) department, in which each led
D) DELETE the underlined portion.
D) department, and each led

Drawings of objects resembling yo-yos adorn ancient


Her efforts to deter crime have been praised by many
Greek vases and the walls of Egyptian temples, 77 if
community leaders, 74 admiring her calm style in
written mention of yo-yos goes back to the fifth century
dealing with problems.
B.C.
74
77
A) NO CHANGE
B) by admiring A) NO CHANGE
C) who admire B) and
D) being that they admire C) since
D) because

The birds roost on sandbars in the river during 75 the


In the 1920s Pedro Flores, a Filipino immigrant,
night. During the day they feast on the remains of last
introduced the toy in the United 78 States. He started a
year’s crops in the surrounding fields.
yo-yo manufacturing company in California.
75
78
Which choice most effectively combines the two
sentences at the underlined portion? Which choice most effectively combines the two
A) the night, and during the day they feast sentences at the underline portion?
B) the night; feasting during the day A) States, as Flores was starting
C) the night: daytimes they feast B) States; he started
D) the night: during the day they feast C) States; starting
D) States he started

CONTINUE
84 Subordination and Coordination

Immersed in the icy water off the Antarctic Peninsula, Late in the afternoon, a cluster of seaweed on the beach
Lynne Cox wasn’t sure if she could accomplish her caught my attention. 82 An examination of the tangled
goal to be the first person to swim a mile through the heap, I saw a lightning whelk.
glacier-strewn sea. At forty-five, she had been training 82
for two years for this event, 79 which she hoped her
A) NO CHANGE
preparations would pay off. B) Examining
79 C) A close look at
D) In order to examine
A) NO CHANGE
B) and
C) then
D) DELETE the underlined portion. Although falling rain separates moonlight into the same
spectrum of color present in a solar rainbow, a lunar
rainbow is likely to be perceived as a delicate white
People go to this narrow strip of land in the Gulf of 80 arc against the dark night sky, 83 and which poorly
Mexico, there they sunbathe, fish, swim, explore the illuminated objects appear colorless at night.
towns, or simply stroll along the more than eighty miles 83
of beautiful shoreline.
A) NO CHANGE
80 B) which
C) because
A) NO CHANGE
D) DELETE the underlined portion.
B) Mexico to
C) Mexico, they
D) Mexico, tourists
84 In fact, the human eye loses its sensitivity to color
at low levels of illumination, a camera with a very long
I also learned the names of shells that weren’t in exposure setting can capture a lunar rainbow’s colors.
my collection 81 for which they might be if I kept 84
returning to Padre Island.
A) NO CHANGE
81 B) Though the
C) The
A) NO CHANGE
D) Consider that the
B) but
C) they
D) but when they
9 Parallel Structure

1 Sincerely, gracious, and always in full command of The greatest emphasis, however, is on the basic skills of
her art, she seemed completely absorbed in every song writing, reading, and 2 mathematical abilities and
she sang. aptitudes.

1 2

A) NO CHANGE Which choice provides another example most like the


B) Sincere, gracious, ones already in the sentence?
C) Sincere graciously A) NO CHANGE
D) Sincerely, graciously B) the subject where math skills are practiced.
C) basic mathematical computations.
D) mathematics.

She led projects to build houses and 3 to bring


running water to isolated towns.

A) NO CHANGE
B) bring about
C) bringing
D) DELETE the underlined portion.

85
86 Parallel Structure

Depression often sets in and 4 survival seem doubtful Over an entire summer, I learned a lot about plumbing,
as people go to work and come home in complete electrical wiring, and 7 doing a carpenter’s work.
darkness. 7
4 Which choice provides a supporting example most like
the ones already in the sentence?
A) NO CHANGE
A) NO CHANGE
B) survival itself often seems
B) that of a carpenter.
C) survival, it often seems
C) to do carpentry.
D) survival seeming
D) carpentry.

For Tan, the reward of writing is not found in how many


Even parents who grew up listening to early rock tunes
books she sells, but 5 if she were to learn about herself
often find recent songs sexist, racist, or 8 containing
through writing.
violence.
5
8
A) NO CHANGE Which choice provides another example most like the
B) in what she learns ones already in the sentence?
C) if she would learn A) NO CHANGE
D) what is she to learn B) with themes of violence.
C) violent.
D) referring to violent themes.

Tan finally realized that her discontent stemmed not


from the writing itself, but 6 with the subject matter—
It’s impossible to predict which people will dig into their
she didn’t care about business. She decided what she
pockets or 9 if they were to open their purses, and I’ve
really wanted to do was write fiction.
stopped trying to guess.
6
9
A) NO CHANGE
B) for A) NO CHANGE
C) from B) would have opened
D) in C) open
D) might be opening

CONTINUE
Parallel Structure 87

Alligators have a remarkable metabolism that enables After two weeks’ effort, I would always set the harmonica
them to withstand cool weather and 10 allow them to aside and 13 had went back to the baseball field.
survive traumatic injuries such as losing a limb. 13
10 A) NO CHANGE
A) NO CHANGE B) would of gone
B) allowing them survival of C) go
C) allows them survival of D) would of went
D) allows them to survive

That smell told us that it was time to walk the vines


After college, she enrolled in Boston’s New England and 14 the best of the crop being selected for canning.
Conservatory of Music, where her studies, she hoped, 14
would lead to a degree in voice and a 11 celebrated
A) NO CHANGE
concert singer. B) we select the best of the crop for canning.
11 C) select the best of the crop for canning.
D) canning the best of the crop that we selected.
A) NO CHANGE
B) career as a
C) professional
D) DELETE the underlined portion. Their disappointment turned to 15 excited because
they realized that they could use this time to acquire the
equipment they would need.
Before you can entertain any second thoughts, the plane 15
has taxied and 12 taken off and is circling upward into
A) NO CHANGE
the huge cobalt sky. B) excitement
12 C) exciting
D) being excited
A) NO CHANGE
B) took
C) had taken
D) had took

CONTINUE
88 Parallel Structure

Afterward, the type was cleaned in various chemicals Max the dog would immediately run after the ball
and 16 were put back into the Linotype to be re-created and 19 the ball would be stopped with his nose.
as tomorrow’s headlines and stories. 19
16 A) NO CHANGE
A) NO CHANGE B) the ball would stop
B) put C) stopping it
C) they were put D) stop it
D) DELETE the underlined portion.

During the Moon Festival, I would show off the lanterns


The air smells of coffee, 17 not smelling of ink. my mother had made, 20 eat mooncake, and drink tea

17 with my grandparents.

A) NO CHANGE 20
B) ink it is not. A) NO CHANGE
C) not ink. B) eaten mooncake, and drank
D) but not like ink. C) eaten mooncake, and drunk
D) ate mooncake, and drank

Walking cleared my head and 18 brightens my spirits


in a way no other exercise routine ever had. It is sometimes as important to learn the politeness

18 conventions used by speakers of other languages and


cultures 21 as learning their vocabulary and grammar.
A) NO CHANGE
B) was brightening 21
C) might brighten
A) NO CHANGE
D) brightened
B) as having learned
C) when he is learning
D) as it is to learn

CONTINUE
Parallel Structure 89

My tastes are similarly diverse, whether in art, literature, In 1753, at the age of twenty-two, Banneker 25
music, movies, TV shows, or 22 long walks are enjoyed. constructed a clock out of hand-carved wooden parts,

22 displayed his mechanical skills, and displaying his


interest in learning.
A) NO CHANGE
B) long walks’ enjoyment. 25
C) long walks.
A) NO CHANGE
D) enjoying long walks.
B) displayed his skills when he constructed a clock
out of hand-carved wooden parts and displayed his
interest in mechanical skills.
C) displayed both his interest in learning and his
Unbricking a kiln after a firing is like 23 a person
mechanical skills when he constructed a clock out
uncovering buried treasure. of hand-carved wooden parts.
D) displayed his interest in mechanical skills by
23 constructing a clock out of hand-carved wooden
parts and his interest in learning.
A) NO CHANGE
B) someone
C) a potter
D) DELETE the underlined portion. Yet others, salt-encrusted, “sleep” in ancient caverns,
waking after centuries to feed and 26 to be bred.

26
Although high school is not compulsory, 24 attendees
A) NO CHANGE
have become virtually universal.
B) for breeding.
24 C) to breed.
D) breeding.
A) NO CHANGE
B) the number of attendants has
C) attendants have
D) attendance has

CONTINUE
90 Parallel Structure

Moua is proud of her accomplishments and 27 those of Crucial to the war effort, these women were
other Hmong writers in the United States. immortalized by a fictional poster character of the era:

27 Rosie the Riveter. Her strong arms, direct gaze, 30


wearing rugged work clothes convey the competence and
A) NO CHANGE
B) that of determination that so many women brought with them
C) about into their new roles.
D) with
30

A) NO CHANGE
B) in
We trained our binoculars on the white 28 herons
C) and
laughed at the frolicking mallards, and could not get over D) her
the huge flocks of geese.

28
A growing number of people are working to reverse that
A) NO CHANGE
B) heron’s trend, arguing that these grand old buildings are cultural
C) herons, artifacts just as worthy of preservation 31 as other
D) herons; historic structures.

31

Standing on the edge of the marsh at early evening, we A) NO CHANGE


B) for
watched a perfectly formed V of geese approach, fly
C) with
low overhead, and then continue to the other side of D) like
the marsh. I can hear the beating of their wings and 29
their mournful honking even now.

29

A) NO CHANGE
B) even now they make their mournful honking
sounds.
C) even now there is mournful honking.
D) they honk mournfully even now.

CONTINUE
Parallel Structure 91

Each barn saved also represents a link to an Monarchs have smaller bodies and 35 insufficiently
agricultural 32 heritage. The rescued barns can provide developed nervous systems than migratory birds.
important historical and cultural information. 35
32 A) NO CHANGE
Which choice most effectively combines the two B) less
sentences at the underlined portion? C) more insufficient
A) heritage; providing D) inadequate
B) heritage, and provided
C) heritage so rescued barns provide
D) heritage and can provide
The features of birds that help them accomplish their
long migrations are aerodynamic design, acute vision,
and 36 the ability to regulate their body temperature
So, Harvard, who held a master’s degree in urban
and maintain energy.
government and administration, bet her husband that
she could pass the 33 tests and then signed on as a 36

police recruit. A) NO CHANGE


B) regulating body temperature and maintaining
33
energy with their ability.
A) NO CHANGE C) with their body temperature regulation and energy
maintenance ability.
B) tests, where she then signed
D) the regulation of body temperature and their ability
C) tests, in that she then signed
to maintain energy.
D) tests and then signing

Smaller and 34 paler then the average pinecone, a


morel blends perfectly into its natural surroundings.

34

A) NO CHANGE
B) more pale then
C) paler than
D) more pale than

CONTINUE
92 Parallel Structure

Her initial exhaustion turned into exhilaration as she Also, because moonlight is less powerful 39 then
moved through water that was clearer and 37 blue as sunlight, lunar rainbows are faint and easily obscured by
any she’d swum in before. surrounding lights and air pollution.

37 39

A) NO CHANGE A) NO CHANGE
B) more blue then B) than sunlight,
C) bluer than C) as sunlight,
D) bluer then D) then sunlight is,

My grandmother can talk football as 38 fluently and


confidently as any sportscaster.

38

A) NO CHANGE
B) fluent and confidently
C) fluently and confident
D) fluent and confident
10 Modifier Placement

You watch as each turtle then slowly, laboriously, 1 The second thing Roosevelt did was to arrange for
buries the eggs by turning in circles, thus pushing sand Anderson to sing before the entire nation via radio from
back into the holes with surprisingly efficient flippers. the steps of the Lincoln Memorial. 2 Anderson wrote

1 that she was in her autobiography so nervous she barely


remembered that day.
A) NO CHANGE
B) until the eggs are buried with surprisingly efficient 2
flippers, pushing sand back into the holes, turning
in circles. A) NO CHANGE
C) turning in circles with surprisingly efficient flippers B) Anderson wrote in her autobiography that she was
until the eggs are buried, pushing sand back into so nervous she barely remembered that day.
the holes. C) Anderson wrote that she was so nervous in her
D) turns in circles, back into the holes, pushing sand autobiography she barely remembered that day.
until they are buried with surprisingly efficient D) Anderson wrote that she was so nervous she barely
flippers. remembered that day in her autobiography.

93
94 Modifier Placement

3 The rubber-boot wedding was one of the most 5 The Anaheim pepper often graces doorways tied
exciting and, despite the boots, until now, romantic into ropes called ristras.
ceremonies so far. It took place on a wide pond. 5
3 A) NO CHANGE
A) NO CHANGE B) Tied into ropes called ristras, doorways are often
graced with the Anaheim pepper.
B) The rubber-boot wedding until now, despite the
boots, was one of the most, so far, exciting and C) The Anaheim pepper tied into ropes, often graces
romantic ceremonies. doorways, called ristras.
C) The rubber-boot wedding until now was one of the D) Tied into ropes called ristras, the Anaheim pepper
most exciting, and romantic, so far, ceremonies often graces doorways.
despite the boots.
D) Until now, the rubber-boot wedding was one of
the most exciting and, despite the boots, romantic
ceremonies so far. 6 Domed and bowl-shaped, my sons sit in a park
where the players meander about on artificial turf.

6
As I forked up my burrito, tears of gratitude streamed
A) NO CHANGE
down my face. 4 That chili sauce was the hottest that
B) My sons sit, in a park, domed and bowl-shaped
had ever passed my lips mingled with tears of pain. C) My sons in a park sit domed and bowl-shaped
D) My sons sit in a domed, bowl-shaped park
4

A) NO CHANGE
B) Then tears of pain, the hottest that had ever passed
my lips, were with the hottest chili sauce mingled. Each role requires a certain physical type, 7 so, while
C) Mingled with those were tears of pain: that sauce auditioning, performers tend to possess the same general
was the hottest that had ever passed my lips.
physical characteristics.
D) That sauce was the hottest that had ever passed my
lips: mingled with those were tears of pain. 7

A) NO CHANGE
B) thus possessing the same physical characteristics,
the performers tend to audition.
C) so the performers who audition tend to possess the
same general physical characteristics.
D) so the same general physical characteristics tend
to be in the possession of the performers who
audition.

CONTINUE
Modifier Placement 95

8 In 1945, in Oklahoma, Mankiller, the name stems The children regularly engaged in activities 11
from a Cherokee military title, was born. stimulating their curiosity and creativity, which were

8 designed by Wyeth.

A) NO CHANGE 11
B) Being in Oklahoma in 1945, Mankiller, the name A) NO CHANGE
stems from a Cherokee military title, was born.
B) that would stimulate their curiosity as designed by
C) Mankiller, whose name stems from a Cherokee Wyeth.
military title, was born in 1945 in Oklahoma.
C) designed by Wyeth, they were to stimulate their
D) Mankiller’s name stems from a Cherokee military curiosity.
title in 1945 in Oklahoma, she was born.
D) that were designed by Wyeth to stimulate their
curiosity.

9 Her success with economic development led to her


election in 1983 as deputy chief. They were expected to keep 12 scrupulously written

9 journals containing sketches and notes.

A) NO CHANGE 12
B) Leading to her election in 1983 as deputy chief, her A) NO CHANGE
success was with economic development.
B) written journals scrupulously containing
C) Her election in 1983 as deputy chief was resulting
from her economic success. C) journals, which were scrupulously containing
written
D) In 1983, electing her as deputy chief, her success
with economic development was recognized. D) scrupulously written journals, which were
containing

He is celebrated 10 for his vivid and haunting


Noticing the girl’s fascination, 13 Tania’s grandmother
images by art historians, many of which continue to be
enrolled her in the Havana Music Conservatory, where
published in children’s books today.
she became the youngest student in the school.
10
13
A) NO CHANGE
A) NO CHANGE
B) by art historians for his vivid and haunting images,
B) she was enrolled by her grandmother
C) for his vivid and haunting images, by art historians,
C) Tania was enrolled
D) by vivid and haunting art historians who consider
his images, D) she enrolled her

CONTINUE
96 Modifier Placement

The first night in my new home, 14 an incredible racket 16 Just yesterday I sat across the aisle from a woman
suddenly awakened me emanating from somewhere who was composing music in pink-tinted glasses in a
inside the house. notebook.

14 16

A) NO CHANGE A) NO CHANGE
B) suddenly, an incredible racket awakened me B) Just yesterday I sat across the aisle from a woman
C) I was suddenly awakened by an incredible racket who was composing in pink-tinted glasses music in
a notebook.
D) suddenly, awakened by an incredible racket, I was
C) In pink-tinted glasses, just yesterday I sat across the
aisle from a woman who was composing music in a
notebook.
D) Just yesterday I sat across the aisle from a woman
After coming across a copy of Okada’s novel in 1970, 15
in pink-tinted glasses who was composing music in
writer Jeff Chan got excited about Okada, including his a notebook.

friends.

15
Some people say that a woman’s hair is her crowning
A) NO CHANGE
glory. 17 Well, my decade-old hairstyle had begun to
B) Okada made an impression on writer Jeff Chan and
his friends. look like a tarnished crown. I opted for something new—
C) he and his friends got excited about Okada’s work. braid extensions.
D) writer Jeff Chan and his friends got excited about
Okada. 17

Which choice most effectively combines the two


sentences?
A) Well, my decade-old hairstyle had begun to look
like a tarnished crown: I opted for something
new—braid extensions.
B) Well, my decade-old hairstyle had begun to look
like a tarnished crown, so I opted for something
new—braid extensions.
C) Well, my decade-old hairstyle, instead of opting
for something new, braid extensions, had begun to
look like a tarnished crown.
D) Well, my decade-old hairstyle had begun to look
like a tarnished crown thus braid extensions were
new.

CONTINUE
Modifier Placement 97

Shifting her attention slightly, 18 I am asked which In the mid-sixties, Coretta Scott King began to take
church I attend and invited to visit her congregation. on a more active role in the movement, and after her

18 husband’s assassination in 1968, she helped carry on the


cause of racial and economic justice. As founder of the
A) NO CHANGE
B) there’s a question about which church I attend, and Martin Luther King, Jr. Center for Nonviolent Social
I am invited Change, 20 the work continues to fulfill the goals of
C) Renee asks which church I attend and invites me
peace and justice.
D) I’m asked about the church I attend, inviting me
20

A) NO CHANGE
Now, walking along Montague Street in Brooklyn, I B) she continued to work
notice all sorts of vegalahs. 19 There’s the gleaming C) the work is continuing
D) the ongoing work
silver vegalah, its handle neatly repaired with duct tape.
That one belongs to the apartment manager, who lives
upstairs.
21 Iago is the villain of the tragedy, Othello. Iago strives
19 to undo Othello, a respected general.
Which choice most effectively combines the two
21
sentences?
A) There’s the gleaming silver vegalah with a handle Which choice most effectively combines the two
that belongs to the apartment manager, neatly sentences?
repaired with duct tape, who lives upstairs. A) Iago, the villain of the tragedy Othello, strives to
B) There’s the gleaming silver vegalah with a handle undo Othello, a respected general.
neatly repaired with duct tape; it belongs to the B) Iago, villain of the tragedy Othello, strives to undo a
apartment manager, who lives upstairs. respected general who has the same name.
C) There’s the gleaming silver vegalah who lives C) The villain of the tragedy Othello, Iago, respects the
upstairs with a handle neatly repaired with duct general Othello and strives to undo him.
tape that belongs to the apartment manager.
D) As the villain of the tragedy Othello, he, Iago,
D) There’s the one that belongs to the apartment strives to undo Othello, a respected general.
manager, a gleaming silver vegalah with a handle
neatly repaired with duct tape, who lives upstairs.

CONTINUE
98 Modifier Placement

22 Pleasantly heavy and warm from the sun, I can still When I process text at my computer, I remember my
imagine the feel of the fruit. father at his Linotype machine. 24 When comparing

22 his workplace to mine, it is quiet and clean.

A) NO CHANGE 24
B) I can still imagine the feel of the fruit, pleasantly A) NO CHANGE
heavy and warm from the sun.
B) Making a comparison between his workplace and
C) Pleasantly, the fruit is still warm and heavy where I mine, it is quiet and clean.
can feel it in my imagination from the sun.
C) Compared to his workplace, my office is quiet and
D) Pleasantly heavy, I can still imagine the feel of the clean.
warm fruit from the sun.
D) Quiet and clean compared to his workplace, which
wasn’t.

23 Recently, however, the discovery of taxol, a


substance found in the bark of the yew, has given the tree 25 Just last month, for instance, I received a call from
new status. In the 1960s, scientists discovered that taxol a neighbor. Her plea for my help resulted in a happy
was a promising anticancer drug. ending for everyone involved.

23 25

Which choice most effectively combines the underlined Which choice most effectively combines the two
sentences? sentences?
A) In the 1960s, scientists discovered that taxol was a A) Just last month, for instance, her plea for my help
promising anticancer drug; finding that substance from a neighbor’s call resulted in a happy ending
in the bark of the yew has recently given the tree for everyone involved.
new status. B) Just last month, for instance, I received a call from
B) Recently, however, the discovery of taxol in the a neighbor, whose plea for my help resulted in a
1960s, a substance found in the bark of the yew, has happy ending for everyone involved.
given the tree new status as a promising anticancer C) For instance, just last month I received a call from
drug. a neighbor of which the plea for help resulted, for
C) Recently, however, the discovery of taxol, a everyone involved, including my neighbor, in a
substance found in the bark of the yew, has given happy ending.
the tree new status and in the 1960s, scientists D) For instance, just last month I received a call from a
discovered that it was a promising anticancer drug. neighbor, resulting in a happy ending for everyone
D) Recently, however, scientists discovered that yew involved for my help.
has a bark that has given the tree new status since
the 1960s as a promising anticancer drug.

CONTINUE
Modifier Placement 99

One picture shows a little boy standing on a I didn’t even know how to get the horse to start moving.
platform. 26 On one side of him is a dog. The boy holds Amil had a good-natured smile on his face, but I think he
the dog’s collar in his hands. could tell how nervous I was. 28 Sitting at the controls

26 of a jumbo jet, I might just as well have been flying.

Which choice most effectively combines the underlined 28


sentences?
A) NO CHANGE
A) On one side of him is a dog of which the boy holds
the dog’s collar in his hands. B) I might just as well have been sitting at the controls
of a jumbo jet.
B) On one side of the boy is a dog; he holds the dog’s
collar in his hands. C) Amil could tell I was nervous as if I was flying,
sitting at the controls of a jumbo jet.
C) On one side of the boy is a dog which holds his
collar in his hands. D) I might, sitting at the controls of a jumbo jet, have
been just as nervous as if I was flying.
D) On one side of him is a dog that is holding its collar
in his hands.

For these reasons, there’s 29 increasing agreement


She suspects insects are less successful at sea because with the view that “good neighbors keep their noise to
they so readily adapted to land. 27 As partial evidence, themselves.”
Dr. Cheng cites the fact that Halobates spend their
29
lives walking on top of the water. Walking on the water
A) NO CHANGE
eliminates the need to compete with the crustaceans below.
B) agreement increasing with the view
27 C) agreement with the view increasing
D) agreement with the increasing view
Which choice most effectively combines the two sentences?
A) As partial evidence, Dr. Cheng, citing the fact that
Halobates spend their lives walking on top of the
water, eliminates the need to compete with the
crustaceans below.
B) As partial evidence, Dr. Cheng eliminates the need
to compete with the crustaceans below by citing the
fact that Halobates spend their lives walking on top
of the water.
C) Dr. Cheng, as partial evidence of the fact that
Halobates spend their lives walking on top of the
water, eliminates the need to compete with the
crustaceans below.
D) As partial evidence, Dr. Cheng cites the fact that
Halobates spend their lives walking on top of the
water, thus eliminating the need to compete with
the crustaceans below.

CONTINUE
100 Modifier Placement

30 Cheek to cheek, the dance is performed to the Bolted to a two-hundred-ton platform sixty-six feet
relaxed two-step tempo, and the bands often play long below sea level at Conch Reef, 32 scientists at this
past midnight. research outpost are offered a unique opportunity to

30 study the impact of pollution on coral reefs.

A) NO CHANGE 32
B) Couples dance cheek to cheek to the relaxed two- A) NO CHANGE
step tempo,
B) this research outpost offers scientists
C) A relaxed two-step tempo, the couples dance cheek
to cheek, C) scientists researching at this outpost are offered
D) Cheek to cheek, the two-step tempo relaxes D) research scientists at this outpost are offered
dancing couples,

Dr. Grant’s invention of the golf tee and his innovative


The practice of advancing a person’s age seems to me to dental bridge may seem to be an unlikely pairing. 33
reflect the value a society places on life experience and However, each accomplishment exemplifies a
longevity. 31 This idea was demonstrated often by my characteristic common to innovators. That characteristic
elderly relatives. My relatives took pride in reminding is the ability to identify and solve a problem.
younger folk of their “Korean age.”
33
31 Which choice most effectively combines the underlined
Which choice most effectively combines the underlined sentences?
sentences? A) However, each accomplishment exemplifies a
A) This idea, taking pride, was demonstrated often characteristic common to innovators and that
by my elderly relatives, reminding younger folk of characteristic is the ability to identify and solve a
their “Korean age.” problem.
B) This idea was taking pride in reminding younger B) However, each accomplishment exemplifies a
folk of their “Korean age” by my elderly relatives. characteristic: to identify and solve a problem
common to innovators.
C) This idea was demonstrated often by my elderly
relatives, who took pride in reminding younger C) However, each accomplishment exemplifies a
folk of their “Korean age.” characteristic common to innovators: the ability to
identify and solve a problem.
D) This idea was demonstrated often by my elderly
relatives that were taking pride in reminding D) However, the ability to identify and solve a
younger folk of their “Korean age.” problem is common to innovators with these
accomplishments.

CONTINUE
Modifier Placement 101

My mother grew quiet as I shared my hopes of going Periodically, Ellen looks through a porthole in the wall
away to college. 34 She looked at me sadly at the age of of the kiln to determine the fire’s intensity. At last, when
eighteen and said that I was abandoning our traditions the temperature soars well above a thousand degrees,
by leaving home. she knows the firing is nearing its end. 36 Having

34 died down, she bricks up the firebox as well, sealing the


remaining heat inside.
A) NO CHANGE
B) She looked at me sadly and said that I was 36
abandoning our traditions by leaving home at the
age of eighteen. A) NO CHANGE
C) At the age of eighteen, she looked at me sadly B) Finally it dies
and said that I was abandoning our traditions by C) With a blaze that dies
leaving home.
D) Once the blaze dies
D) She, at the age of eighteen, looked at me sadly
and said that I was abandoning our traditions by
leaving home.

37 In a few days the kiln has cooled. At that time, Ellen


opens the chamber.
One evening at a restaurant, 35 a dish of stuffed
37
mushrooms was ordered with his meal, assuming the
Which choice most effectively combines the underlined
dish would be served as an appetizer. sentences?
35 A) In a few days, when the kiln has cooled, Ellen
opens the chamber.
A) NO CHANGE B) The kiln, cooling in a few days, opens its chamber
B) a dish was ordered of stuffed mushrooms to Ellen.
C) he ordered a dish of stuffed mushrooms C) Ellen, cooling in a few days, opens the chamber of
the kiln.
D) ordering a dish of stuffed mushrooms
D) Cooling in a few days, Ellen opens the chamber of
the kiln.

CONTINUE
102 Modifier Placement

38 Rosie, it turns out, is a desert tortoise. My It was a handbook of the Miami language published
grandmother had started raising Rosie over twenty years by the Miami Nation of Oklahoma. 40 It seemed too
ago. small, too light, to contain the building blocks of an

38 entire language, one that had lain dormant for me for


over thirty years.
Which choice most effectively combines the underlined
sentences? 40
A) Rosie, my grandmother, it turns out had started
raising a desert tortoise over twenty years ago. A) NO CHANGE
B) My grandmother had started raising Rosie, it turns B) Too small, too light, it seemed to have lain dormant
out, as a desert tortoise over twenty years ago. for me for over thirty years, one that contained the
C) Rosie, it turns out, is a desert tortoise that my building blocks of an entire language.
grandmother had started raising over twenty years C) Containing the building blocks of an entire
ago. language that for me seemed too small, too light, it
D) Rosie, it turns out, is a desert tortoise that started had lain dormant for over thirty years.
raising my grandmother over twenty years ago. D) Seemingly lying dormant for me for over thirty
years, the building blocks, too light, too small, were
of an entire language.

Believing that many more voices need to be heard, 39


people in the Hmong community are urged to write, to
In 1980, physicist Luis Alvarez and his son, geologist
add their voices to the literary community.
Walter Alvarez, announced a startling discovery.
39 analyzing rocks from the same geologic layer in both

A) NO CHANGE Europe and New Zealand, 41 traces of iridium were


B) urging people in the Hmong community found, an element more common in outer space than on
C) people in the Hmong community whom Moua
Earth.
urges
D) she urges people in the Hmong community 41

A) NO CHANGE
B) iridium was found in trace amounts,
C) they found traces of iridium,
D) finding traces of iridium,

CONTINUE
Modifier Placement 103

Beverly Harvard—the chief of police in Atlanta and the At the heart of Rogers’s humor was a respect for every
first African American woman to lead a major city’s human being. Nevertheless, there was a sharp edge to
police department—began her career in law enforcement his wit. Proud of his Native American roots, 44 he
in an unusual way: she answered a challenge made by her ridiculed the snobbery of those who bragged about their
husband. 42 A woman her size (five feet, four inches), illustrious forebears: “My ancestors didn’t come over on
he didn’t think she had the strength to pass the grueling the Mayflower—they met the boat.”
physical tests required for police recruits. 44
42 A) NO CHANGE
A) NO CHANGE B) there was ridicule of the snobbery
B) At five feet, four inches, he didn’t think a woman C) the snobbery was ridiculed
her size D) ridiculing the snobbery
C) He didn’t think a woman her size (five feet, four
inches)
D) A woman her size, he didn’t think that at five feet,
four inches she After we pull the heavy turtle back to the water, I don
fins, a mask, and a snorkel 45 watching the turtle after
it’s released.
Withers’ profession gave him access to famous
45
people. 43 He has photographed seven of the last eight
A) NO CHANGE
U.S. presidents. He has also photographed every major
B) that watch
civil rights leader since the 1950s. C) in order to watch
D) that will be watching
43

Which choice most effectively combines the underlined


sentences?
A) He has photographed seven of the last eight U.S.
presidents and every major civil rights leader since
the 1950s.
B) He has photographed seven of the last eight U.S.
presidents since the 1950s and every major civil
rights leader.
C) He has photographed seven of the last eight U.S.
presidents, who include every major civil rights
leader since the 1950s.
D) He, seven of the last eight U.S. presidents, has also
photographed every major civil rights leader since
the 1950s.
11 Precision and Concision

Mom’s favorite wedding was a wonderful blending Ms. Komiyama moved to her house in 1948, when 2
of cultures and traditions: the bride and groom were she had reached the age of eighteen years old.
Hungarian immigrants dressed in American denim; 2
the 1 ceremony of two-minute duration which was in
A) NO CHANGE
English was followed by hours of Hungarian celebrations. B) her age was eighteen years old.
1 C) she had attained eighteen years in age.
D) she was eighteen years old.
A) NO CHANGE
B) two-minute ceremony
C) two-minute ceremony that took only a minute and
was Taeko Komiyama, however, can reach the 3 summit
D) ceremony that took two minutes and that was attaining it in only forty minutes.

A) NO CHANGE
B) summit
C) summit’s top
D) summit by getting there

104
Precision and Concision 105

Even astronauts orbiting the earth chew sugar-free 4 But I think my 7 experience needless to say was richer
bubble gum while in orbit. than theirs.

4 7

A) NO CHANGE A) NO CHANGE
B) bubble gum during their time in space. B) experience was
C) bubble gum which does not have sugar in it. C) experience indeed was
D) bubble gum. D) experience can be considered to be

A finger jabbed my arm as I waited for my order at a My sons sit in a domed, bowl-shaped park where the
taco stand in Santa Fe. “You don’t want to order it hot,” players meander about on artificial turf 8 that is not
a man 5 I didn’t know due to the fact we hadn’t met natural grass.
insisted, flagging down the stand owner. 8
5 A) NO CHANGE
A) NO CHANGE B) that is not real.
B) I had never met C) that’s not grass.
C) of whom I had not been introduced D) DELETE the underlined portion and end the
sentence with a period.
D) hitherto before I had not made the acquaintance of

We stopped outside the front gate of the large


I was too surprised at the stranger’s audacity to object,
governmental post 9 office of some size, and my
but 6 it was soon, little time had passed, that as I
friend showed me what to do.
forked up my burrito, tears of gratitude streamed down
my face. 9

6 A) NO CHANGE
B) office, which was quite sizable, and
A) NO CHANGE C) office, so sizable that
B) not long after, it was soon when D) office, and
C) before long, right away
D) before long,

CONTINUE
106 Precision and Concision

Still emphasizing self-help 10 as a route in order to Some parents have been moved by worries about the
get more self-esteem, they created a Department of lyrics’ influence on their children to 13 be motivated
Commerce to coordinate future business development to seek ways of restricting access to songs rock bands
with the needs of existing enterprises. deplore.

10 13

A) NO CHANGE A) NO CHANGE
B) to route B) be motivated toward seeking
C) as a route to C) have sought
D) as a way to gain and achieve more D) seek

11 At first, Tan originally tried to write her stories One possible solution to the censorship dilemma is to
from a non-Chinese perspective, assuming that work rate albums as we do movies. Ratings give parents a
from a Chinese-American point of view would never be rough guide to the nature of 14 a recording’s contents.
published. 14
11 A) NO CHANGE
A) NO CHANGE B) the contents of a recording that could have been
put on tape, compact disc, or record.
B) Tan originally
C) a recording’s, on record, compact disc, or tape,
C) First, Tan originally contents.
D) In the beginning, Tan first D) the recording whether it is on compact disc, record,
or tape.

I recently added a second 12 bathroom to my house,


doing all the work myself. He is celebrated by art historians for his vivid and
haunting images, many of which continue to be
12
published in children’s books 15 for the young today.
A) NO CHANGE
B) bathroom, adding 15
C) bathroom in addition
A) NO CHANGE
D) bathroom not long ago
B) for young people
C) for young readers
D) DELETE the underlined portion.

CONTINUE
Precision and Concision 107

One afternoon, I 16 obtained the nourishment of some Attend a Fourth of July celebration these days and you’re
Texas-style barbecued chicken and apple pie. likely to see fireworks exploding into 19 a surprisingly

16 astonishing array of effects, from Statues of Liberty and


U.S. flags to arrays of celestial flowers.
A) NO CHANGE
B) lunched on 19
C) nourished myself by way of
A) NO CHANGE
D) partook nourishment in the form of
B) an astonishing
C) a spectacularly astonishing
D) an astonishingly amazing
“Compose poetry while working in the yard,” my
friend 17 told and advised me, “so that your muscles
will activate your creative juices.” Smith realized the extent of her gift when her friend Alex

17 Haley, who had gathered essential material 20 critical


to writing his best-selling novel Roots from a griot in
A) NO CHANGE
B) told Gambia, began to refer to her as “my American griot.”
C) informed and told 20
D) narrated and related to
A) NO CHANGE
B) for
C) important to
Having recently moved to Florida, I find myself living in D) that was essential to
a subtropical environment for 18 the initial first time.

18
Today, Smith’s repertoire is so vast that she could
A) NO CHANGE
B) initially the speak 21 consecutively for twelve hours straight
C) primarily the without running out of material.
D) the
21

A) NO CHANGE
B) continuously nonstop
C) perpetually
D) DELETE the underlined portion.

CONTINUE
108 Precision and Concision

When a batter finally makes contact, the ball dribbles I wonder how my cart will age, whether it will warp or
into the infield, where the nearest player usually 22 wobble or develop an endearing squeak. I enjoy 25 the
ends up throwing the ball at the first baseman’s feet or, if operating of it because it makes me feel like an urban
the fielder is precocious, over the first baseman’s head. citizen.

22 25

A) NO CHANGE A) NO CHANGE
B) accomplishes a result of B) the experience of operating it
C) attains the consequence of C) making use of it as a possession
D) results in D) using it

Alligators are native to the south eastern United Each afternoon, Woods put her babies in bed for their
States. 23 They live and reside in freshwater ponds, naps, a time she cherished for her own 26 personal
lakes, swamps, marshes, and streams. literary scholarship and reading.

23 26

A) NO CHANGE A) NO CHANGE
B) The habitat where they live is B) investigation of written texts and
C) Living in the habitat of C) relaxation with books and
D) They live in D) DELETE the underlined portion.

An alligator’s brain is small—about the size of a walnut. How does a bilingual speaker decide what language
Despite this, alligators are 24 superb and excellent to use? The language of the speaker’s most recent
hunters. conversation can be the 27 determining factor.

24 27

A) NO CHANGE A) NO CHANGE
B) superbly excellent B) factor that decides the determination of the
C) excellently superb language choice.
D) superb C) factor responsible for determining the language
choice decision.
D) factor that determines it.

CONTINUE
Precision and Concision 109

The plane has been stripped of all its seats, so everyone is It’s often hard to determine who first uttered a given
kneeling on the bare metal floor 28 on their knees. word and almost as hard to know who first wrote it

28 down 31 originally.

A) NO CHANGE 31
B) down on their knees. A) NO CHANGE
C) resting on their knees. B) in the beginning.
D) DELETE the underlined portion and end the C) initially.
sentence with a period.
D) DELETE the underlined portion and end the
sentence with a period.

The plane is headed to 29 an altitude of three thousand


feet. Lin selected these words from 32 a Dr. Martin Luther

29 King Jr. speech called “I Have a Dream” in 1963.

A) NO CHANGE 32
B) three thousand feet up in altitude. A) NO CHANGE
C) an altitude of three thousand feet up. B) a speech given by Dr. King in the sixties,
D) an altitude of three thousand feet up in the air. specifically 1963.
C) the “I Have a Dream” speech by Dr. Martin Luther
King Jr. himself.
D) Dr. Martin Luther King Jr.’s 1963 “I Have a Dream”
For as long as I remember, 30 it is true that I have speech.
wanted to play the harmonica.

30
Lin continues to challenge herself in a career that
A) NO CHANGE
combines an appreciation for sculpture, architecture, and
B) it is a fact that
C) it has always been the case that the 33 complexity of human experience.
D) DELETE the underlined portion.
33

A) NO CHANGE
B) complexity of the nature and character
C) characteristically complex and inherent
complications
D) complications of the character and complex nature

CONTINUE
110 Precision and Concision

In our one-acre garden, the 34 weight of the tomato At the parade’s end, he offered the flour to the highest
plants would slump with ripening fruit, the stems would bidder, starting with his own $200 bid. He stated that
break, and the pungent smell of late summer would be proceeds would go to the U.S. Sanitary Commission, 36
released. an organization equivalent and similar to today’s Red

34 Cross, to aid Civil War soldiers.

A) NO CHANGE 36
B) size of the A) NO CHANGE
C) ripening B) the equivalent of
D) DELETE the underlined portion. C) a similar equivalent of
D) the self-same organization equivalent to

About the same time, the Roman author Pliny


pronounced the yew “unpleasant and fearful to look Tran knew that he and Maya would have to 37 apply
upon.” The old British practice of planting yews in their energies to the activity of labor for the money
cemeteries did nothing 35 in changing or altering that needed to sign a lease.
reputation. 37
35 A) NO CHANGE
A) NO CHANGE B) employ themselves vigorously to the task at hand
B) to alter that acquired C) work a long time
C) in the way of changing that D) accomplish it
D) to alter that

Soon McLean stepped onstage with his sax. I wasn’t 38


prepared for the impact. Into every corner of the room,
he blew wild, sweet, almost disjointed notes that seemed
snatched from a dozen different places at once.

38

A) NO CHANGE
B) ready for the unexpected impact.
C) prepared, regarding the impact, for what happened.
D) expecting the impact, which I didn’t see coming.

CONTINUE
Precision and Concision 111

Art critics and historians consider the negatives 42 Advocates supporting noise pollution controls are
significant for several reasons. First, 39 since they came concerned about the noise from construction equipment
from the past, Mo Williams’s photographs provide a rare and airplanes.
and valuable record of the daily life of African Americans 42
in a small town in the early 1900s.
A) NO CHANGE
39 B) Advocates who are in favor of
C) Advocates of
A) NO CHANGE
D) People who advocate for and support
B) being that they have preserved the past,
C) because they are from a long time ago,
D) DELETE the underlined portion.
For some people, traditional American Indian music
is 43 associated and connected with high penetrating
Dr. Cheng now believes that a fourth hypothesis is 40 vocals accompanied by a steady drumbeat.
the likely explanation. 43
40 A) NO CHANGE
A) NO CHANGE B) connected by some of them
B) in all probability the likely C) linked by association
C) likely the most probable D) associated
D) very probably the likely

Among the O’odham tribes of Arizona, waila has


At first, I was 41 alarmed like a distress signal was been 44 popular for more than a century.
flashing, lighting up, but the horse was moving slowly, 44
and I realized that I would be able to hang on.
A) NO CHANGE
41 B) popular, one might say, for
C) really quite popular for
A) NO CHANGE
D) popular for the duration of
B) alarmed with fear and distress,
C) alarmed by the immediate awareness of danger,
D) alarmed,

CONTINUE
112 Precision and Concision

After a certain age, many Americans I know would 45 Underwater explorer Jacques 48 Cousteau, exploring
balk, refuse, and hesitate at the idea of adding a year or under the sea, predicted that one day humans would be
two to what they regard as their actual age. “freed from the bondage of the surface” to work and even

45 live underwater.

A) NO CHANGE 48
B) balk and hesitate A) NO CHANGE
C) refuse and balk B) Cousteau—exploring under the sea—
D) balk C) Cousteau who explored under the sea,
D) Cousteau

Many people were excited when they heard about 46


this seemingly true and factual proof of the existence of In the 1960s, this became reality when the U.S. Navy
fairies, but Conan Doyle was more excited than most. deployed its first undersea habitat, SeaLab. 49

46 49

A) NO CHANGE At this point, the writer is considering adding the


B) this seemingly evident and apparent following phrase, replacing the period after SeaLab with
a comma:
C) what seemed to be apparent
D) this apparent an underwater environment where people can live
and work.
Should the writer add this phrase here?
A) Yes, because it offers a brief definition of the SeaLab
Though the two remained cordial, their friendship habitat.
was damaged 47 due to the fact that they had the B) Yes, because it helps the reader envision what the
Navy deployed.
disagreement. C) No, because the SeaLab has already been similarly
described.
47
D) No, because it provides a digression that leads the
A) NO CHANGE sentence away from its primary focus.
B) by the fact of the
C) due to the fact of their having the
D) by the

CONTINUE
Precision and Concision 113

The achievement that Dr. George Franklin Grant is most It was a routine Saturday 53 evening. The clattering of
widely remembered for today is his 50 invention and pots and pans echoed through the house.
creation of the wooden golf tee. 53
50 A) NO CHANGE
A) NO CHANGE B) evening, like many others.
B) invention, which was the creation C) evening, typical for us.
C) invention, that is, the creation D) evening, just as usual.
D) invention

54 In due course, after waiting a while, my neighbor


In May, 1863, five adventurers camping in a gulch high signaled for the waiter and asked him why he still hadn’t
in the Rocky Mountains 51 happened upon one of the received his mushrooms.
largest gold deposits in North America. 54
51 A) NO CHANGE
A) NO CHANGE B) A little later,
B) accidentally stumbled up against C) While waiting in due course,
C) unintentionally without planning discovered D) After a due course of time,
D) ascertained and verified the existence of

My prairie eye and my woods eye are 55 equally


In 1884, Sarah remarried, this time to a successful 52 dominant in terms of strength.
man who was twenty years older than she and who was 55
also a gold miner, Stephen Bickford.
A) NO CHANGE
52 B) equal and the same strength wise.
C) the same when strength is considered.
A) NO CHANGE
D) equally strong.
B) man older than she was in years who had been
engaged in gold mining,
C) gold miner twenty years her senior,
D) pioneering old-timer gold miner,

CONTINUE
114 Precision and Concision

Last week, my father told me he had to go on a business Often these sights are the lovely outcome of federal,
trip 56 in connection with his work and that I’d be state, and local programs to restore 59 and preserve
staying with his sister for three days. the lasting character of the natural beauty of the national

56 landscape.

A) NO CHANGE 59
B) having something to do with his job A) NO CHANGE
C) that involved traveling B) and preserve
D) DELETE the underlined portion. C) the character of the preservation of
D) in a characteristic fashion the preservation of

Today, many people use kayaks recreationally for white-


water sports and for touring wilderness areas 57 that Since Wright’s death in 1959, the school has continued to
are extremely wild. thrive thanks to its Fellowship members, former students

57 of Wright’s who now serve as teachers and mentors for


others. 60 These teachers and their students work,
A) NO CHANGE
B) of great remoteness. study, and build together, using Wright’s guiding theories
C) that are uncivilized. and the skills they’ve learned to create buildings that are
D) DELETE the underlined portion and end the both beautiful and functional.
sentence with a period.
60

A) NO CHANGE
Banneker quickly became engrossed in his studies and B) Side by side and hand in hand, these
began to calculate the paths of the Sun, Moon, and other C) As one group with each other, these
D) By cooperating and working together, these
celestial bodies. Using 58 them, he predicted a solar
eclipse that occurred the next year.

58

A) NO CHANGE
B) these calculations,
C) those,
D) these things,

CONTINUE
Precision and Concision 115

The book arrived in a disappointing package. It was small Often, when I feel a need 64 within myself to have a
and narrow, wrapped in plain brown paper. My mother’s tranquil moment, I try to summon images of the tall
faint 61 handwriting covered the outside. grasses and furry cattails and the mallards, herons,

61 and geese that can be found in the 32,000-acre marsh.

A) NO CHANGE 64
B) handwriting, which she did right-handed, A) NO CHANGE
C) right-handed handwriting, B) to experience a condition of tranquility,
D) writing, using her right hand, C) for tranquility,
D) for it,

I 62 glanced and looked through the pages. Then I read


about the different parts of speech, about Miami vowels “Auntie” is a traditional title of respect in the African
and consonants. nation of Ghana, Ocloo’s native land. Ocloo knew

62 that 65 her countrywomen would need to have a steady


source of income.
A) NO CHANGE
B) was looking and glancing, 65
C) glanced
A) NO CHANGE
D) glanced, looking
B) her countrywomen who called Ghana their
homeland
C) the women who lived in and had grown up in
Ghana
I hadn’t been to Horicon Marsh in a few years, but my
D) her fellow women in the country of Ghana
memories of visiting the preserve were still 63 vivid.

63

A) NO CHANGE
B) vivid up to the present moment.
C) vividly.
D) vividly fresh and alive.

CONTINUE
116 Precision and Concision

During World War II, millions of women took industrial In fact, the crater had been detected by Glen Penfield, 69
jobs in shipyards, aircraft plants, steel mills, and other on the job as a geologist working in the Yucatan region of
defense-related enterprises throughout the United States. Mexico.
The labor force that 66 shrank and decreased as men 69
joined the military forces grew as women flooded into
A) NO CHANGE
jobs newly open to them. B) a geologist working
66 C) since he was a geologist who worked
D) the geologist who made the discovery while at
A) NO CHANGE work
B) shrank
C) decreasingly shrank
D) DELETE the underlined portion.
Many scientists theorize the Chicxulub asteroid event
helped bring about the extinction of the 70 dinosaurs,
which happened and took place about 65 million years
Saving barns makes good sense for 67 more than a
ago.
variety of reasons.
70
67
A) NO CHANGE
A) NO CHANGE
B) dinosaurs by means of occurring
B) a various number
C) dinosaurs
C) a variety
D) dinosaurs, which happened, as a matter of fact,
D) a varied assortment

Though engineering projects have shrunk the Platte


With a brief course in the proper use of scuba gear, even
and 71 thus have in reality drastically reduced the
a beginner can dive safely when accompanied by an
cranes’ habitat, each year they return, messengers of
instructor 68 acting as a guide.
spring in a land just emerging from bitter winter.
68
71
A) NO CHANGE
B) whose job is to instruct, teach, and guide him or A) NO CHANGE
her. B) thus have in fact
C) who goes along as a scuba teacher and educator. C) lessened and
D) instructing the diver, as a teacher. D) DELETE the underlined portion.

CONTINUE
Precision and Concision 117

Historians speculate that one of the world’s oldest toys is Cox knew, however, that the longer she stayed in
the yo-yo. Drawings 72 of objects adorn ancient Greek the water, the longer it would take to bring her body
vases and the walls of Egyptian temples, and written temperature—which fell to 95.5 degrees Fahrenheit
mention of yo-yos goes back to the fifth century B.C. by the end of the swim— 74 back to a normal body

72 temperature.

A) NO CHANGE 74
B) that call attention to objects that look something A) NO CHANGE
like the toy that I have just mentioned
B) back to normal.
C) that include objects that almost slightly resemble
yo-yos C) in other words, back to normal.
D) of objects resembling yo-yos D) which was normal.

Antarctic seals rely on body fat for warmth, so Cox In 1946, he left the Army and began working 75 at a

gained twelve pounds, 73 it was weight that she hoped job that was a self-employed photographer.

would keep her warm in the icy water. 75

73 A) NO CHANGE
B) for himself as
A) NO CHANGE
C) as
B) she put on weight
D) DELETE the underlined portion.
C) she gained it so
D) weight

Before retiring 76 recently after being employed, she


worked for many years as a waitress.

76

A) NO CHANGE
B) recently, so that she doesn’t have to go to work
anymore,
C) recently and resigning from her position,
D) recently,

CONTINUE
118 Precision and Concision

She certainly doesn’t fit the stereotypical image 77 The first person 79 initially to offer a scientific
associated with that of a typical fan of American football. explanation of rainbows was Rene Descartes.

77 79

A) NO CHANGE A) NO CHANGE
B) that is often seen to be closely connected with that B) in an initial attempt
of C) with an initial idea
C) that is linked up with that of D) DELETE the underlined portion.
D) of

She eagerly studies the weekly listing of upcoming


televised games 78 that comes out every week.

78

A) NO CHANGE
B) that appears on a regular basis.
C) which is published regularly.
D) DELETE the underlined portion and end the
sentence with a period.
12 Style and Tone

The process takes hours, but you remain quiet and Anderson’s splendid voice was broadcast into homes all
still. You are 1 validating the sight of these odd, across the country. With her mastery of various musical
slow, determined beings and their prehistoric ritual. styles and the richness and control of her renditions, she
You realize that there is nothing quite as astounding as made that Easter Sunday 2 one of the nicest days of
witnessing one of life’s subtler and more elusive natural that month.
wonders. 2
1 Which choice most effectively summarizes the impact of
the event?
Which of the choices best emphasizes the writer’s intense
involvement in witnessing this process? A) NO CHANGE
A) NO CHANGE B) a very important religious holiday.
B) plunged in C) a nationwide celebration of song.
C) immersed in D) the dream of the woman who sponsored the event.
D) corroborating

119
120 Style and Tone

More recently, American culture during the 1980s, 3 5 Reading would have long been regarded as a private
typified by the popular song “Don’t Worry, Be Happy,” act, one best undertaken in silence. People often think
fostered a host of domestic problems. of reading as a way to escape from the concerns of the

3 world.

The writer wants to provide an example of 1980s 5


American culture superficially celebrating the positive.
Which choice most effectively accomplishes the writer’s A) NO CHANGE
goal? B) If reading had been long
A) NO CHANGE C) Reading has long been
B) when the baby boomers became the “thirty- D) Suppose reading were long
something” crowd,
C) which was certainly not an easy time for everybody,
D) despite a worldwide trend toward greater
democratic freedom, The man next to her took my package and carefully
sewed the edges of the cloth with thick, black thread. 6
He paused often to talk to me and my friend.
For her new home she chose to live on the fourth-
6
highest peak in Japan’s Hakone range, Mt. Kintoki, which
The writer wants to highlight the worker’s care and
rises 1,213 meters above the sea. 4 Climbing up Mt. attention to his task. Which is the most effective choice?
Kintoki, one passes many trees. A) NO CHANGE
B) The heavy thread made a very noticeable seam.
4 C) He took the time to line up each short, tight stitch.
Which choice most effectively describes the natural D) It seemed that he was a very diligent worker.
landscape that hikers visiting Mt. Kintoki see?
A) NO CHANGE
B) The path up Mt. Kintoki passes through plains,
then climbs through bamboo groves and pine
forests before it reaches the barren summit.
C) The Hakone Mountains are very popular, and the
path toward Taeko Komiyama’s home is a well-
known, pleasant climb.
D) Mt. Kintoki is located only about two hours
from Tokyo, so many tourists climb its slopes on
weekends, and a lesser number make the ascent on
weekdays.

CONTINUE
Style and Tone 121

The actors who perform in television commercials are I saw an old-fashioned honey-gatherer selling wild honey
some of the most fortunate people in show business, 7 he had brought down from the mountains. From two
but their faces aren’t as well known as those of most great pots, which were tied to a pole he carried across his
movie actors. shoulders, the man poured honey into 9 empty jars.

7 9

Which choice best highlights a contrast with the previous The writer wishes to emphasize the meeting of traditional
statement? and modern cultures. Which choice most effectively
A) NO CHANGE accomplishes this goal?
B) and if they are lucky, these actors can move from A) NO CHANGE
commercials to film. B) his customers’ empty imported peanut butter jars.
C) and they usually have fairly short contracts. C) his customers’ jars, which quickly filled with the
D) since people who act in commercials are generally sweet stuff.
paid more than most movie actors. D) containers that were supplied by his customers.

The first stage is marked by eagerness. One morning As I pushed the lawnmower back and forth 10 with all
you tear into a wall, feeling 8 a trickle of enthusiasm my strength, I composed a couplet: “The graceful grass
as plaster crumbles beneath your wrecking bar. That beneath my feet /Is crushed and trimmed in one big
enthusiasm sustains you for days as you take out the old sweep.”
to build the new. 10
8 The writer wants to convey the boredom of cutting grass.
Which choice best achieves this goal?
Which choice maintains the logical and stylistic
coherence of the paragraph? A) NO CHANGE
A) NO CHANGE B) in a monotonous march,
B) a surge C) and grass clippings covered my shoes,
C) a twinge D) slowly and carefully,
D) an ounce

CONTINUE
122 Style and Tone

In New York City, Arthur Mitchell, director of the Tradition has it that the first fireworks were created in
fledgling Dance Theater of Harlem, 11 listened to her the eleventh century when Chinese soldiers blasted
improvising on the piano one day. their 13 gunpowder into the sky to celebrate a victory

11 in battle.

The writer wants to express the coincidental nature of 13 ♦

the event. Which choice most effectively accomplishes


Which choice adds the most detailed and relevant
this goal?
information to the sentence?
A) NO CHANGE
A) NO CHANGE
B) happened to overhear
B) gunpowder while others held their hands over their
C) witnessed and admired ears.
D) was impressed with C) gunpowder while they were in the midst of some
type of celebration.
D) gunpowder, which might have been a custom at the
time.
Getting used to various new animals is a different story.
At first I was startled by the small lizards, the 12 teeny
creatures that are everywhere, even inside the house. The Navajo code remained classified after the war and
12 was later used, along with codes made from other

The writer wants to emphasize being startled. Which American Indian languages, in the Korean Conflict and
choice most effectively achieves this goal?
the Vietnam War. Now that the Navajo code is no longer
A) NO CHANGE
used, the code talkers, whose 14 secret work saved
B) miniature “dinosaurs” that unexpectedly dart into
your path, American lives, can finally receive public recognition for
C) wiggly things that walk about,
their actions.
D) reptiles that call attention to themselves,
14

A) NO CHANGE
B) hush-hush actions
C) concealed, hidden efforts
D) doings under wraps

CONTINUE
Style and Tone 123

Raptors, or birds of prey, cannot afford to be grounded “Beautiful,” Renee 17 exclaims, “just like Janet Jackson.”
for weeks waiting for a large number of flight feathers 17
to regrow. They must be able to fly if they are to hunt
Which choice most effectively emphasizes the sense of
and eat. Raptors, including eagles and hawks, therefore approval in Renee’s statement?
A) NO CHANGE
normally shed their feathers slowly, 15 one or two at a
B) verbalizes,
time.
C) comments,
15 D) states,

Which choice best completes the description of the


feather-shedding process?
A) NO CHANGE Only seconds pass before the static line, which is a
B) in an all-at-once blizzard.
rope attached to you and to the floor of the plane, 18
C) often losing them in clumps.
D) leaving them flightless for long periods of time. becomes tight and opens your parachute automatically.

18

The writer wants to emphasize the violent action of


It will take Renee all morning to 16 remake me into the parachute opening. Which choice most effectively
achieves this goal?
the image and likeness of a stately and striking African
A) NO CHANGE
American diva. B) tightens and opens
C) becomes taut and yanks open
16
D) quickly becomes taut and opens
The writer would like to emphasize the drama of the
change. Which choice most effectively achieves this
goal?
A) NO CHANGE
B) transform me from an everyday mother of two into
C) perfect the hairstyle that will turn me into
D) ply her trade so that I end up looking like

CONTINUE
124 Style and Tone

My father was able to teach me a few things about the I can see myself now, standing in a steamy kitchen,
harmonica. He told me that it originated in southern overalls spattered with pulp, sweat dripping from my
China around 1100 B.C. This early version, the sheng, chin. It was impossible to keep a straight face as seeds
consisted of long reeds placed in water in a wooden squirted through the 21 air and splatted on the walls
bowl. At one time, the harmonica was often confused around us.
with the armonica, a musical instrument invented by 21
Benjamin Franklin 19 The most difficult thing about
Which choice most effectively reinforces the sense of
playing the harmonica is pulling air in through the holes; humor described in the first half of the sentence?
A) NO CHANGE
blowing out is easy.
B) air while we watched.
19 C) air, creating a sight to behold.
Which choice most effectively continues the historical D) air on those hot summer days.
sketch of the harmonica?
A) NO CHANGE
B) The harmonica as we know it wasn’t introduced
Then one of the other musicians would take the lead,
until 1828 in London.
C) London, where the harmonica was introduced, is a pulling strange patterns from his instrument, 22 the
foggy city frequented by tourists who enjoy visiting
equivalent in sound of someone dancing under a
Big Ben and Buckingham Palace.
D) Some of Benjamin Franklin’s more famous blinking strobe light.
inventions include the Franklin stove and the
lightning rod. 22

The writer would like to describe the sound of the music


in visual terms to help the reader imagine the jazz played
that night. Which choice best accomplishes this goal?
Many of the tales Hurston heard during her Southern A) NO CHANGE
trips 20 revealed interesting information: why the B) the highlight of the evening for some audience
members.
woodpecker has a red head, why the alligator is black, or
C) the sound making me see why so many people like
how the snake got its rattles. jazz.
D) the notes having a visible effect.
20

Which choice most effectively sets up the examples that


follow in the sentence?
A) NO CHANGE
B) explained the origins of animal characteristics:
C) offered important insight into the past:
D) concerned long-ago events:

CONTINUE
Style and Tone 125

My father was a Linotype operator, and the Linotype Talk to friends and neighbors, and you’ll have a hard
machine seemed formidable. My father’s hands flew time finding anyone who claims to be in favor of air and
across an enormous keyboard that was 23 unlike any I water pollution. Our environment is a common resource,
saw in other places. something 25 we all enjoy during exciting weekends in

23 the park.

Which choice best completes the description of the 25


keyboard?
Which choice most effectively concludes the sentence
A) NO CHANGE with an emphasis on citizen involvement in preserving
B) something amazing to behold. the environment?
C) like several typewriter keyboards spliced together. A) NO CHANGE
D) like the alphabet waiting to be turned into words. B) we all have a right to use and a responsibility to
protect.
C) that reminds us of our relationship to the land and
its contribution to our peace and well-being.
24 Stormy must have sensed that I was a novice. D) we all reflect upon and appreciate for the many
benefits it provides.
Suddenly, on her own, she started to walk toward a field
of grass. At first, I was alarmed but she was moving
slowly, and I realized that I would be able to hang on.
In 1970, the school board in Pittsfield, New Hampshire,
24
approved a dress code that prohibited students from
Which choice most effectively introduces the actions wearing certain types of 26 clothing.
that follow?
A) NO CHANGE 26
B) Amil really knew how to ride!
Which choice most effectively completes the sentence so
C) Though very tall, Stormy was a gentle creature. that it illustrates the term dress code (as it is used in the
D) Amil had bought Stormy from another rancher sentence)?
some years ago. A) NO CHANGE
B) clothing that were considered inappropriate.
C) clothing, including sandals, bell-bottom pants, and
“dungarees” (blue jeans).
D) clothing that are permitted in some schools today.

CONTINUE
126 Style and Tone

Though Hernandez died in 2000, her legacy—the Ballet In 1917, when our house was built, city planners
Folklorico—lives on, 27 as an outstanding dance developed an orderly scheme for the collection of trash.
company. Garbage cans would be picked up from the narrow alleys

27 that ran behind the rows of houses. That way, the 29


standard-width sidewalks that the houses faced would
Which choice most effectively describes the valuable
contributions made by the Ballet Folklorico? remain free of trash receptacles.
A) NO CHANGE
29
B) sharing Mexico’s cultural heritage with the world.
C) as a well-known performing group. Which choice best supports the logic of the city planners’
D) showing that the group will continue into the trash collection system described in this paragraph?
future. A) NO CHANGE
B) paved cement
C) tidy, tree-lined
D) old, familiar
Not long ago, the most advanced in a series of undersea
habitats, Aquarius, was rested on the ocean floor in
the Florida Keys. 28 University of North Carolina
Today’s garbage trucks are enormous, far wider than
professors worked on the Aquarius project.
the slender alleys. 30 Unlike the older trucks, they are
28 painted bright yellow.
Which choice best explains which governing bodies are
30
in charge of the Aquarius project?
A) NO CHANGE Which choice provides the most specific support for the
B) Aquarius is operated jointly by the National statement in the preceding sentence?
Oceanic and Atmospheric Administration and the A) NO CHANGE
University of North Carolina. B) Twice a year, city residents pick up litter from the
C) University professors and scientific researchers alleys.
were often at odds while collaborating on the C) Indeed, the new trucks can barely navigate
Aquarius habitat. ordinary city streets.
D) Several special interest groups support research D) Nevertheless, the alleys remain free of trash and
related to the Aquarius habitat project. yard clippings.

CONTINUE
Style and Tone 127

The waiter went to the kitchen and returned 31 in a The fire grows steadily throughout the day as the potter
moment. feeds it lumber scraps and then logs. By nightfall 33 a

31 controlled inferno roars in the kiln.

Which choice most clearly and effectively expresses the 33


waiter’s promptness?
The writer would like to highlight the fire’s extreme heat.
A) NO CHANGE Which choice best achieves this goal?
B) when he had a moment. A) NO CHANGE
C) some time later. B) the fire is stronger than ever
D) after a while. C) there is more heat being produced
D) a kind of intense blaze takes place

Unbricking a kiln after a firing is like uncovering buried


treasure. As the potter 32 takes bricks away to create an Tortoises are land-dwelling, vegetarian turtles. They eat
opening into the oven, an expanding view of gleaming a diet of grass clippings, lettuce, broccoli, melons, and
shapes rewards the artist for months of hard work. other 34 vegetables and fruit.

32 34

The writer would like to suggest caution and anticipation Which choice provides the most specific and precise
in opening the kiln. Which choice best achieves this information?
goal? A) NO CHANGE
A) NO CHANGE B) things they could eat.
B) removes bricks by hand C) edibles.
C) removes one brick at a time D) fresh foods.
D) experiences great anticipation and quickly removes
bricks

After his father died in 1759, Banneker took over the


responsibility of the family farm and the care of his mother
and younger 35 sisters. In addition, he pursued scientific
studies and taught himself to play the flute and violin.

35

A) NO CHANGE
B) sisters. Therefore,
C) sisters, thus,
D) sisters, therefore,

CONTINUE
128 Style and Tone

In the hot waters surrounding Juan de Fuca Ridge in The tan wood and rusted red steel of the residence— 38
the Pacific Ocean, thermophiles ensure the survival of which presents an interesting sight to catch the eye—
other marine life. 36 Here, the ocean floor is scarred by mirror the colors of the surrounding desert.
earthquakes and underwater volcanoes. 38
36 Which choice adds the most effective visual detail to
complete the description in the first part of the sentence?
Which choice most specifically and vividly describes the
underwater terrain? A) NO CHANGE
A) NO CHANGE B) an architectural structure that took the student
several weeks to build—
B) Here, there are signs of both seismic and volcanic
activity. C) which resembles a railroad boxcar in size and
shape—
C) Here, the results of earthquakes and volcanic
eruptions are evident. D) a piece of architecture positioned in the Arizona
desert—
D) Here, the effect of earthquakes and volcanic activity
is apparent.

I had expected something larger and heavier, as


I soon came to love the silence of the photo substantial as my Webster’s Dictionary. 39 A folded note
darkroom, 37 illuminated only by its dim, red safelight. from my mother was tucked between the pages.
Something about that isolation and darkness seemed 39
mysterious.
The writer would like to emphasize the difference
37 between the writer’s expectations of the book and its
actual appearance. Which choice best achieves this goal?
Which choice most effectively connects this sentence A) NO CHANGE
with the following one? B) Instead, the Miami handbook rested comfortably in
A) NO CHANGE the palm of my hand.
B) located in the Art Building on the south campus. C) In fact, it lacked a paper jacket to protect the cover.
C) which was open for use by photography students. D) These pages contained important cultural history.
D) occasionally shared with others from the class.

CONTINUE
Style and Tone 129

I was immediately overwhelmed with images of my In 1990, Auntie Ocloo shared the Africa Prize for
great-grandfather. I remembered the way he used to Leadership with President Olusegun Obasanjo of Nigeria
read to me when I was very young. I remembered him for their efforts to fight hunger. 42 Ocloo always wore
pointing to a tree, a cloud, a dog, and naming them in bright African clothes and loved to cook. She helped
the Miami language, sharing them with me in the 40 them, for example, learn to calculate costs and profits on
language of his own childhood. the rice and stew they sell on street corners like the one

40 where Auntie Ocloo first sold her marmalade.

The writer wants to suggest the significance that the 42


Miami language held for her great-grandfather. Which
Which choice results in the most effective transition to
choice best achieves this goal?
the concluding sentence?
A) NO CHANGE
A) NO CHANGE
B) voice I remembered.
B) Ocloo used her half of the $100,000 prize to
C) Miami language. establish a program to train women in business.
D) words that were so hard for me to understand. C) Ocloo was the first woman recipient of the Africa
prize.
D) Ocloo’s company, Nkulenu Industries, produced
products like canned tomatoes and soup bases.
Often, when I feel a need for tranquility, I try to summon
images of the tall grasses and furry cattails and the
mallards, herons, and geese that can be found in Horicon Coleman had developed an unorthodox approach over
Marsh. 41 Thinking about the geese is always especially years of effort, first in his home state of Texas and then
comforting. in Los Angeles, where he was routinely snubbed by older
41 musicians 43 such as Dexter Gordon and Miles Davis.

Which choice best supports the point that memories of 43


visiting the wildlife preserve have a personal effect on
the narrator? Which choice provides the clearest and most effective
A) NO CHANGE explanation for why the older musicians snubbed
Coleman?
B) This is the largest freshwater marsh in the United
States. A) NO CHANGE
C) It’s been designated as a “Globally Important Bird B) disturbed by his experimental sound.
Area.” C) who happened to hear him play.
D) It’s an extremely large and beautiful place. D) who also played jazz.

CONTINUE
130 Style and Tone

Next came Coleman’s controversial but successful 1959 Largely as a result of the trust’s leadership, 46 on the
debut with his quartet at a New York club called the Five other hand, nearly a dozen states have launched barn-
Spot. This engagement—coupled with the release of two preservation efforts.
more albums (The Shape of Jazz to Come and The Change 46
of the Century)—reversed Coleman’s fortunes. 44 Some
A) NO CHANGE
musicians were concerned that his influence would ruin B) thus
jazz. C) through
D) DELETE the underlined portion.
44

Which choice best supports the claim made by the


previous sentence?
A) NO CHANGE Just imagine what you might see if you were to go diving.
B) Veteran players were now competing for the chance As you glide through the pleasant blue-green waters 47
to sit in with his band.
of the coral reef, some fish may swim up and peer at you
C) He had already won the approval of John Lewis of
the Modern Jazz Quartet. through the clear glass of your waterproof mask.
D) The alto sax player from Texas was twenty-nine
years old at the time. 47

The writer wants to describe the beauty and appeal of the


underwater scene. Which of the choices most effectively
achieves this goal?
Sculptures inspired by elements of the ships 45 can also A) NO CHANGE
be seen. Panels positioned among the sculptures display B) among the variety of sea plants and sea animals,
some of them
photographs, letters, welding certificates, and other
C) among brilliantly colored tropical fish, several
“Rosie” artifacts. friendly and harmless little ones
D) you and your fellow divers might be surprised at
45 what kinds of creatures
Which choice most effectively helps readers visualize the
memorial?
A) NO CHANGE
B) line the walkway.
C) add to the memorial’s impact.
D) make up part of the design.

CONTINUE
Style and Tone 131

Early each spring, a 48 chunk of land along the Platte Like many morel hunters, I learned from an expert. I
River in Nebraska is a temporary home to one of the learned even more by reading 50 reputable, detailed
world’s most spectacular wildlife displays: the migration field guides about wild mushrooms.
of the sandhill cranes. 50
48 Which choice provides the most relevant and specific
information?
Which choice most effectively portrays a length of land
extending along the river for a considerable distance? A) NO CHANGE
A) NO CHANGE B) in between trips to and from the woods.
B) ribbon C) to gain the expertise I wanted and needed at this
point.
C) nugget
D) very carefully on the topic that pertains to the
D) bundle activity.

I’m not talking about imaginary creatures, but about Historians speculate that one of the world’s oldest toys is
deliciously real morel mushrooms— 49 funny-looking, the 51 yo-yo, though they know for sure that the oldest
textured, edible fungi that appear in springtime. toy is the doll. Drawings of objects that look like yo-yos
49 adorn ancient Greek vases and the walls of Egyptian

Which choice adds the most specific detail about the temples, and written mention of yo-yos goes back to the
appearance of a morel mushroom?
fifth century B.C.
A) NO CHANGE
B) earthy, oddly amusing, interesting-looking 51
C) odorless and unusually shaped
A) NO CHANGE
D) sand-colored, cone-shaped, sponge-like
B) yo-yo, but it is hard to know for sure, considering
the yo-yo’s history.
C) yo-yo, though no one is certain why some ancient
yo-yos were made out of terracotta, a fragile clay.
D) yo-yo.

CONTINUE
132 Style and Tone

While many cultures had their variations of the yo-yo, “I can’t play a piano, I can’t play a guitar, all I can do is
the American version can be traced to the Philippines, take a picture,” Withers said in a recent interview. 54
where yo-yos 52 have been a national pastime for At long last, Ernest C. Withers had recorded some five
centuries. million photographic images.

52 54

Which choice provides the most effective evidence Which choice best expresses the fact that Withers was
of the long history of enthusiasm for the yo-yo in the still taking photographs at the time this passage was
Philippines? written?
A) NO CHANGE A) NO CHANGE
B) have been a popular hobby for years. B) At last count,
C) were carved out of fine wood or animal horns. C) To sum up,
D) resembled a toy that was popular in China. D) All in all,

As an African American intimately familiar with the Proud of his Native American roots, he ridiculed the
geography and people of the South, he was often the first snobbery of those who bragged about their illustrious
photographer present as historic moments took place. forebears: 55 “Live your life so that whenever you lose,
Using his hometown as his base, Withers documented you’re ahead.”
the key people and events of 53 the world in which he 55
grew up, observed, and learned.
Which quotation most effectively supports the statement
53 made in the first half of this sentence?
A) NO CHANGE
Which choice provides the most specific information in B) “I don’t make jokes—I just watch the government
support of the claim made by the preceding sentence? and report the facts.”
A) NO CHANGE C) “My ancestors didn’t come over on the
B) the Memphis music scene, baseball’s Negro Mayflower—they met the boat.”
Leagues, and the Civil Rights Movement. D) “I never met a man I didn’t like.”
C) his world, which have become memorable because
of their significance.
D) the places that he thought would be important or
newsworthy.

CONTINUE
Style and Tone 133

Just off the southern coast of Texas, not far from the My grandmother can talk football as fluently and
Mexican border, lies one of my favorite places, Padre confidently as any sportscaster. 58 However, she’ll
Island. 56 However, some of my best childhood proclaim in her Korean accent that a veteran quarterback
memories are of days spent there with my father. should “retire already—too old” and that a “Heisman-

56 winning rookie tailback” will easily gain a thousand


yards this season.
A) NO CHANGE
B) Nevertheless, some 58
C) On the other hand, some
A) NO CHANGE
D) Some
B) On the other hand, she’ll
C) Otherwise, she’ll
D) She’ll
People go to this narrow strip of land in the Gulf of
Mexico to sunbathe, fish, swim, explore the towns, or 57
pass the time engaged in leisurely pursuits. Time Dollar participants gain a strengthened sense of

57 neighborhood and community. Neighbors are getting


to know each other, and 59 teenagers have gotten
Which choice adds the most specific detail to the
description of Padre Island? involved.
A) NO CHANGE
59
B) simply stroll along the more than eighty miles of
beautiful shoreline. Which choice most clearly shows how Time Dollar
C) go up and down the beach, not far at all from the programs are promoting communication between
coast of Texas. community members?
D) spend time on the beach gazing out at the waters of A) NO CHANGE
the Gulf of Mexico. B) senior citizens are getting out more often.
C) teenagers are enjoying new experiences.
D) senior citizens are connecting with teenagers.

CONTINUE
134 Style and Tone

As I weigh the turtle and measure its carapace (shell), Though 61 shy and quiet on land, these animals are
another volunteer, Kiko, 60 watches as I record the graceful swimmers.
data. Tourists always seem delighted by the opportunity 61
to closely observe a turtle.
Which choice most effectively sets up a contrast with the
60 sentence’s later description of the sea turtles’ swimming?
A) NO CHANGE
Which choice most effectively completes the sentence B) sluggish and cumbersome
and provides a transition into the next sentence?
C) mysterious and reclusive
A) NO CHANGE
D) sleek and colorful
B) explains the project to people who have gathered
nearby.
C) stands near me as I work at the examination table.
D) was the one who introduced me to this turtle I follow the turtle as 62 it beats its flippers like wings
project.
and soars through the canyon reefs toward the deep
water beyond.

62

Which choice most effectively completes the sentence by


focusing on the turtle?
A) NO CHANGE
B) I dream of being a marine biologist who spends a
lifetime making the ocean a better place.
C) it swims farther into the ocean; when I finally
surface, my friends are distant figures on the sand.
D) I am soon left behind, and I look forward to
returning to the sunny Hawaiian Saturday.
13 Syntax

You watch as the turtles each slowly dig a hole and fill it, 3 Anderson began by singing the national anthem,
one egg at a time, 1 in which there are over a hundred after singing the national anthem, she sang several opera
eggs. pieces and spirituals.

1 3

A) NO CHANGE A) NO CHANGE
B) there are B) After she sang several opera pieces and spirituals,
C) with Anderson began by singing the national anthem.
D) DELETE the underlined portion. C) Anderson began by singing the national anthem,
and after that, she sang several opera pieces and
spirituals.
D) By singing the national anthem, Anderson began
and then sang several opera pieces and spirituals.
Before her a sea of faces stretched all the way to the
Washington Monument, and behind her 2 towered
the statue of Abraham Lincoln.

A) NO CHANGE
B) Abraham Lincoln towered as a statue.
C) was towering the statue of Abraham Lincoln.
D) the statue of Abraham Lincoln was towered.

135
136 Syntax

Small doses of nostalgia may be 4 harmless but Though this trend might remind us of campus fads of
enough, anything beyond that can get awfully dangerous years 7 past, as such packing Volkswagen Bugs, it
awfully fast. comes as good news.

4 7

A) NO CHANGE A) NO CHANGE
B) enough harmless, but B) passing, like
C) harmless enough, C) passing, as
D) harmless enough, but D) past, like

Japanese students observe 5 a rigorous annually The first bubble gum, Blibber-Blubber, was developed
schedule that begins in the second week of April and by the Fleer Corporation in 1906, but it proved to be 8
extends through the following March. too sticky and brittle to be successfully marketed.

5 8

A) NO CHANGE A) NO CHANGE
B) an annual rigorously B) to
C) an annual rigorous C) much to
D) a rigorous annual D) DELETE the underlined portion.

One popular practice among students in many colleges 9 By pulling carts, oxen were amidst the automobile
has been 6 for participation in a marathon oral traffic.
reading of an important literary work. 9
6 A) NO CHANGE
A) NO CHANGE B) Oxen pulled carts amidst
B) by participating C) Oxen pulled carts, being amidst
C) as participants D) Oxen were amidst while pulling carts in
D) to participate

CONTINUE
Syntax 137

Whenever I tell friends about the ten years I lived in The process of debate will in itself do more to promote
Fairbanks, Alaska, I find myself focusing not on the both moral growth and mutual 13 understanding, then
desolate wilderness beauty nor on the frigid arctic 10 any arbitrary system of restraints on what rock fans hear.
temperatures. I find myself focusing on the drastic 13
differences in daylight hours of summer and winter.
A) NO CHANGE
10 B) understanding then,
C) understanding than
Which choice most effectively combines the two
sentences at the underlined portion? D) understanding; than
A) temperatures, but instead I find myself focusing
B) temperatures; but instead
C) temperatures, but rather
When I heard this advice, I concluded that my friend’s
D) temperatures: but
suggestion was 14 wildly irrational.

14

But the wall 11 your intent to cut an arch through turns A) NO CHANGE
out to be brick, not wood. B) wilder
C) wild
11 D) the wildest
A) NO CHANGE
B) your intending to cut
C) your intent on cutting
There are other occupants of the glassed-in room,
D) you intend to cut
however, for whom I have not developed such
fondness. 15 To speak of the tree frogs.

You tackle the project once more, with enthusiasm, 12 15

but by a moderation of enthusiasm with experience. A) NO CHANGE


B) The tree frogs to speak of.
12
C) I’m speaking of the tree frogs.
A) NO CHANGE D) Speaking of the tree frogs.
B) but experience has moderated with enthusiasm.
C) but with the experience of moderating enthusiasm.
D) but with enthusiasm moderated by experience.

CONTINUE
138 Syntax

16 The exact formulas for how the shells are packed to The quill of this replacement feather is trimmed so that
achieve such effects are zealously guarded by fireworks when the replacement feather is eventually attached to
manufacturers. the quill stub still in the bird’s body, the repaired feather

16 will 19 be equal in length to the original, whole feather.

A) NO CHANGE 19
B) To achieve such effects from how the shells are A) NO CHANGE
packed means the exact formulas are zealously
guarded B) be equally long
C) The exact formulas are zealously guarded for how C) equal the length
the shells are packed to achieve such effects D) equal length
D) Exactly how the shells are packed to have formulas
in it is zealously guarded

Renee starts by cornrowing my natural hair, braiding


it 20 as close to my scalp.
A teenager 17 whose holding a shoebox containing a
20
kitten as tiny as a gingersnap smiles even as a line of girls
in frilly white communion dresses file by. A) NO CHANGE
B) as closely
17
C) close as
A) NO CHANGE D) close
B) thats
C) as
D) who’s
Her parents encouraged her to study 21 diligent, and
gave her piano and voice lessons.

21
On each wing, all flighted birds have ten primary flight
feathers, each one shaped 18 slight different. A) NO CHANGE
B) diligently,
18
C) more diligent,
A) NO CHANGE D) dilettante,
B) slight differently.
C) slightly differently.
D) slightly more different.

CONTINUE
Syntax 139

Many bilingual teenagers in Miami feel 22 more Other tales were 24 exaggeratedly and humorous
comfortably speaking English than Spanish; the reverse accounts of harsh bosses and smarter workers, or of
is often true of their parents. trickster animals such as Brer Rabbit and his cousins.

22 24

A) NO CHANGE A) NO CHANGE
B) most comfortable B) exaggeratedly humorously
C) more comfortable C) exaggerated humorously
D) most comfortably D) exaggerated, humorous

23 In Eatonville, Hurston went to a place where she Gridley collected $4,350 for the Sanitary
knew the oral tradition would be thriving. She sought Commission. 25 Being inspiring and motivating
out the porch of the general store. himself, Gridley traveled to Virginia City, Nevada, to

23 auction the sack of flour to residents there.

Which choice most effectively combines the two 25


sentences?
A) NO CHANGE
A) In Eatonville, on the porch of the general store,
Hurston went to a place where she knew the oral B) Himself being inspired and motivated,
tradition would be thriving. C) By inspiring and motivating,
B) In Eatonville, Hurston went to the porch of the D) Inspired and motivated,
general store as the oral tradition thrived.
C) In Eatonville, Hurston went to a place where she
knew the oral tradition would be thriving, which
was namely why she sought out the porch of the 26 Although immigrants, Tran, Maya, and their
general store.
D) In Eatonville, Hurston went to a place where she four children left much behind, including their family
knew the oral tradition would be thriving: the restaurant.
porch of the general store.
26

A) NO CHANGE
B) Concerning immigration,
C) After immigrating,
D) As immigrants,

CONTINUE
140 Syntax

27 Tran worked as a hospital clerk. Maya was an The third hypothesis asserts that ocean water
assembler in an electronics factory. contains 30 too little oxygen for insects.

27 30

Which choice most effectively combines the underlined A) NO CHANGE


sentences? B) too few
A) Tran worked as a hospital clerk, although Maya was C) lesser
an assembler in an electronics factory.
D) fewer
B) Tran worked as a hospital clerk, and Maya was an
assembler in an electronics factory.
C) Tran, a hospital clerk, and Maya was an assembler
in an electronics factory.
He raised cattle for a living, but he enjoyed most the herd
D) Tran worked, he found a job as a hospital clerk,
whereas Maya was an assembler in an electronics of 31 tall, magnificence horses he kept.
factory.
31

A) NO CHANGE
It was always a special occasion 28 for which I could B) tall, magnificent
C) magnificently, tall
visit my father at work.
D) tall magnificence
28

A) NO CHANGE
B) which 32 The music is mainly instrumental. The bands
C) when generally consist of guitar, bass guitar, saxophones,
D) where
accordion, and drums.

32

Through her 29 research on Halobates and other Which choice most effectively combines the two
sentences?
insects, Dr. Cheng has considered—and then
A) The music is mainly instrumental, yet the bands
dismissed—three hypotheses concerning the scarcity of generally consist of guitar, bass guitar, saxophones,
accordion, and drums.
marine insects.
B) The music, generally consisting of guitar, bass
29 guitar, saxophones, accordion, and drums, is
mainly instrumental bands.
A) NO CHANGE C) The music is mainly instrumental; the bands
B) having researched generally consist of guitar, bass guitar, saxophones,
accordion, and drums.
C) researching
D) The music and bands are mainly instrumental guitar,
D) doing research bass guitar, saxophones, accordion, and drums.

CONTINUE
Syntax 141

33 This is not a deliberate act. I just unconsciously At a depth of sixty-six feet, this decompression takes
assume that all pens belong to me. seventeen hours. When the mission is complete and it

33 is time for the aquanauts to return to land, the pressure


inside the habitat must be 35 gradual and returned to
Which choice most effectively combines the two
sentences? that of sea level.
A) This is not a deliberate act, yet I just unconsciously
assume that all pens belong to me. 35
B) It is not a deliberate act to unconsciously assume
A) NO CHANGE
that all pens belong to me.
B) gradually and
C) This is not a deliberate act; I just unconsciously
assume that all pens belong to me. C) gradually
D) I am not deliberate, I just unconsciously assume D) gradual
that all pens belong to me.

The two principal types of kayaks are the easily


This situation greatly extends the time that scientists can maneuverable white-water kayak and the 36 largest sea
work underwater. For example, at a depth of sixty feet, kayak.
scientists starting from the surface could only work one
36
hour per day. 34 Divers using SeaLab, can work each
A) NO CHANGE
day as long as they desire.
B) very biggest
34 C) more large
D) larger
A) NO CHANGE
B) Using SeaLab, divers can work each day for as long
as they desire.
C) Using SeaLab for as long as they desire, divers can
work each day.
D) Using SeaLab each day, divers can work desired
lengths.

CONTINUE
142 Syntax

37 He had limited access to formal education. 39 I held them up to the light. When I did, I could
Banneker nevertheless demonstrated a keen curiosity scarcely make out traces of faint figures.
and a consuming interest in acquiring knowledge. 39
37 Which choice most effectively combines the underlined
sentences?
Which choice most effectively combines the underlined
sentences? A) I held them up to the light, causing me to scarcely
make out traces of faint figures.
A) Though he had limited access to formal education,
Banneker nevertheless demonstrated a keen B) I held them, scarcely making out traces of faint
curiosity and a consuming interest in acquiring figures, up to the light.
knowledge. C) I, holding them up to the light, and when I did, I
B) Nevertheless, he had limited access to formal could scarcely make out traces of faint figures.
education, so Banneker demonstrated a keen D) Holding them up to the light, I could scarcely make
curiosity and a consuming interest in acquiring out traces of faint figures.
knowledge.
C) He, having limited access to formal education,
Banneker nevertheless demonstrated a keen
curiosity and a consuming interest in acquiring As a child, Claudia Taylor became deeply attached to
knowledge.
the wild beauty of her surroundings. 40 As an adult,
D) Having limited access to formal education, this
demonstrated a keen curiosity and a consuming she devoted herself to protecting habitats. She also
interest in acquiring knowledge.
devoted herself to restoring habitat that she saw rapidly
disappearing.

In lightless vacuums and locales once thought 38 to 40

hot, to cold or to poisonous, to sustain life, there exists a Which choice most effectively combines the underlined
sentences?
wealth of microbial organisms.
A) As an adult, she devoted herself to protecting
38 habitats and restoring those that she saw rapidly
disappearing.
A) NO CHANGE B) As an adult, she devoted herself to protecting
B) too hot, too cold, or too poisonous habitats; also to restoring habitat that she saw
rapidly disappearing.
C) too hot, too cold, or too poisonous,
C) She devoted herself to protecting and restoring
D) to hot, to cold, or to poisonous
habitats as an adult that she saw rapidly
disappearing.
D) She, as an adult, devoted herself to protecting
habitats and to restoring them, rapidly
disappearing.

CONTINUE
Syntax 143

41 The other theory underlying Wright’s designs is 43 A 441-foot-long walkway represents the keel of
that of organic architecture. Organic architecture means a Liberty Ship. The walkway leads to a platform that
that structures should appear to be an extension of their commands a view of San Francisco Bay.
surroundings. 43
41 Which choice most effectively combines the underlined
sentences?
Which choice most effectively combines the underlined
sentences? A) A 441-foot-long walkway represents the keel of a
Liberty Ship, so that walkway leads to a platform
A) The other theory underlying Wright’s designs is that commands a view of San Francisco Bay.
that of organic architecture: structures should
appear to be an extension of their surroundings. B) A 441-foot-long walkway, representing the keel of a
Liberty Ship, leads to a platform that commands a
B) The other theory underlying Wright’s designs view of San Francisco Bay.
is that of organic architecture in that structures
should of their surroundings appear to be C) A 441-foot-long walkway represents the keel of a
extensions. Liberty Ship commanding a view of San Francisco
Bay.
C) Wright’s designs’ other underlying theory is organic
architecture; also, structures should appear to be an D) A Liberty Ship’s 441-foot-long walkway leads to a
extension of their surroundings. platform that commands a view of San Francisco
Bay.
D) The other theory underlying Wright’s designs
is that of organic architecture, he meant that
structures should appear to be an extension of their
surroundings.
The orange and black monarch butterfly is the 44 most
easiest recognized and striking butterfly species in North

42 “Auntie” is a traditional title of respect in the America.

African nation of Ghana. Ghana is Auntie Ocloo’s native 44


land. A) NO CHANGE
42 B) most easy
C) easiest
Which choice most effectively combines the underlined D) most easily
sentences?
A) “Auntie” is a traditional title of respect in the
African nation of Ghana, Ocloo’s native land.
B) “Auntie” is a traditional title of respect in the
African nation of Ghana, which is famous for being
Auntie Ocloo’s native land.
C) “Auntie,” a traditional title of respect in the African
nation of Ghana, is Auntie Ocloo’s native land.
D) Ghana is Auntie Ocloo’s native land, so “Auntie” is a
traditional title of respect there.

CONTINUE
144 Syntax

45 Monarchs that don’t migrate to Mexico live only 47 At the heart of Rogers’s humor was a respect for
four to six weeks. The migrating generations live at least every human being. Nevertheless, there was a sharp edge
eight months. to his wit.

45 47

Which choice most effectively combines the underlined Which choice most effectively combines the underlined
sentences? sentences?
A) Monarchs that don’t migrate to Mexico live, not A) Nevertheless, at the heart of Rogers’s humor was
the migrating generations living at least eight a respect for every human being except that there
months, only four to six weeks. was a sharp edge to his wit.
B) Since monarchs that don’t migrate to Mexico live B) At the heart of Rogers’s humor was a respect for
only four to six weeks, the migrating generations every human being as well as having a sharp edge
live at least eight months. to his wit.
C) Monarchs that don’t migrate to Mexico live only C) At the heart of Rogers’s humor, there was a respect
four to six weeks, but the migrating generations for every human being; however, his wit did have a
live at least eight months. sharp edge to it.
D) Monarchs living only four to six weeks are the D) Because there was a sharp edge to his wit, at the
ones that don’t migrate to Mexico; migrating heart of it, Rogers’s humor had a respect for every
generations, ones that do go to Mexico, live at least human being.
eight months.

48 I was overcome with excitement. I carefully


He honed his ranching skills, becoming a fine roper and
extracted them from the seaweed before dashing down
rider, though he would later spoof the cowboy image in
the beach to show my father.
some of the films 46 in which he starred.
48
46
Which choice most effectively combines the underlined
A) NO CHANGE sentences?
B) that he was a star. A) Overcome with excitement, I carefully extracted
C) starring those ones. them from the seaweed before dashing down the
beach to show my father.
D) and starred them.
B) I, overcome with excitement, carefully extracted it
from the seaweed before dashing down the beach
to show him it.
C) I was overcome with excitement; I extracted them
from the seaweed before strolling down the beach
to show my father what I had found.
D) I carefully extracted them from the seaweed
before dashing down the beach to show my father,
overcome with excitement.

CONTINUE
Syntax 145

Lunar rainbows, or “moonbows,” are little known Time Dollar Exchange programs recognize that each
because they are difficult to see and the conditions person is valuable and has something 51 meaningfully
required to create them rarely occur. The moon must be to contribute.
full and low in the sky; 49 upon which rain must be 51
falling.
A) NO CHANGE
49 B) more meaningfully
C) more meaningful
A) NO CHANGE
D) meaningful
B) at the same time,
C) it’s necessary that
D) notice that

50 Moonbows were reportedly once a common sight at


both the New York and Canadian parts of Niagara Falls.
Bright surrounding lights made them impossible to see,
however.

50

Which choice most effectively combines the underlined


sentences?
A) Now that bright surrounding lights make them
impossible to see, moonbows were reportedly
once a common sight at both the New York and
Canadian parts of Niagara Falls.
B) Moonbows, bright surrounding lights made them
impossible to see, were reportedly once a common
sight at both the New York and Canadian parts of
Niagara Falls.
C) Moonbows were reportedly once a common
sight at both the New York and Canadian parts of
Niagara Falls, but bright surrounding lights have
made them impossible to see.
D) Since moonbows were reportedly once a common
sight at both the New York and Canadian parts of
Niagara Falls, bright surrounding lights have been
installed, making the phenomenon impossible to
see.
14 Punctuation

Mom keeps a crazy conglomeration of wedding 1 Several weeks before the musical engagement, the
gear; rubber boots; a swanky, black formal; blue jeans; organization that owned the hall 2 canceled the
dignified dresses in three pastel colors; sneakers; beach contract because members objected to an African
sandals, and a ski hat. Every item—except the ski hat-- American singing there.
has come in handy at least once. 2
1 A) NO CHANGE
A) NO CHANGE B) canceled, the contract;
B) gear: rubber boots, a swanky black formal, blue C) canceled the contract;
jeans, dignified dresses in three pastel colors, D) canceled, the contract,
sneakers,
C) gear: rubber boots; a swanky black formal; blue
jeans; dignified dresses, in three pastel colors;
sneakers;
D) gear; rubber boots, a swanky black formal, blue
jeans, dignified dresses, in three pastel colors,
sneakers;

146
Punctuation 147

First, she was introduced to Supreme Court 3 Bubble 6 gum is more elastic than its older sibling,
justices members, of the House, and Senate, executive chewing gum.
department heads, and other dignitaries. 6
3 A) NO CHANGE
A) NO CHANGE B) gum, is more elastic than
B) justices members of the House C) gum, is more elastic, than
C) justices, members of the House D) gum is more elastic than,
D) justices, member of the House

Although Walter Diemer, now a retired senior vice 7


As 4 philosopher George Santayana said “Those who president of the Fleer Corporation never patented his
cannot remember the past are condemned to repeat it.” invention, he had hoped to keep his recipe a secret from

4 other gum manufacturers.

A) NO CHANGE 7
B) philosopher, George Santayana said A) NO CHANGE
C) philosopher, George Santayana, said B) president, of the Fleer Corporation.
D) philosopher George Santayana said, C) president of the Fleer Corporation,
D) president, of the Fleer Corporation

Hikers typically find that the climb takes at least three


hours. Taeko 5 Komiyama, however, can reach the For example, the chili used extensively in the Southwest
summit in only forty minutes. is the Anaheim 8 pepper; it’s fairly mild, with a SHU

5 rating of 1,000 to 3,500.

A) NO CHANGE 8
B) Komiyama; however A) NO CHANGE
C) Komiyama however, B) pepper; its
D) Komiyama, however; C) pepper, its
D) pepper, it’s

CONTINUE
148 Punctuation

When I was a 9 child, it was a five-hour, once-a- The number one 12 novel, Amy Tan’s The Kitchen God’s
summer trek, to the nearest major league ballpark. Wife, is boldly circled.

9 12

A) NO CHANGE A) NO CHANGE
B) child, it was a five-hour, once-a-summer trek B) novel, Amy Tan’s The Kitchen God’s Wife is boldly
C) child, it was a five-hour once-a-summer, trek C) novel Amy Tan’s The Kitchen God’s Wife, is boldly,
D) child it was a five-hour once-a-summer trek, D) novel: Amy Tan’s The Kitchen God’s Wife, is boldly

I don’t want us to be forced to the 10 edge, of a bowled About then the building inspector stops 13 by, to
stadium, like the remnants of unwanted cereal. ensure proper drainage, she says, you’ll have to landscape

10 the lawn.

A) NO CHANGE 13
B) edge of a bowled stadium, like, A) NO CHANGE
C) edge of a bowled stadium like, B) by; to
D) edge of a bowled stadium, like C) by, to,
D) by to

One man noticed my 11 interest, and said, to me, “We


will teach you how to do this if you wish. I am sure that One possible solution to the censorship 14 dilemma is,
with a little practice you could learn.” to rate albums as we do movies.

11 14

A) NO CHANGE A) NO CHANGE
B) interest and said B) dilemma, is to rate albums
C) interest; and said C) dilemma is to rate albums
D) interest and said, D) dilemma is, to rate albums,

CONTINUE
Punctuation 149

The children’s favorite stories were those that he told Leon 18 says that one of the most challenging and
while 15 standing at his easel sketching the giants rewarding accomplishments of her life has been to
monsters and other fantastic creatures that inhabited his compose an opera and then conduct it herself.
lively tales. 18
15 A) NO CHANGE
A) NO CHANGE B) says,
B) standing at his easel, sketching the giants, C) says:
monsters, D) says;
C) standing, at his easel sketching the giants, monsters
D) standing at his easel; sketching the giants,
monsters,
Okada’s only surviving writings are No-No 19 Boy, and
some technical brochures for business corporations that

The children were required to do 16 more than sit and he wrote to earn a living.

listen quietly through their “lessons.” 19

16 A) NO CHANGE
B) Boy and, some technical brochures
A) NO CHANGE
C) Boy and some technical brochures,
B) more, than sit and listen quietly,
D) Boy and some technical brochures
C) more than sit and listen quietly,
D) more than sit, and listen quietly

For many centuries, fireworks came in only two 20

Neither of the two poetic attempts was splendid, but at colors, amber and silver, which were emitted by burning

least 17 words, the writer’s clay were forming into a few black powder.

images. 20

17 A) NO CHANGE
B) colors amber and silver,
A) NO CHANGE
C) colors, amber and silver
B) words: the writer’s clay
D) colors, amber, and silver
C) words, the writer’s clay,
D) words the writer’s clay,

CONTINUE
150 Punctuation

A typical shell is generally packed with a mixture of When storyteller Mary Carter 24 Smith practices her
color-emitting chemical pellets called stars. Barium art and everybody listens.
chloride produces 21 green for example, while 24
strontium chloride emits red.
A) NO CHANGE
21 B) Smith, practices her art
C) Smith, practices her art,
A) NO CHANGE
D) Smith practices her art,
B) green, for example while
C) green, for example, while
D) green, for example; while
Joe is seven, living in those two or three years when
children can manage to throw a baseball a few feet but
A musician balancing a cello case, two Buddhist monks when what they’re really interested in are things closer
in saffron robes, and a group of stockbrokers in crisp, 22 at 25 hand, bugs, butterflies, dirt (if they’re in the
charcoal gray suits get on the subway at Wall Street. infield), and grass (if they’re in the outfield).

22 25

A) NO CHANGE A) NO CHANGE
B) charcoal gray suits, B) hand, bugs, butterflies, dirt,
C) charcoal, gray suits C) hand: bugs, butterflies, dirt
D) charcoal gray, suits D) hand: bugs, butterflies, dirt,

This group, known as the Navajo code talkers, took Once they’ve picked it up, they look at it quizzically for a
part in every assault the U.S. Marines conducted in the while and 26 then look up to: see what all the ruckus is
Pacific from 1942 to 1945, 23 transmitting information, about.
on tactics, troop movements, orders, and other vital 26
communications over telephones and radios.
A) NO CHANGE
23 B) then, look up, to see
C) then look up to see,
A) NO CHANGE
D) then look up to see
B) transmitting information on:
C) transmitting information on
D) transmitting: information on

CONTINUE
Punctuation 151

Using highly sensitive hearing, sight, and smell, as well as I do 30 know, however, that I owe much of my success
touch 27 receptors on the sides of their jaws alligators in the sport to Max, my favorite partner for soccer
easily identify prey. practice.

27 30

A) NO CHANGE A) NO CHANGE
B) receptors, on the sides of their jaws B) know: however,
C) receptors on the sides of their jaws; C) know, however
D) receptors on the sides of their jaws, D) know however,

The greatest threat to the alligator is the loss of its habitat, The word vegalah is Yiddish for “little wagon,” and when
as wetlands are 28 drained: to build housing. I was growing up in the Bronx, that’s what we called

28 the 31 hip-high, wire basket on two wheels that my


parents used for all sorts of errands.
A) NO CHANGE
B) drained to build housing. 31
C) drained, to build, housing.
A) NO CHANGE
D) drained, to build housing.
B) hip-high, wire, basket, on two wheels
C) hip-high wire basket on two wheels,
D) hip-high wire, basket on two wheels
I 29 nod, partly, in agreement partly because my head
rocks back and forth as she pulls and tugs while braiding
my hair. During these years, Coretta Scott King became a 32

29 calm, forceful presence, in the Civil Rights Movement.

A) NO CHANGE 32
B) nod, partly in agreement, A) NO CHANGE
C) nod partly, in agreement, B) calm, forceful presence
D) nod, partly in agreement C) calm, forceful, presence
D) calm forceful presence,

CONTINUE
152 Punctuation

Her first job was as a newspaper 33 reporter she moved I often asked my father to teach me to play the
to St. Louis to write for the Globe-Democrat. harmonica, but I quickly grew 36 impatient, with the

33 wheezing, tinny notes that I produced.

A) NO CHANGE 36
B) reporter; she moved A) NO CHANGE
C) reporter, she moved; B) impatient with the wheezing, tinny, notes
D) reporter, she moved C) impatient with the wheezing, tinny notes
D) impatient, with the wheezing tinny notes,

After she retired, Woods 34 wrote Stepping Up to


Power, a book about her political life. It is known that William Shakespeare’s plays

34 immortalized many words and expressions popular in


his day. Have 37 you ever said you were tongue-tied?
A) NO CHANGE
B) wrote: 37
C) wrote,
A) NO CHANGE
D) wrote;
B) you, ever said you were
C) you ever said, you were
D) you ever said you were,
You feel like you are soaring upward, though actually you
have 35 only slowed down.

35 After the contests, people came to her on their own 38


to share: even more stories.
A) NO CHANGE
B) only, slowed down. 38
C) only slowing down.
A) NO CHANGE
D) only slowed, down.
B) to share, even more, stories.
C) to share even more stories.
D) to share, even, more stories.

CONTINUE
Punctuation 153

39 Next, my mother and I, cored the tomatoes. My friend Stacia and I recently took a class in jazz music,

39 so one night we went down to the Village Vanguard to


hear a 42 performance, by saxophonist, Jackie McLean.
A) NO CHANGE
B) Next my mother and I, 42
C) Next my mother, and I
A) NO CHANGE
D) Next, my mother and I
B) performance by saxophonist,
C) performance, by saxophonist
D) performance by saxophonist
For almost two millennia, the 40 yew, an evergreen tree
with a poisonous fruit and a twisted trunk, was scorned
or feared by many people, particularly Europeans. The backup band of a pianist, a bass player, and 43 the

40 famous drummer Billy Higgins, started with a piece


Stacia and I immediately recognized—“Seven Minds.”
A) NO CHANGE
B) yew, an evergreen tree, with a poisonous fruit 43
C) yew, an evergreen tree, with a poisonous fruit,
A) NO CHANGE
D) yew an evergreen tree, with a poisonous fruit,
B) the famous drummer; Billy Higgins,
C) the famous drummer Billy Higgins
D) the famous, drummer Billy Higgins,
Mr. Gridley, the loser, then decorated a sack of flour with
red, white, and blue 41 ribbons: shouldered it; and led
the band through Austin. The print shops where my father worked, whether

41 in a publishing house in Vermont or in a small-town


newspaper office in Idaho, always presented the same
A) NO CHANGE
B) ribbons, shouldered it, vivid experience. The tap dance of Linotype 44 keys the
C) ribbons shouldered it, pungent scents of printing ink and cleaning chemicals,
D) ribbons, shouldered it; and the slapping sound of the rollers dripping with ink.

44

A) NO CHANGE
B) keys;
C) keys,
D) keys—

CONTINUE
154 Punctuation

I’ve thought many times of the instinctive way our bodies She could now 48 hear, the thrum and whir of the bee
respond to the beat of music by wanting to get up and colony as she sat in her living room.
dance, 45 or how, a parent quiets a crying baby by 48
walking back and forth.
A) NO CHANGE
45 B) hear the thrum, and whir
C) hear the thrum and whir;
A) NO CHANGE
D) hear the thrum and whir
B) or how:
C) or; how
D) or how
We don’t tolerate those 49 who exploit or abuse, the
public lands we use for recreation, the water we drink,
I think that at some deep level we humans need 46 and the air we breathe.
steady rhythmic, movement to feel centered and content. 49
46 A) NO CHANGE
A) NOCHANGE B) who exploit, or abuse
B) steady, rhythmic C) who exploit or abuse
C) steady, rhythmic, D) who, exploit or abuse,
D) steady; rhythmic

It should be no surprise that musicians these days are


I am a honeybee enthusiast. 47 My fascination with adding touches of 50 rock country and reggae to waila.
these little creatures, puts me into some interesting 50
situations now and then.
A) NO CHANGE
47 B) rock, country, and reggae to: waila.
C) rock country and reggae, to waila.
A) NO CHANGE
D) rock, country, and reggae to waila.
B) fascination with these little creatures
C) fascination, with these little creatures
D) fascination with, these little creatures

CONTINUE
Punctuation 155

The United States District 51 Court of New Hampshire; Senator Hernandez built a dance studio and hired
agreed to hear Kevin’s case. Europe’s finest teachers to instruct Amalia 54 in:

51 classical and modern ballet.

A) NO CHANGE 54
B) Court, of New Hampshire A) NO CHANGE
C) Court of New Hampshire B) in, classical
D) Court of New Hampshire, C) in classical
D) in classical,

The 52 court noted, however that restrictions may be


justified in some circumstances, such as in the school Though Hernandez died in 2000, her 55 legacy: the
setting. Ballet Folklorico—lives on, sharing Mexico’s cultural

52 heritage with the world.

A) NO CHANGE 55
B) court noted, however, A) NO CHANGE
C) court, noted however, B) legacy,
D) court noted however C) legacy—
D) legacy

Growing 53 up, in Mexico in the 1920s Hernandez


believed she was born to dance. Small piles of uncapped ballpoints 56 gather as if of

53 their own accord, on my night table.

A) NO CHANGE 56
B) up in Mexico in the 1920s, A) NO CHANGE
C) up, in Mexico in the 1920s, B) gather;
D) up in Mexico, in the 1920s, C) gather,
D) gather:

CONTINUE
156 Punctuation

A garbage truck’s best features are hard to 57 beat The 60 waiter, although fluent in English, did not know
the bright yellow paint, the sound of the warning horn how to interpret this particular request.
as it backs up, and, most amazing of all, the way its 60
mechanical arm lifts and dumps the trash cans into its
A) NO CHANGE
open back end. B) waiter, although,
57 C) waiter although,
D) waiter although
A) NO CHANGE
B) beat:
C) beat,
D) beat; The flat, 61 open, Illinois, prairie allowed me to drink
in huge vistas.

61
My mother has often asked 58 me; why I don’t like
A) NO CHANGE
to do the things or eat the foods that I did when I was B) open Illinois prairie:
younger. C) open Illinois prairie,
D) open, Illinois prairie
58

A) NO CHANGE
B) me:
Holm claims to have a prairie eye, feeling uncomfortable
C) me,
D) me in enclosed, mysterious spaces like the 62 woods, he
prefers the “magnitude and delicacy” of the open prairie.

62
While traveling abroad recently, my neighbor 59
A) NO CHANGE
experienced a common but poorly understood B) woods,
communication problem. C) woods; he
D) woods he
59

A) NO CHANGE
B) experienced, a common
C) experienced a common,
D) experienced; a common

CONTINUE
Punctuation 157

Growing within many plots are such 63 oddities, The nex 66 morning using twigs, for kindling she starts
as purple carrots, white tomatoes, striped eggplants, a small blaze in the firebox, located directly below the
potatoes with blue flesh, and other crops a casual main chamber.
gardener would find strange indeed. 66
63 A) NO CHANGE
A) NO CHANGE B) morning, using twigs for kindling,
B) oddities C) morning, using twigs for kindling;
C) oddities: D) morning using twigs, for kindling,
D) oddities;

Soon after I arrived, my 67 aunt said she had a gift for


Many seed savers order endangered plants through me.
a 64 small; but growing number of organizations that 67
specialize in rare seeds.
A) NO CHANGE
64 B) aunt, said
C) aunt said,
A) NO CHANGE
D) aunt said;
B) small but growing
C) small, but growing
D) small but growing,
The two principal types of 68 kayaks are; the easily
maneuverable white-water kayak and the larger sea
Over many weeks, her collection slowly 65 grows clay kayak.
bowls, cups, vases and sculptures fill the studio. 68
65 A) NO CHANGE
A) NO CHANGE B) kayaks are:
B) grows: clay, bowls, cups, vases, and sculptures, C) kayaks are
C) grows, clay bowls; cups; vases; and sculptures; D) kayaks, are
D) grows: clay bowls, cups, vases, and sculptures

CONTINUE
158 Punctuation

Kayaks can float in 69 less than: a foot of water so a Something about that 72 isolation, and darkness
nature watcher can quietly paddle through shallows seemed mysterious.
frequented by shorebirds and other wildlife. 72
69 A) NO CHANGE
A) NO CHANGE B) isolation, and, darkness
B) less than a foot of water, so C) isolation and darkness
C) less than, a foot of water, so D) isolation and darkness,
D) less than a foot of water: so

73 The hot, dry wind, blowing off the desert seemed to


All kayakers can appreciate the nuances of nature as they blast right through me.
travel on water in this 70 simple, but versatile boat. 73
70 A) NO CHANGE
A) NO CHANGE B) hot, dry, wind
B) simple C) hot, dry wind
C) simple— D) hot dry wind,
D) simple;

When I remember that human beings share many of the


Some live in airless 71 seams of burning rock; same 74 hopes, and fears, dreams and disappointments,
miles beneath Earth’s surface, others live around the then I can more easily form new relationships and fortify
hydrothermal vents of deep-sea volcanoes. old ones.

71 74

A) NO CHANGE A) NO CHANGE
B) seams, of burning rock B) hopes, and fears, dreams,
C) seams of burning rock C) hopes and fears, dreams
D) seams, of burning rock, D) hopes and fears dreams

CONTINUE
Punctuation 159

Furthermore, highway maintenance costs and pollution In 2002, Mai Neng Moua created a new literary 77
decrease in areas planted with native flowers and grasses, tradition, with the publication of the first-ever anthology,
which require 75 little, if any, mowing or chemical of Hmong American writers.
treatment. 77
75 A) NO CHANGE .
A) NO CHANGE B) tradition, with the publication of the first-ever
anthology
B) little, if any
C) tradition with the publication of the first-ever
C) little if any, anthology
D) little; if any D) tradition with the publication, of the first-ever
anthology,

In 2002, Minnesota writer and 76 editor, Mai Neng


Moua, created a new literary tradition. Frank Lloyd Wright, one of the twentieth century’s most
creative and innovative architects, based most of his
76
designs on two theories. One is his 78 mentor Louis
A) NO CHANGE
Sullivan’s idea; that “form follows function”— that is, a
B) editor Mai Neng Moua
C) editor, Mai Neng Moua building’s use determines its design.
D) editor Mai Neng Moua,
78

A) NO CHANGE
B) mentor, Louis Sullivan’s idea,
C) mentor, Louis Sullivan’s idea
D) mentor Louis Sullivan’s idea

CONTINUE
160 Punctuation

Dr. Esther Afua 79 Ocloo: first chairwoman of Ornette Coleman had something new to offer. He had
Women’s World Banking, preferred to be called Auntie developed an unorthodox 82 approach over years
Ocloo. of effort, first in his home state of Texas and then in

79 Los Angeles, where he was routinely snubbed by older


musicians such as Dexter Gordon and Miles Davis.
A) NO CHANGE
B) Ocloo; 82
C) Ocloo,
A) NO CHANGE
D) Ocloo−
B) approach: over years of effort, first
C) approach: over years of effort first
D) approach over years, of effort, first
Since 1975, twenty-five million people—three-fourths
of them 80 women; in more than forty less-developed
nations have received microloans. Next came Coleman’s 83 controversial, but successful

80 1959 debut with his quartet at a New York club called the
Five Spot.
A) NO CHANGE
B) women, 83
C) women−
A) NO CHANGE
D) women.
B) controversial; but successful
C) controversial but successful
D) controversial but successful:
For many African Americans, the 81 word, freedom
would serve as a goal, a guiding principle, and a rallying
cry. Free 84 jazz, as Coleman’s style came to be known,

81 provided an unconstrained sound to match the intensity


and conviction of the times.
A) NO CHANGE
B) word, freedom, 84
C) word freedom,
A) NO CHANGE
D) word freedom
B) jazz, as Coleman’s style came to be known
C) jazz as Coleman’s style came to be known,
D) jazz as Coleman’s style came to be known

CONTINUE
Punctuation 161

Among the most effective tools are new tax incentives Such an 88 impact many scientists believe would have
for those who 85 renovate, rather than demolish, old blanketed the planet with debris, blocking out sunlight
barns. and cooling Earth for many years.

85 88

A) NO CHANGE A) NO CHANGE
B) renovate, rather than demolish B) impact, many scientists believe,
C) renovate rather than demolish, C) impact, many scientists believe
D) renovate, rather than, demolish D) impact many scientists believe,

Every winter, 86 thousands of visitors, come to Between 100,000 and 500,000 birds gather here for a six-
southern Florida. week rest during their long journey from 89 Texas and

86 New Mexico to the Arctic Circle.

A) NO CHANGE 89
B) thousands, of visitors A) NO CHANGE
C) thousands of visitors B) Texas, and New Mexico
D) thousands of visitors’ C) Texas and New Mexico,
D) Texas and, New Mexico

In 1980, physicist Luis Alvarez and his 87 son geologist


Walter Alvarez announced a startling discovery. I’m not talking 90 about: imaginary creatures but

87 about deliciously real morel mushrooms—earthy,


oddly amusing, interesting-looking fungi that appear in
A) NO CHANGE
B) son, geologist Walter Alvarez, springtime.
C) son geologist, Walter Alvarez, 90
D) son geologist Walter Alvarez,
A) NO CHANGE
B) about, imaginary creatures but
C) about, imaginary creatures, but
D) about imaginary creatures, but

CONTINUE
162 Punctuation

Many 91 monarchs, west of the Rocky Mountains Cox grew up swimming in the 94 cold lakes of New
migrate to the southern California coast, where they Hampshire and Maine.
overwinter in eucalyptus groves. 94
91 A) NO CHANGE
A) NO CHANGE B) cold, lakes of New Hampshire
B) monarchs west of the Rocky Mountains C) cold lakes, of New Hampshire
C) monarchs west, of the Rocky Mountains D) cold, lakes of New Hampshire,
D) monarchs, west of the Rocky Mountains,

In addition to capturing many public personages on


Most 92 monarchs, millions of them across the United film, Withers also 95 photographed: waitresses, church
States and Canada—migrate as many as three thousand congregations, nightclub audiences, and Little League
miles to Oyamel fir forests near Mexico City. baseball games.

92 95

A) NO CHANGE A) NO CHANGE
B) monarchs— B) photographed waitresses,
C) monarchs; C) photographed: waitresses
D) monarchs D) photographed waitresses

Astronauts aboard the space 93 shuttle Discovery


demonstrated that while a yo-yo would spin in a near-
zero gravity environment, it refused to sleep.

93

A) NO CHANGE
B) shuttle, Discovery;
C) shuttle Discovery,
D) shuttle, Discovery

CONTINUE
Punctuation 163

Gradually, he grew into the role of homespun sage and Hawaii’s green sea 98 turtle, has been classified as an
philosopher, much like that of the kindly old 96 captain endangered species for over 25 years.
he played in his last film, Steamboat Round the Bend. 98
96 A) NO CHANGE
A) NO CHANGE B) turtle has been classified as
B) captain, he played in his last film, C) turtle, has been classified as,
C) captain, he played in his last film D) turtle has been classified: as
D) captain he played in his last film

That was how I learned the names of the beautiful objects


in my 97 collection; Scotch bonnets, Atlantic cockles,
bay scallops, and sand dollars.

97

A) NO CHANGE
B) collection
C) collection,
D) collection:
15 Frequently Confused
Words

Marion did not intentionally 1 loose her passport so The 1815 eruption of Mount Tambora in Indonesia was the
that she could stay in Spain longer. most powerful ever recorded. Many geologists still flock to

1 the 3 cite to explore the enormous and still active crater.

A) NO CHANGE 3
B) loss A) NO CHANGE
C) lose B) sight
D) loosen C) site
D) seen

Surprisingly, bacon is a great 2 compliment to


caramel-flavored ice cream. The students watched as the beekeeper collected honey

2 and calmly allowed the bees to 4 envelop her arm


without being stung.
A) NO CHANGE
B) completion 4
C) complement
A) NO CHANGE
D) comment
B) envelope
C) develop
D) develop

164
Frequently Confused Words 165

Some historians argue that Napoleon Bonaparte had a The American journalist Nellie Bly profoundly 8
strong 5 immortal character and call him one of the effected investigative journalism with her undercover
greatest villains in history. story of a women’s mental institution.

5 8

A) NO CHANGE A) NO CHANGE
B) memorial B) affected
C) amoral C) afflicted
D) moral D) infected

In many cultural traditions, it is considered rude to not Romeo was faced with a life-changing decision: 9
6 accept food as a guest in someone’s home. Guests whether to honor his family or pursue his love, Juliet.
should always eat a small portion of whatever they are 9
served, even if they don’t like it.
A) NO CHAGNE
6 B) wither
C) weather
A) NO CHANGE
D) when
B) incept
C) except
D) recept
Krista didn’t know how to 10 precede when her history
teacher fell asleep during her presentation.
The bubonic plague, also known as the Black Death, 10
spread across Europe in the 1300s and 7 inflicted a
A) NO CHANGE
third of the human population. B) proceed
7 C) proceeds
D) cede
A) NO CHANGE
B) afflicted
C) irritated
D) effected

CONTINUE
166 Frequently Confused Words

The dictator and his family line 11 reined over the Harvey Milk 15 effected great social change by acting
people of Gwynevyr for 300 years. as a community organizer for the LGBT movement.

11 15

A) NO CHANGE A) NO CHANGE
B) reigned B) affected
C) rained C) afflicted
D) waned D) inflicted

This beautiful flower is 12 indigenous to this island. Make sure to 16 site the appropriate sources in your

12 bibliography.

A) NO CHANGE 16
B) indigent A) NO CHAGNE
C) indignant B) sight
D) indigo C) cite
D) sighting

The Little Mermaid removed the flower 13 pedals, one


by one, as she daydreamed about her prince. The great Genghis Khan tried to 17 illicit fear in his

13 enemies.

A) NO CHANGE 17
B) petals A) NO CHANGE
C) pettles B) elicit
D) pediatrics C) elucidate
D) exude

The doctor will 14 proscribe nasal spray to those


suffering from allergies.

14

A) NO CHANGE
B) prescribe
C) scribe
D) predetermine

CONTINUE
Frequently Confused Words 167

In order to ensure an orderly distribution of his late wife’s The partnership proved to be a success, as Jane’s
assets, George needed to 18 appraise the real property. strengths 22 complimented Joe's weaknesses.

18 22

A) NO CHANGE A) NO CHANGE
B) apprise B) complemented
C) apprehend C) contemplated
D) appreciate D) compelled

Bruce was 19 discrete about his true identity as Batman. Please 23 insure that this mishap does not happen

19 again.

A) NO CHANGE 23
B) disagree A) NO CHANGE
C) concrete B) ensure
D) discreet C) sure
D) secure

Uber’s concept of hiring independent contractors to


serve as taxi drivers is an 20 ingenuous idea. During a class discussion on Shakespeare’s King Lear, the

20 English professor cited Christopher Moore's 24 Fool as


a re-telling of the Elizabethan tragedy, rather than one
A) NO CHANGE
B) ingenious grand allusion to it.
C) invariable 24
D) ingratiating
A) NO CHANGE
B) illusion
C) delusion
Trump invested a great deal of his own 21 capitol into D) elution
his early real estate development projects.

21

A) NO CHANGE
B) capital
C) captain
D) capitulation

CONTINUE
168 Frequently Confused Words

It takes courage to 25 except when you're wrong and A sincere enthusiast of 28 oral history, Roy G. Bivens
humbly apologize. seized every opportunity to tell an epic tale.

25 28

A) NO CHANGE A) NO CHANGE
B) accept B) orally
C) expect C) aura
D) intercept D) aural

The felling of primary forest for farmland and timber Any serious fisherman will tell you that the best way
threatens the existence of 26 indignant tribes that live to catch a fish is to remain 29 stationery and quiet,
in the Amazon rain forest. whether you cast your line from a boat or the shore.

26 29

A) NO CHANGE A) NO CHANGE
B) inherited B) stagnant
C) digynous C) sanctuary
D) indigenous D) stationary

The Confederacy failed to 27 succeed from the Union The central location of Jodie’s desk 30 implied her in
and lost the American Civil War. the local high school’s calculus cheating scandal.

27 30

A) NO CHANGE A) NO CHANGE
B) accede B) implicated
C) exceed C) indicated
D) secede D) intimidated

CONTINUE
Frequently Confused Words 169

Molly appreciated the visible spectrum of colors so much When she sailed or played basketball, Tameka observed
that she decided to 31 dice her hair all the colors of and calculated practical physics problems in her head.
the rainbow. She loved the look, but it required a lot of When she had to sit through a classroom 34 lectern on
maintenance. physics, Tameka became disinterested and fell asleep.

31 34

A) NO CHANGE A) NO CHANGE
B) die B) lecture
C) dye C) lector
D) dry D) lecturer

Medieval physicians 32 advised their patients based on Microbes are 35 immanent, and many are much more
the Theory of Humors, which was a diagnostic practice powerful than their size suggests.
that considered several things, including a person’s 35
temperament, their illness with relationship to the four
A) NO CHANGE
“elements” (earth, air, fire, and water), and the “qualities” B) eminent
(hot, cold, wet, and dry) of those elements. C) imminent
D) ubiquitous
32

A) NO CHANGE
B) adviced
The so-called “Abrahamic” religions draw from 36
C) advanced
D) advertised extant texts that have caused controversy for millennia.

36

A) NO CHANGE
I’d like 33 too go with you two, too. B) exiled
33 C) extent
D) exigent
A) NO CHANGE
B) to
C) two
D) tutu

CONTINUE
170 Frequently Confused Words

When our server at Le Toro Negro talked down to us Because Pinocchio was a marionette that lacked many
because we couldn't pronounce the Spanish words human characteristics, his friend Jiminy Cricket served
correctly, we refused to tip him on 37 principal. as his official 40 conscious.

37 40

A) NO CHANGE A) NO CHANGE
B) principate B) conscient.
C) principality C) cornichon.
D) principle D) conscience.

As a couple, they were 38 complementary— she was South Carolina 41 succeeded from the Union on
tall where he was short and he was funny where she was December 20, 1860; five more states followed suit in
serious. She could talk for hours at length, while he rarely January of 1861.
spoke in public. 41
38 A) NO CHANGE
A) NO CHANGE B) acceded
B) alimentary C) exceeded
C) complimentary D) seceded
D) supplementary

In The 
Epic of Gilgamesh, the title character searches for
When he was cornered by a tribe of monkeys in the a plant, which will make him 42 amoral, a symbolic
jungle, João was able to 39 allude them by cleverly quest shared by many literary heroes who attempted to
throwing his voice. escape death.

39 42

A) NO CHANGE A) NO CHANGE
B) elude B) memorial
C) exclude C) immortal
D) preclude D) moral

CONTINUE
Frequently Confused Words 171

In order to ensure an orderly distribution of his late According to Theodore Schultz, education is a major
wife’s assets, George needed to have the property 43 form of human 46 capitol: a nation which invests in
appraised. scholarship will generate greater economic growth than

43 would otherwise be possible.

A) NO CHANGE 46
B) apprised. A) NO CHANGE
C) apprehended. B) capital
D) appreciated. C) captain
D) capitulation

The modern Abrahamic religions draw from 44


extant texts that have caused controversy for millennia. When he encountered one section of Mars’ 47

44 indignant population, the Red Martians, John Carter was


impressed by their advanced technology and wealthy
A) NO CHANGE
B) exiled culture.
C) extent 47
D) exigent
A) NO CHANGE
B) indicative
C) indigent
When she sailed or played basketball, Natalya observed D) indigenous
and calculated practical physics problems in her head.
When she had to sit through a classroom 45 lectern on
physics, she became disinterested and fell asleep. When Ignatius saw the very large “DO NOT ENTER”

45 sign hanging from the door of the abandoned house,


he had an original and 48 ingenuous idea: he turned
A) NO CHANGE
B) letter around, walked away, and everyone lived happily ever
C) lector after.
D) lecture
48

A) NO CHANGE
B) ingenious
C) ingenues
D) igneous

CONTINUE
172 Frequently Confused Words

Despite her 49 eminence as the first author to earn over The potential investor shook her head and turned back
$1 billion, J.K. Rowlng had humble beginnings; she was to the room one last time. “I just can’t put down that kind
living on public assistance when she wrote the first Harry of 52 capitol without a guarantee of a return,” she said,
Potter book. and closed the door.

49 52

A) NO CHANGE A) NO CHANGE
B) imminence B) capitulation
C) innocence C) capital
D) immanence D) caption

Despite the company’s hints that the phone launch In Victorian England, where many topics of conversation
was 50 immanent, the eager consumers had to wait were 53 prescribed, people communicated secret
months for the bugs in the new operating system to be messages through floriography, the language of flowers.
fixed. 53
50 A) NO CHANGE
A) NO CHANGE B) proscribed,
B) eminent C) inscribed,
C) imminent D) described,
D) innominate

The warm classroom, combined with the monotone


With a desire to track the illegal elephant ivory trade, the voice that the professor 54 afflicted upon his students,
journalist devised an 51 ingenuous solution. With the put them all to sleep.
aid of his team and a skilled taxidermist, he developed 54
fake elephant tusks and equipped them with GPS
A) NO CHANGE
devices. B) inflected
51 C) inflicted
D) affected
A) NO CHANGE
B) ingenious
C) igneous
D) ingénue

CONTINUE
Frequently Confused Words 173

After he made the mistake of using the wrong verb form, Most people don’t eat the 58 corps of an apple because
Jake’s French teacher 55 apprised him of the proper it has less flavor and a lot of seeds.
usage. 58
55 A) NO CHANGE
A) NO CHANGE B) chore
B) excised C) core
C) apprized D) court
D) appraised

Julie was quite convinced that her younger brother


With the creation of Google’s new parent company, Jacob 59 had pored too much syrup on his pancakes.
Alphabet, the organizations will go through some 59
major 56 personal changes; many people will want to
A) NO CHANGE
follow the cutting-edge innovation projects to the new B) had poured
company. C) have pored
D) have poured
56

A) NO CHANGE
B) personnel
Robert believes that everyone in the world should
C) personality
D) personally be fascinated by his research project, but most non-
scientists find it a bit of a 60 boar.

60
In 1987, the Anchorage School Board banned the
A) NO CHANGE
dictionary from libraries for containing 57 illicit B) bore
material; it was argued that students would use this C) bohr
resource for looking up unsavory words. D) boer

57

A) NO CHANGE
B) tacit
C) elicit
D) implicit

CONTINUE
174 Frequently Confused Words

The tyrant and his descendants 61 reined over the Dr. Solo looked at his empty waiting room and sighed.
people of St. Guinevere for 300 years. Though he’d moved to the area months ago, he had still

61 not acquired many 64 patience.

A) NO CHANGE 64
B) reigned A) NO CHANGE
C) rained B) patients.
D) waned C) patents.
D) patinas.

The students watched as the beekeeper collected honey


and calmly allowed the bees to 62 envelop her arm After her professor’s explanation about using MLA style
without being stung. in academic papers, the student raised her hand. “But

62 how do I 65 sight a tweet?” she asked.

A) NO CHANGE 65
B) envelope A) NO CHANGE
C) develop B) site
D) develope C) side
D) cite

Overtired and under stress from his holiday orders,


the chocolatier could not decide whether to 63 The 66 whether is rapidly changing in our northwest
envelope the pretzels in dark chocolate or almond bark. broadcast area; just today, we’ve seen snow, rain, and

63 sunshine.

A) NO CHANGE 66
B) redevelop A) NO CHANGE
C) develop B) wealthier
D) envelop C) weather
D) wether

CONTINUE
Frequently Confused Words 175

In many cultures, it is considered rude to not 67 Most of his team members groaned when they saw
accept food as a guest in someone’s home. Guests should the huge “surprise” obstacle at the end of the race, but
always eat a small portion of whatever they are served, Masudi took it as an opportunity to test his 70 mettle.
even if they don’t like it. 70
67 A) NO CHANGE
A) NO CHANGE B) medal.
B) incept C) metal.
C) except D) meddle.
D) receipt

In 2005, a shepherd in Turkey lost most of his flock when


Dr. Martin Luther King Jr. 68 effected great social a single sheep 71 wondered off a cliff and the 1,499
change through his leadership in nonviolent protests for others followed it.
civil rights. 71
68 A) NO CHANGE
A) NO CHANGE B) wanders
B) affected C) wandered
C) afflicted D) wonders
D) inflicted

The man I spoke to had an 72 accent so thick that I


The Bubonic Plague, also known as the Black Death, could barely understand him.
spread across Europe in the 1300s and 69 inflicted a 72
third of the human population.
A) NO CHANGE
69 B) assent
C) ascent
A) NO CHANGE
D) essence
B) afflicted
C) irritated
D) effected

CONTINUE
176 Frequently Confused Words

Although the table appeared to be carved out of a single Critics of modern cinema wonder if filmmakers have
block of wood, it was actually composed of six 73 foregone basic elements like story and character in favor
discrete parts. of flashy, yet increasingly familiar, special 75 affects.

73 75

A) NO CHANGE A) NO CHANGE
B) secrete B) defects.
C) discreet C) impacts.
D) replete D) effects.

I don’t care if Bill has every 1980s movie memorized—I The jury was stuck. The defendant’s actions existed
can’t stand him or his constant 74 illusions to pop somewhere in the gray area between right and
culture. wrong— 76 an immoral space.

74 76

A) NO CHANGE A) NO CHANGE
B) allusions B) an amoral
C) alluvia C) a memorial
D) collusions D) a balmoral

CONTINUE
Frequently Confused Words 177

Rania successfully completed her first novel during When he finally quit his terrible job, Justin wanted
National Novel Writing Month 77 thorough diligently to 79 ensure that he wouldn’t be able to go back. So
writing 2,000 words per day. he swapped his employer’s prized autographed photo of

77 Bruce Springsteen with one of Weird Al Yankovic.

A) NO CHANGE 79
B) threw A) NO CHANGE
C) through B) reassure
D) though C) assure
D) posture

If Impressionist paintings truly depict daily life in


1800s France, then people spent a good deal of time 78
laying in parks or by the riverside.

78

A) NO CHANGE
B) allaying
C) lying
D) allying

CONTINUE
16
More Exercises
on Conventional
Expressions

1 3

Over the past (A) two years, apparel manufacturers have The museum is submitting (A) proposals to several (B)
(B) worked to meeting (C) the revised federal standards foundations in (C) the hope to gain (D) funds to build a
for the design (D) of uniforms. No error (E) tropical butterfly conservatory. No error (E)

2 4

According to last week’s survey, most voters were Most of (A) the hypotheses that (B) Kepler developed
disappointed by (A) legislators’ (B) inability working (C) to explain physical forces were later rejected as (C)
together on (D) key issues. No error (E) inconsistent to (D) Newtonian theory. No error (E)

178
More Exercises on Conventional Expressions 179

5 8

The iris, the colored part of the eye, contains (A) delicate Not very particular in (A) nesting sites (B), house
patterns that are (B) unique to (C) each person, offering wrens may nest (C) in birdhouses, mailboxes, building
a powerful means of (D) identification. No error (E) crevices—even in the pockets of (D) hanging laundry.
No error (E)

6
9
It was (A) a Chinese American grower who finally
succeeded with adapting (B) the now familiar (C) orange Contrary to what (A) many people believe, heat lightning
tree to (D) the American climate. No error (E) is not lightning (B) caused by heat; it is ordinary
lightning that occurs at too great a distance (C) for its
accompanying thunder to be audible (D). No error (E)

Plants grown for (A) their flowers should be treated with


10
(B) a low-nitrogen fertilizer because excessive nitrogen
promotes (C) the growth of leaves at the expense of (D) Mediators were standing by, prepared to intervene in (A)
flowers. No error (E) the labor dispute even though (B) both sides had refused
(C) earlier offers for (D) assistance. No error (E)

CONTINUE
180 More Exercises on Conventional Expressions

11 14

According to some theorists, what (A) any (B) particular No one but (A) a fool would readily (B) lend money to a
bird can eat could change with even (C) the slightest (D) person who is known (C) to be (D) a frequent gambler.
variation in the shape of its beak. No error (E) No error (E)

12 15

At a time when (A) interest in twentieth-century classical At the highest level (A) of competitive chess, players are
music seems (B) on the verge to disappear (C), the so familiar to (B) one another’s strategies that they can
avantgarde compositions of the 1960s and 1970s manage practically (C) read their opponents’ minds; the result is
(D) to retain their popularity. No error (E) drearily predictable (D) play. No error (E)

13 16

A powerful advocate to (A) equal rights, Belva Lockwood Like many (A) people, Luanne believes that parents can
was (B) twice a candidate for President long before the foster musical ability in small children out of (B) playing
Nineteenth Amendment to the Constitution (C) allowed (C) classical music for them while they are (D) infants.
women to (D) vote. No error (E) No error (E)

CONTINUE
More Exercises on Conventional Expressions 181

17 20

The X chromosome was named (A) “X” because Centuries ago (A), the Greek physician Hippocrates
researchers were baffled by its behavior, not because (B) advocated collecting (B) data from (C) patients in order
of its (C) resemblance to (D) the letter X. No error (E) to be drawing (D) conclusions and diagnose diseases. No
error (E)

18
21
Vanessa had a tendency of changing (A) her mind often,
so often (B) in fact that her friends gave up expecting (C) In 1991 Burmese military officials arrested dissident
her to show up at (D) their parties. No error (E) political leader Aung San Suu Kyi, thereby (A) preventing
(B) her from leaving (C) Burma for accepting (D) the
Nobel Peace Prize in Oslo. No error (E)

19

A recent report suggests that (A) safety-conscious drivers


22
need not (B) limit themselves to driving (C) large, heavy
vehicles, because the structural integrity of a vehicle is For (A) more than a century, wooden ships called
more important (D) than its size and weight. No error “skipjacks” were regularly used (B) to harvest (C) oysters
(E) in Chesapeake Bay, but only a few such vessels (D)
remain in service today. No error (E)

CONTINUE
182 More Exercises on Conventional Expressions

23 26

Built from 1609 to 1617 to rival (A) the Hagia Sophia, Arguing that (A) every chemical element is composed
Istanbul’s Sultanahmet Mosque was celebrated (B) not so of (B) fundamental particles of a weight unique to (C)
much for its architecture but because of (C) the exquisite that element, the English meteorologist John Dalton
blue tiles that adorned its (D) interior. No error (E) presented (D) the first modern atomic theory in 1803.
No error (E)

24
27
Today in class we discussed the common, but mistaken
(A), assumption that when (B) two people are no longer One way in which orchestra bells differ with (A) the
(C) dating they are incapable to be (D) friends. No error xylophone is that the bells produce (B) more resonant
(E) (C) tones than (D) the xylophone does. No error (E)

25 28

The intricate pattern on (A) a butterfly’s wing is Recently (A) adopted as (B) an emblem of humanitarian
composed of (B) thousands of microscopic scales, each aid, the Red Crystal is, unlike its counterparts (C) the
of which (C) is (D) the product of a single cell. No error Red Cross and the Red Crescent, entirely devoid of (D)
(E) religious connotation. No error (E)

CONTINUE
More Exercises on Conventional Expressions 183

29 32

Prior to (A) the election of Henry Cisneros as mayor of From (A) his smile it (B) was evident that Burton had
(B) San Antonio in 1981, no major city in the United successfully (C) reached an agreement to be averting (D)
States had had (C) a mayor from Mexican descent (D). a strike. No error (E)
No error (E)

33
30
The flavor of (A) a Meyer lemon, a fruit native to (B)
What was (A) most apparent at the food science China, is sweeter and less acidic than (C) that of a
convention was (B) the industry’s continuing common lemon, and a Meyer lemon’s skin is edible (D).
emphasizing on (C) health-promoting ingredients (D). No error (E)
No error (E)

34
31
Many environmentalists think that it is (A) just as
Interest to revive (A) the ancient Olympic Games as (B) important to use (B) existing energy sources efficiently
an international event grew when (C) the ruins of the than (C) it is to develop pollution-free means of
ancient city of Olympia were uncovered (D) by German generating (D) energy. No error (E)
archaeologists in the mid-nineteenth century. No error
(E)

CONTINUE
184 More Exercises on Conventional Expressions

35 38

In recognition to (A) her extraordinary (B) contributions Scientists have discovered that walnut trees respond
to gospel music, in 1988 the National Endowment for to (A) environmental stresses such as (B) drought by
the Arts presented (C) Willie Mae Ford Smith with (D) a producing (C) significant amounts of (D) a substance
National Heritage Award. No error (E) similar to aspirin. No error (E)

36 39

New Zealand had fostered (A) a thriving local music Teeming with (A) hundreds of species of tropical fish,
scene for decades (B), but it was not until the late 1980s the clear (B) waters of the Caribbean Sea provide (C) a
when (C) New Zealand musicians began to reach (D) paradise for (D) scuba divers. No error (E)
international audiences. No error (E)

40
37
Commercial growers tend to (A) breed tomato plants for
Dr. Ling strongly (A) encouraged each of (B) us qualities that increase (B) the profitability of a crop, such
contacting (C) our federal and state representatives and as resistance of (C) disease, rather than (D) for flavor and
urge them to increase (D) funding for elementary school nutritional value. No error (E)
music programs. No error (E)

CONTINUE
More Exercises on Conventional Expressions 185

41 44

Most climatologists agree that (A) global warming is The most engaging feature of (A) the election was (B)
caused from (B) greenhouse gases that are continually undoubtedly (C) the candidates’ debate with (D) foreign
(C) being released into (D) Earth’s atmosphere. No error policy issues. No error (E)
(E)

45
42
The panda, whose (A) Chinese name means “bear cat,”
Speed-reading was once promoted mainly (A) to people resembles (B) the cat in (C) its ability of contorting (D)
who were eager (B) to catch up on recreational reading, its body into various shapes. No error (E)
but now the method appeals to (C) busy executives
coping about (D) information overload. No error (E)

46

Despite her desire to emphasizing (A) the soundness (B)


43
of the car, the seller eventually disclosed the most recent
It was not until 1982, the year two major supermarket (C) repairs to it (D). No error (E)
chains began (A) replacing paper bags with (B) plastic
ones, when (C) the use of plastic shopping bags became
(D) widespread. No error (E)

CONTINUE
186 More Exercises on Conventional Expressions

47 50

As a favored member of the Elizabethan court, Sir Walter Contrary to (A) popular opinion, the readership of
Raleigh assumed an independence (A) that eventually subscription magazines in print is (B) actually increasing
(B) became intolerable to (C) the queen, who had long (C) despite the phenomenal popularity of the Internet as
been (D) his friend and benefactor. No error (E) a source (D) of information and entertainment. No error
(E)

48
51
Through generally insecure and excessively (A)
deferential to the opinions (B) of others, President Chess players find that playing against a computer is
Buchanan would sometimes adhere stubbornly on (C) an helpful to improve (A) their (B) skills, even though
ill-advised policy, even in the face of (D) vigorous public (C) no chess-playing computer has yet won (D) a
criticism. No error (E) championship. No error (E)

49

For thousands of (A) years the inhabitants of Australia


have been (B) digging underground water reservoirs in
order for surviving (C) on one of the world’s driest (D)
continents. No error (E)

CONTINUE
More Exercises on Conventional Expressions 187

52 54

Confucianism is more a code of ethics than like (A) a Even when (A) Barbara Jordan put questions toward (B)
religion; it presents no deities but fosters (B) instead a political nominee, her elegant diction evoked (C) in
(C) a respect for one’s ancestors and for an orderly (D) listeners memories of (D) her eloquent political speeches.
society. No error (E) No error (E)

53 55

When the village elders present (A) recommendations, No one who travels on (A) the expressways that circle
there is (B) hardly ever (C) any opposition against (D) our cities can help but noticing (B) the growing (C)
their proposals. No error (E) number of junkyards that defile (D) the beauty of the
countryside. No error (E)

CONTINUE
188 More Exercises on Conventional Expressions

Part 2

1) Do you believe _________________ ghosts?


2) Anthony is engaged ________________ my cousin.
3) Ms. Ballas substituted ________________our regular teacher.
4) I can’t distinguish one twin _____________ the other.
5) Did you forgive him _______________ lying to you?
6) Children rely ________________their parents for food and shelter.
7) Tim wore sunglasses to protect his eyes ____________ the sun.
8) Chris excels _______________ sports.
9) Andrea contributed her ideas ________________ the discussion.
10) I hope you succeed _______________ your new job.
11) I’m very fond ____________________ their children.
12) The firefighters rescued many people _____________ the burning building.
13) I don’t care ___________________ spaghetti. I’d rather eat something else.
14) Charles doesn’t seem to care ______________ his bad grades.
15) Sometimes Bobby seems to be jealous ___________ his brother.
16) Max is known ___________________ his honesty.
17) Mr. and Mrs. Jones have always been faithful _____________ each other.
18) Do you promise to come? I’m counting _____________ you to be here.
19) Trucks are prohibited _________________ using residential streets.
20) The little girl is afraid ______________ an imaginary bear that lives in her closet.
21) Do you take good care _____________ your health?
22) I’m worried _____________ this problem.
23) I don’t agree _____________ you.
24) We decided _______________ eight o’clock as the time we should meet.
25) Who did you vote _____________ in the last election?
26) How many students were absent ___________ class yesterday?
27) It is important to be polite _____________ other people.
28) The farmers are hoping _____________ rain.
29) Jason was late because he wasn’t aware ___________ the time.
30) We will fight _______________ our rights.
31) I am not familiar ______________ that author’s works.

CONTINUE
More Exercises on Conventional Expressions 189

32) He doesn’t approve ____________ smoking.


33) I subscribe _____________ several magazines.
34) Water consists _____________ oxygen and hydrogen.
35) I became uncomfortable because she was staring _____________ me.
36) She hid the candy _____________ the children.
37) He never argues _____________ his wife.
38) I arrived _____________ this country two weeks ago.
39) We arrived _____________ the airport ten minutes late.
40) Has Mary recovered _____________ her illness?
41) I pray _____________ peace.
42) I am envious _____________ people who can speak three or four languages fluently.
43) Why are you angry _____________ me? Did I do something wrong?
44) They are very patient _____________ their children.
45) The students responded _____________ the questions.
46) I am grateful _____________ you _____________ your assistance.
47) The criminal escaped _____________ prison.
48) Elizabeth is not content _____________ the progress she is making.
49) Paul’s comments were not relevant _____________ the topic under discussion.
50) Have you decided _____________ a date for your wedding yet?
51) My boots are made _____________ leather.
52) I’m depending _____________ you to finish this work for me.
53) Patricia applied _____________ admission _____________ the university.
54) Daniel dreamed _____________ some of his childhood friends last night.
55) Mr. Miller dreams _____________ owning his own business someday.
56) The accused woman was innocent _____________ the crime with which she was charged.
57) Our high school soccer team was very excited _____________ going to the national finals.
58) I’m not acquainted _____________ that man. Do you know him?
59) Mark Twain is known _____________ his stories about life on the Mississippi.
60) A person who is addicted _____________ drugs needs professional medical help.
61) Jack is married _____________ Joan.
62) Could I please have the dictionary when you are finished _____________ it?
63) A: Aren’t you ready yet? We have to be at the ferry dock _____________ 7:45.
64) B: I’ll never make it. I’m still dressed _____________ my pajamas.
65) My car is equipped _____________ air conditioning and a sun roof.

CONTINUE
190 More Exercises on Conventional Expressions

66) The school children were exposed _____________ the measles by a student who had them.
67) Gandhi was committed _____________ nonviolence. He believed in it all of his life.
68) The large table was covered _____________ every kind of food you could imagine.
69) Barbara turned off the TV because she was tired _____________ listening to the news.
70) The choices in that restaurant are limited _____________ pizza and sandwiches.
71) A. Are you in favor of a worldwide ban on nuclear weapons, or are you opposed _____________ it?
72) B: I’m in favor of it. I’m terrified _____________ the possibility of a nuclear war starting by accident. However, my
wife is against disarmament.
73) The department store was filled _____________ toys for the holiday sale.
74) John’s bald head is protected _____________ the hot sun when he’s wearing his hat.
75) The store was crowded _____________ last-minute shoppers on the eve of the holiday.
76) I think you’re involved _____________ too many activities. You don’t have enough time to spend with your family.
77) Your leg bone is connected _____________ your hip bone.
78) Their apartment is always messy. It’s cluttered _____________ newspapers, books, clothes, and dirty dishes.
79) Don’t leave those seedlings outside tonight. If they’re exposed _____________ temperatures below freezing, they will
die.
80) An interior decorator makes certain that the color of the walls is coordinated _____________ the color of the
carpets and window coverings.
81) We finished packing our sleeping bags, tent, first-aid kit, food, and warm clothes. We are finally prepared
_____________ our camping trip.
82) I was very disappointed _____________ that movie. The whole first hour was devoted _____________ historical
background, with a cast of thousands fighting endless battles. I was bored _____________ it before the plot took
shape.
83) A: Are you still associated _____________ the International Red Cross and Red Crescent?
84) B: I was, until this year. Are you interested _____________ working with them?
85) A: I think I’d like to. They are dedicated _____________ helping people in times of crisis and I admire the work they
have done. Can you get me some information?
86) Alice isn’t interested _____________ (look) _____________ for a new job.
87) Henry is excited _____________ (leave) _____________ for India.
88) You are capable _____________ (do) _____________ better work.
89) I have no excuse _____________ (be) _____________ late.
90) I’m accustomed _____________ (have) _____________ a big breakfast.
91) The rain prevented us _____________ (complete) _____________ the work.
92) Fred is always complaining _____________ (have) _____________ a headache.
93) Instead _____________ (study) _____________, Margaret went to a ballgame with some of her friends.
94) Thank you _____________ (help) _____________ me carry my suitcases.

CONTINUE
More Exercises on Conventional Expressions 191

95) Mrs. Grant insisted _____________ (know) _____________the whole truth.


96) I believe _____________ (be) _____________ honest at all times.
97) You should take advantage _____________ (live) _____________ here.
98) Fatima had a good reason _____________ (go, not) _____________ to class yesterday.
99) Everyone in the neighborhood participated __________ (search) __________ for the lost child.
100) I apologized to Yoko _____________ (make) _____________ her wait for me.
101) The weather is terrible tonight. I don’t blame you _____________ (want, not) _____________ to go to the meeting.
102) Who is responsible _____________ (wash) _____________ and (dry) _____________ the dishes after dinner?
103) In addition _____________ (go) _____________ to school full time, Spiro has a part-time job.
104) I stopped the child _____________ (run) _____________ into the street.
105) Where should we go for dinner tonight? Would you object _____________ (go) _____________ to an Italian
restaurant?
106) The mayor made another public statement for the purpose _____________ (clarify) _____________ the new tax
proposal.
107) The thief was accused _____________ (steal) _____________ a woman’s purse.
108) The jury found Mr. Adams guilty _____________ (take) _____________ money from the company he worked for
and (keep) _____________ it for himself.
109) Larry isn’t used _____________ (wear) _____________ a suit and tie every day.
110) I’m going to visit my family during the school vacation. I’m looking forward _____________ (eat) _____________
my mother’s cooking and (sleep) _____________ in my own bed.

CONTINUE
192 More Exercises on Conventional Expressions

Part 3

Even though Marine had found evidence that Herman The special combo at the Japanese restaurant is
Newton—not Isaac Newton— discovered universal comprised 4 with yellowtail, salmon, and snapper.
gravitation, her professors argued that her theory didn’t 4
hold 1 on.
A) NO CHANGE
1 B) from
C) of
A) NO CHANGE
D) in
B) an argument
C) water
D) a candle
Do you have any insight 5 in the reason why the
mangrove finch’s habitat has decreased to a size of just
Although the black diamond ski run was clearly marked, twelve city blocks?
Simon skied 2 out of bounds in order to beat his 5
opponent to the bottom of the mountain.
A) NO CHANGE
2 B) on
A) NO CHANGE C) into
B) boundless D) upon
C) out on bounds
D) outside of bounds

Jane disapproves 6 at her friends’ failure to remove


their shoes before entering her house.
3 For all intensive purposes, Washington High School
6
has ceased to be an academic institution; it has been
converted into an independent theater. A) NO CHANGE
B) in
3 C) of
D) DELETE the underlined portion.
A) NO CHANGE
B) For all intents and purposes,
C) For all intense purposes,
D) For extensive purposes,

CONTINUE
More Exercises on Conventional Expressions 193

Joe became indifferent 7 of his babysitter after she Do not mull 10 on your losses. Focus on the wins!
wrongly accused him of stealing cookies from the cookie 10
jar.
A) NO CHANGE
7 B) at
C) over
A) NO CHANGE
D) onto
B) at
C) towards
D) to
“You can be a real pain 11 about the neck,” said the
man, cleaning up his dog’s mess.
While the imp was preoccupied 8 on devouring its 11
breakfast, Link tiptoed behind the imp and slayed it with
A) NO CHANGE
his sword. B) of
8 C) by
D) in
A) NO CHANGE
B) in
C) at
D) with A Japanese electronics company has invented a new type
of rubber that generates electricity 12 in stretching.

12
People often do not have control 9 in the
A) NO CHANGE
circumstances in their lives. B) when stretched.
9 C) over stretching.
D) under stretch.
A) NO CHANGE
B) over
C) on
D) at

CONTINUE
194 More Exercises on Conventional Expressions

The environmentalist spoke to the audience about the When the third customer came in with an expired
advantages of lab-grown meat, but her words appeared to coupon, the waitress flew 16 on the handle.
go 13 on one ear and out the other. 16
13 A) NO CHANGE
A) NO CHANGE B) from
B) over C) off
C) under D) over
D) in

The tortoise won the race against the hare, fair 17 but
Identifying the right ratchet in the messy toolbox was square.
like finding a needle 14 with a haystack. 17
14 A) NO CHANGE
A) NO CHANGE B) if
B) in C) with
C) as D) and
D) through

After seeing the judges’ scores, the Olympian felt that she
She forgot to clock 15 when at work, so she had to had the competition in 18 the bag.
manually enter her hours later. 18
15 A) NO CHANGE
A) NO CHANGE B) some
B) after C) a
C) in D) another
D) until

CONTINUE
More Exercises on Conventional Expressions 195

After the politician was caught lying about his finances, Instead of giving a dramatic speech, Elizabeth chose to
he continued to lie instead of 19 coming clean. simplify her message and let the facts speak 22 with

19 themselves.

A) NO CHANGE 22
B) going A) NO CHANGE
C) remaining B) before
D) leaving C) for
D) DELETE the underlined portion.

Decked 20 over in her enormous signature eyeglasses,


the elderly fashion designer glared up at her assistant. After hearing the good news about his final exam grade,

20 Matt jumped 23 at joy.

A) NO CHANGE 23
B) in A) NO CHANGE
C) by B) after
D) out C) from
D) for

Caught eating cookies after his bedtime, the little boy


looked 21 at a deer caught in the headlights. Katherine scraped the bottom 24 on the barrel to fund

21 her trip to Nepal, using every cent in her bank account.

A) NO CHANGE 24
B) like A) NO CHANGE
C) for B) of the
D) before C) from the
D) DELETE the underlined portion.

CONTINUE
196 More Exercises on Conventional Expressions

Because of his great memory, Fritz never had to write As part of the royal family, Princess Diana made some
down appointments. Yet when his college entrance controversial decisions. One rare move was her choice
interview slipped his mind, he invested 25 in an to divorce Prince Charles instead of keeping 28 up
appointment book. appearances.

25 28

A) NO CHANGE A) NO CHANGE
B) at B) on
C) on C) the
D) himself to D) at

Twelve publishers rejected the original manuscript of When Fabrizio broke the head off of the famous David
Harry Potter and the Sorcerer’s Stone, 26 missing a boat statue and glued it back on, he had to bribe his loud-
on a multi-million dollar deal. mouthed colleague to keep a lid 29 on it.

26 29

A) NO CHANGE A) NO CHANGE
B) sinking the B) around
C) missing the C) in
D) taking the D) over

Instead of lectures and simulations, U.S. Navy Seals While Yolanda napped in the park, her poodle was stolen
training is mostly trial 27 in fire. from 30 over her nose.

27 30

A) NO CHANGE A) NO CHANGE
B) on B) around
C) by C) under
D) with D) in front of

CONTINUE
More Exercises on Conventional Expressions 197

Even though Finnish language course was 31 over my Despite the Tasmanian Devil’s small size, its
head, I earned the minimum grade required to go on the cantankerous attitude and incredibly strong bite make it
end-of-year trip to Helsinki. a force to be reckoned 34 about.

31 34

A) NO CHANGE A) NO CHANGE
B) into my head, B) at
C) overhead, C) with
D) out of my head, D) against

A good CEO would never jump down someone’s Leonardo Da Vinci was known 35 for his short fuse,
throat 32 in front of a group because such behavior perhaps due to his incredible perfectionism.
demonstrates weakness. It was apparently very hard to work as an apprentice in

32 his studio.

A) NO CHANGE 35
B) in front to A) NO CHANGE
C) on front of B) with
D) at front of C) about
D) of

Nothing gets 33 on my skin like when the grocer puts


stickers on fruit. After reading a book on the psychology of insecurity and

33 the subsequent behavioral manifestations, Talik became


indifferent 36 of the bully’s efforts to frighten him.
A) NO CHANGE
B) my 36
C) under my
A) NO CHANGE
D) out of my
B) at
C) with
D) to

CONTINUE
198 More Exercises on Conventional Expressions

Besides being beautiful to look at, the fantastic light Many federal agencies—for example, the Food and Drug
displays generated by auroras can also teach us a lot Administration or the Internal Revenue Service—have
about the interactions 37 between solar winds and the authority delegated 39 for Congress to accomplish their
Earth. goals; however, some political theorists question whether

37 the vague guiding principles of these goals result in an


unconstitutional imbalance of power.
A) NO CHANGE
B) for 39
C) against
A) NO CHANGE
D) about
B) with
C) about
D) by
Unwanted radio frequency signals, which can be
produced by the electrical motors in refrigerators or
heating pumps, have the potential to interfere 38 After vandals began to place chewing gum on the
against normal broadcasts. sensitive door sensors of a new, multibillion dollar train

38 system, the Prime Minister of Singapore decided to


prohibit citizens 40 for using or selling this popular
A) NO CHANGE
B) with product in 1992.
C) in 40
D) for
A) NO CHANGE
B) from
C) to
D) of

CONTINUE
More Exercises on Conventional Expressions 199

The plants grew 41 from close proximity to one The children’s eyes widened when they saw the table laid
another, which resulted in later issues in root out with an abundance 43 from delectable treats.
development. 43
41 A) NO CHANGE
A) NO CHANGE B) at
B) by C) of
C) with D) with
D) in

Even those who don’t care for the form of the villanelle
With reference 42 of our previous conversation, I wish take an interest 44 to Elizabeth Bishop’s poem “One
to remain in favor of environmental sanctions. Art.”

42 44

A) NO CHANGE A) NO CHANGE
B) to B) for
C) from C) in
D) about D) with

CONTINUE
200 More Exercises on Conventional Expressions

Prior to the advent 45 of digital fantasy football, the The CEO would say otherwise, but 47 of essence, her
hobby had a loyal, but not nearly so invested, following. basket company was a pyramid scheme.

45 47

A) NO CHANGE A) NO CHANGE
B) to B) from
C) in C) in
D) at D) under

When the only witness 46 on the $16 million jewelry Many people are 48 to the fact that graphic novels are
heist wouldn’t directly answer the lawyer’s questions, the not the comic books they read as kids.
judge ordered him to stop hedging. 48
46 A) NO CHANGE
A) NO CHANGE B) with
B) around C) onto
C) for D) about
D) to

CONTINUE
More Exercises on Conventional Expressions 201

The intensive cooking workshop Rachel attended offered James, the new medical resident, was afraid that the older
little instruction and was mostly a trial 49 in fire. doctors would walk all 51 over him.

49 51

A) NO CHANGE A) NO CHANGE
B) on B) under
C) by C) on
D) with D) across

After Governor Poltroon was caught lying about her Everyone thought that the executive’s speech was well
finances, she continued to lie instead of 50 coming rehearsed; they didn’t know she’d made it up 52 on the
clean. fly.

50 52

A) NO CHANGE A) NO CHANGE
B) going B) out
C) remaining C) above
D) leaving D) in

The plumber misplaced his tools so often, it seemed like


they vanished 53 by thin air.

53

A) NO CHANGE
B) with
C) through
D) into

CONTINUE
202 More Exercises on Conventional Expressions

After Bettina won the baking competition, she said it was During the Cold War, many Americans lived in fear that
as easy 54 by pie. the country would be forced to jump 57 in nuclear

54 combat with Russia.

A) NO CHANGE 57
B) at A) NO CHANGE
C) as B) for
D) of C) into
D) over

The boxer’s ascent to heavyweight champion was a


rags 55 and riches story. Even though she did not care for gardening, Lucy kept

55 her front yard immaculate and lined with rose bushes in


order to keep 58 up appearances.
A) NO CHANGE
B) over 58
C) to
A) NO CHANGE
D) by
B) on
C) the
D) at
“Cut it 56 over!” cried Jaden, shrinking back from the
snake his sister held.

56 Do not wallow 59 over your losses; learn from your


mistakes and try again!
A) NO CHANGE
B) about! 59
C) aside!
A) NO CHANGE
D) out!
B) at
C) in
D) onto

CONTINUE
More Exercises on Conventional Expressions 203

Instead of giving a dramatic speech, Indu chose to Scientists who work in primate language research
simplify her message and let the facts speak 60 with try to get 62 inside of the minds of chimpanzees,
themselves. bonobos, gorillas, and orangutans to teach them how to

60 communicate with humans.

A) NO CHANGE 62
B) before A) NO CHANGE
C) for B) inside
D) DELETE the underlined portion. C) toward
D) across from

Many current employers tell colleges that in today’s labor


market, “soft skills” like communication and leadership Walt Whitman’s poems are sometimes criticized for
are very much 61 on demand. being difficult to read, but, 63 on the other hand, some

61 authorities point to his unique style as a primary catalyst


for the formation of a truly “American” literature.
A) NO CHANGE
B) to 63
C) for
A) NO CHANGE
D) in
B) on the other side,
C) to the other hand,
D) from the other hand,

Although Zora Neale Hurston, Langston Hughes, and


Duke Ellington are some of the most famous names
from the Harlem Renaissance, many people worked 64
under the scenes to turn the 1920s movement into a
rebirth of African American arts.

64

A) NO CHANGE
B) around
C) behind
D) within

CONTINUE
204 More Exercises on Conventional Expressions

At the time of the 1929 stock market crash, finding The sushi lunch combo at the Japanese restaurant
causes for the crash was 65 beside the point because the consists 68 with yellowtail, salmon, and snapper.
nation was too consumed with the immediate fallout. 68
65 A) NO CHANGE
A) NO CHANGE B) from
B) beyond C) of
C) out of D) in
D) to

Based on her research, Marina thought it was likely that


Once they discover how much math is involved with the Herman Newton—not Isaac Newton—had discovered
occupation, many aspiring architects decide, 66 with universal gravitation; however, her hypothesis was
second thought, to pursue other design-based careers. inconsistent 69 to a well-established historical record.

66 69

A) NO CHANGE A) NO CHANGE
B) in B) for
C) for C) with
D) on D) from

The watchmaker was not overly concerned with time; 67


for the contrary, she was often exceptionally tardy for
appointments and gatherings.

67

A) NO CHANGE
B) on
C) against
D) under

CONTINUE
More Exercises on Conventional Expressions 205

Although it later disintegrated, both J. R. R. Tolkien Although the results of the enzyme assay supported the
and C. S. Lewis benefited 70 with a friendship that original hypothesis, these results were later dismissed 72
challenged both writers to reach new creative and as irrelevant when the lab manager discovered that one
intellectual heights. of the cell lines had been cross-contaminated.

70 72

A) NO CHANGE A) NO CHANGE
B) after B) toward
C) to C) from
D) from D) of

For people accustomed 71 in consuming caffeine, two


shots of espresso are unlikely to have a significant effect;
however, it may leave a first-time coffee drinker feeling
highly alert or even slightly nauseated.

71

A) NO CHANGE
B) to
C) for
D) with

CONTINUE

You might also like